You are on page 1of 156

2007 May Sunday SAT Answers and Explanations

2007 May Sunday SAT (Online Course Test2)

1. Collegeboard SAT Online Course Collegeboard


2. 2 5 EssayEssay Explanations
3. CR WR
4.
5. CUUS RP5000
6.esus

Section1: Essay
Essay Prompt

Think carefully about the issue presented in the following excerpt and the assignment below.

We do not take the time to determine right from wrong. Reflecting on the difference
between right and wrong is hard work. It is so much easier to follow the crowd, going along
with what is popular rather than risking the disapproval of others by voicing an objection of
any kind.

Adapted from Stephen J. Carter, Integrity

Assignment:
Is it always best to determine one's own views of right and wrong, or can we benefit from
following the crowd? Plan and write an essay in which you develop your point of view on this
issue. Support your position with reasoning and examples taken from your reading, studies,
experience, or observations.

Sample Essay - Score of 6

Some people may say it is easier to follow the majority, but from my expierences I have
found that it is of utmost importance that a person follows what they believe in.
Sometimes, out of fear of rejection, people will give up their own beliefs in order to avoid
being ostracized or excluded from a group. While this may seem like a harmless thing, it
can have dire consequences. By looking at the situation in Nazi Germany during the
Holocaust, it is easy to see the negative effects of following the crowd.

During the early 1930s, in the midst of an economic depression in Germany, a new figure
came into power that gave the Germans a group of people to unite against. This new
leader, Adolf Hitler, used his clever speeches and fancy parades to dupe the German
population into believing that the cause of their trouble was the Jews that lived amongst
them. At first, no one raised an objection to Hitler s accusations. People believed that he
was harmless, and that there was no way he could achieve total power over the publics
views. Hitler saw this advantage, as he realized no one was willing to stand up to him

From SAT Online Course CUUS


2007 May Sunday SAT Answers and Explanations

because of the threat of a Second World War. He annexed more territory illegally, knowing
that no one would say anything out of fear. The United States and other countries followed
the policy of appeasement, which basically let Germany do what it wanted in an effor to
avoid war.

Were it not for the international communitys inability to act, Hitler could have been
stopped early on. Instead, people followed the majority and let Hitler gradually gain more
and more power as he built his dictatorial regime. This allowed him to build his
concentration camps, where millions of Jews were sent and torn apart from their families.
While there were some brave people in Germany, most of them failed to challenge Hitler,
and by the end of the war, he had killed over six million Jews.

In the case of the Holocaust, there were multiple opportunities where people could have
challenged Hitler s rise to power by openly voicing their criticism of his policies. However,
instead of speaking up in the face of injustice, the nations of the world and the German
citizens kept quiet, as they followed the crowd and watched a totalitarian take power.

Score Explanation
This essay demonstrates outstanding critical thinking by effectively and insightfully
developing the point of view that it is of utmost importance that a person follows what
they believe in. Well organized and clearly focused as it uses the clearly appropriate
historical example of the situation in Nazi Germany during the Holocaust to illustrate the
negative effects of following the crowd, the response displays clear coherence and smooth
progression of ideas (Were it not for the international communitys inability to act, Hitler
could have been stopped early on. . . . While there were some brave people in Germany,
most of them failed to challenge Hitler, and by the end of the war, he had killed over six
million Jews). Using a varied, accurate, and apt vocabulary, the response exhibits skillful
use of language (This new leader, Adolf Hitler, used his clever speeches and fancy parades
to dupe the German population into believing that the cause of their trouble was the Jews
that lived amongst them), and there is meaningful variety in sentence structure
throughout. Demonstrating clear and consistent mastery, this essay receives a score of 6.

Sample Essay - Score of 5

I dont feel that anyone can benefit from just following the crowd. Those who just go
along with what others say and do can never truly accomplish what they want in life. I
think it is very important to take the time to voice your opinion and stick up for what
you believe in. It may be harder, but its the only way to be true to yourself. I know
that it is worth the time to voice your opinion from past experiences.

One experience where I had to stick up for myself was in seventh grade. All of my
girlfriends wanted to wear mini skirts to the youth group dance and it wasnt allowed.
The most persuasive girl, the leader of the pack you could say, tried to drill it in our
minds that wearing a skirt was the only way you could look good. All of the other girls
went along with her scheme and thought if they all wore one they wouldnt get in
trouble. I, however, knew better. I was aware that anyone dressed inappropriately

From SAT Online Course CUUS


2007 May Sunday SAT Answers and Explanations

would be kicked out of the dance so I wore pants. We showed up to the dance and
right away the girls were reprimanded. The girls sadly left our school gym while I
danced the night away with some other friends. I slowly stopped hanging out with this
group of girls and started hanging out with some people I spent some time with at the
dance. This new group of people are to this day my best friends. They are the type of
people who are always there for me and I wouldnt trade them for anyone in the world.
By sticking up for myself and not wearing a skirt, I not only saved myself from being
kicked out of the dance, but I met my best friends.

Times like my seventh grade dance remind me how important it is to stick up for
yourself, even if it is hard. If you are always following someone else, you are never
forming your own ideas and in the future you will not know what to do if you have to
face a challenge on your own. The most successful people today have gotten where
they are because they voiced their opinions. I hope in the future I can use this skill
when Im faced with dilemmas.

Score Explanation
Demonstrating strong critical thinking, this essay effectively develops a point of view (I
think it is very important to take the time to voice your opinion and stick up for what
you believe in) and uses an appropriate example from personal experience as support.
Focused on the story of my seventh grade dance, the well-organized response
displays coherence and progression of ideas (All of the other girls went along with her
scheme . . . . By sticking up for myself . . . I not only saved myself from being kicked
out of the dance, but I met my best friends). This essay exhibits facility in the use of
language, using appropriate vocabulary, and variety in sentence structure is evident
(The most persuasive girl, the leader of the pack you could say, tried to drill it in our
minds that wearing a skirt was the only way you could look good). To earn a score of 6,
the writer should provide further support for the point of view, perhaps by describing
specific people who have gotten where they are because they voiced their opinions.
The personal example could also be enhanced with additiona l details. This essay
demonstrates reasonably consistent mastery and merits a score of 5.

Sample Essay - Score of 5

Throughout my highschool years at Mother McAuley High School, I have found that
following the crowd is the easiest, but the experience is not fulfilling. Taking risks and
making your own choices is what makes you real and determines who you are as a
person. Going against the crowd and voicing your own words, thoughts, and opinions,
as my mother would say, makes you the brightest crayon in the box.

During my first semester, my english class read the Adventures of Huckleberry Finn by
Mark Twain. In that strong novel we encountered Huck, a young boy searching for his
own freedom. Huck grew up in a noneducated, slave-holding town, and was raised to
believe that slaves are just mere property, nothing else. As the story evolves, a major
delemma occurs where Huck has to follow his heart instead of what society wants him
to do and help a slave named Jim. As I was half-way through with the book, I realized
that society is all wrong. Who is to say what we should and should not do? Just

From SAT Online Course CUUS


2007 May Sunday SAT Answers and Explanations

because the town where Huck grew up says it has to be this way does not mean it is so.
No one should go against the will of their heart and everyone should have the liberty
to choose for themselves what is right and wrong.

Although standing apart from the crowd makes you different, following the crowd has
its benefits too. In my teenage years I have experienced the roles of being a leader
and a follower and I have to say I do not see anything wrong with that. With different
groups of friends I have become to understand myself and emerge into my own
identity. I have realized changes in myself when I would be a leader, I was more
confident and able, but also as a follower I became a student. I learned new things and
I started to form my beliefs and values along the decisions I thought were right and I
have become a beautiful woman because of that.

So, I do believe that following the crowd can be helpful in forming your individual self,
but being a leader and going against the current makes it all worth while because if you
never try then youll never know.

Score Explanation
This essay demonstrates strong critical thinking by effectively developing a point of
view (following the crowd can be helpful in forming your individual self, but being a
leader and going against the current makes it all worth while ) and using appropriate
reasons and examples from literature and personal experience as support. Well
organized and focused, the essay displays coherence and progression of ideas (Huck
grew up in a noneducated, slave-holding town, and was raised to believe that slaves
are just mere property, nothing else. As the story evolves, a major delemma occurs
where Huck has to follow his heart instead of what society wants him to do and help a
slave named Jim. As I was half-way through with the book, I realized that society is all
wrong). Through the use of appropriate vocabulary, the response displays facility in the
use of language (Going against the crowd and voicing your own words, thoughts, and
opinions, as my mother would say, makes you the brightest crayon in the box), and
variety in sentence structure is evident. In order to achieve a score of 6, the writer
needs to add more detailed evidence to enhance the personal example provided, as
well as improve his or her control of grammar, usage, and mechanics. Displaying
reasonably consistent mastery, this essay receives a score of 5.

Section4: Critical Reading

View Answers and Explanations

Online - Practice Test #2

From SAT Online Course CUUS


2007 May Sunday SAT Answers and Explanations

1 The engineers heard the claims about the new software with ------- , but the
demonstration of the software was so ------- that it won them over.

(A) enthusiasm . . comprehensive


(B) skepticism . . convincing
(C) excitement . . lackluster
(D) apathy . . routine
(E) hesitation . . inconclusive

ANSWERS AND EXPLA NATIONS

Explanation for Correct Answer B :

Choice (B) is correct. Skepticism is an attitude of doubt or disbelief toward a


particular object. If something is convincing, it is persuasive. The part of the sentence
after the comma indicates that the software demonstration won the engineers over; it
makes sense to describe the demonstration as convincing if it was able to persuade the
engineers to believe the claims about the new software. Because the engineers had to
be persuaded, it is logical to assume that they initially did not believe the claims about
the software. The term skepticism describes an attitude of disbelief, so it makes
sense to describe the engineers as skeptical before they saw the demonstration.

Explanation for Incorrect Answer A :

Choice (A) is incorrect. Enthusiasm is a strong feeling of excitement. If something is


comprehensive, it is inclusive, covering something completely. The part of the
sentence after the comma indicates that the software demonstration won the engineers
over. It makes sense to suggest that the demonstration was comprehensive, because a
demonstration that is complete and thorough might have persuaded the engineers to
believe the claims about the new software. However, the term enthusiasm does not
logically complete the sentence. Because the engineers had to be won over, it is logical
to assume that they initially did not believe the claims about the software. Therefore, it
does not make sense to suggest that the engineers were enthusiastic about the
software before they saw the demonstration.

Explanation for Incorrect Answer C :

Choice (C) is incorrect. Excitement is a state of being excited about, or having a


strong emotional response to, something. If something is lackluster, it is dull or
mediocre. The part of the sentence after the comma indicates that the software
demonstration won the engineers over; it is unlikely that a lackluster, or dull,
demonstration would have won them over. Further, because the engineers had to be
won over, it is logical to assume that they initially did not believe the claims about the
software. Therefore, it does not make sense to suggest that the engineers were excited
about the software before they saw the demonstration.

From SAT Online Course CUUS


2007 May Sunday SAT Answers and Explanations

Explanation for Incorrect Answer D :

Choice (D) is incorrect. Apathy is a lack of interest or concern. In this context,


routine describes something that is ordinary or commonplace. The part of the
sentence after the comma indicates that the software demonstration won the engineers
over. If the engineers needed to be won over, it is possible that they were initially
apathetic about the new softwarein other words, they might have had no interest in it.
However, it is unlikely that an ordinary, routine demonstration would have won over a
group of uninterested engineers; in fact, an ordinary demonstration might have
reinforced their feelings of apathy.

Explanation for Incorrect Answer E :

Choice (E) is incorrect. To hesitate is to hold back in doubt or indecision. If something is


inconclusive, it leads to no definite conclusion or result. The part of the sentence after
the comma indicates that the software demonstration won the engineers over. If the
engineers needed to be won over, it is possible that they were initially hesitantthat is,
it could be that they had doubts or had not yet come to a conclusion about the new
software. However, it is unlikely that an inconclusive demonstration would have won
them over; on the contrary, a demonstration that offered no definitive results might
have caused the engineers to be even more hesitant to believe the claims about the
software.

2 Because the candidate had switched his party allegiance immediately before the
campaign, his former associates called him a ------- , and even his new allies
considered him an ------- .

(A) recruit . . expert


(B) strategist . . imbecile
(C) deserter . . inspiration
(D) prophet . . interloper
(E) traitor . . opportunist

ANSWERS AND EXPLA NATIONS

Explanation for Correct Answer E :

Choice (E) is correct. A "traitor" is someone who betrays anothers trust or is not
faithful to commitments or duties. An "opportunist" is someone who takes advantage of
opportunities without concern about principles or consequences. The sentence indicate s
that the candidate "switched his party allegiance immediately before the campaign." He
could certainly be seen as a traitor; the candidates "former associates" might have
thought he was never genuinely committed to his party in the first place. He could also
be seen as an opportunist; the candidates "new allies" might have felt that he took adv

From SAT Online Course CUUS


2007 May Sunday SAT Answers and Explanations

antage of an opportunity (switching parties) without worrying about the consequences


(being seen as a traitor).

Explanation for Incorrect Answer A :

Choice (A) is incorrect. In this context, a "recruit" is a new member of an organization


or group. An "expert" is someone with special skills or knowledge representing mastery
of a particular subject. The sentence indicates that the candidate "switched his party
allegiance immediately before the campaign." There is no reason to believe that the
candidates "former associates" would have called him a recruithe was leaving them,
not joining them. In fact, the candidates "new allies" would have been more likely to
call him a recruit, because he became a new member of their party. Further, there is no
reason to believe that the candidates new allies would have considered him to be an
expert; there is no indication that the candidate had special skills or knowledge, only
that he switched parties.

Explanation for Incorrect Answer B :

Choice (B) is incorrect. A "strategist" is someone who is skilled in strategy, or in


devising and employing plans. An "imbecile" is a fool or idiot. The sentence indicates
that the candidate "switched his party allegiance immediately before the campaign."
The candidates "former associates" might have considered him to be a strategistthey
may have thought that the candidates decision to change parties was part of a larger
plan. However, there is no reason to believe that the candidates "new allies" would
have described him as an imbecile; nothing in the sentence suggests that the candidate
is an idiot, and members of a party would not necessarily consider someone foolish for
joining that party.

Explanation for Incorrect Answer C :

Choice (C) is incorrect. A "deserter" is someone who abandons a position, often leaving
others in the lurch. In this context, an "inspiration" is someone who exerts a positive
influence on others or motivates them. The sentence indicates that the candidate
"switched his party allegiance immediately before the campaign." It is likely that the
candidates "former associates" saw him as a deserter; the candidate abandoned their
party. However, there is no reason to believe that the candidates "new allies" would
have described him as an inspiration; there is no indication that the candidate
motivates others or has a positive influence, only that he switched parties.

Explanation for Incorrect Answer D :

Choice (D) is incorrect. A "prophet" is someone who has the ability to see or predict
future events. An "interloper" is someone who intrudes in an activity. The sentence
indicates that the candidate "switched his party allegiance immediately before the
campaign." It is possible that the candidates "new allies" considered him to be an interl

From SAT Online Course CUUS


2007 May Sunday SAT Answers and Explanations

oper; they may not have wanted the candidate to join their party, so they may have
seen him as an intruder. But there is no reason to believe that the candidate is a
prophet, or that his "former associates" would have described him as one. There is no
indication that the candidate has the ability to see or predict future events.

3 All copies of Verna Hart s poster commemorating the 1992 Charlotte Ja zz Festival
were sold in a few hours, an unmistakable ------- of the ------- of this artists work.

(A) refutation . . popularity


(B) investigation . . value
(C) demonstration . . mediocrity
(D) duplication . . durability
(E) indication . . appeal

ANSWERS AND EXPLA NATIONS

Explanation for Correct Answer E :

Choice (E) is correct. An indication is something that acts as a sign or as evidence for
the existence of something else. If something has appeal, it is attractive or pleasing.
The structure of the sentence suggests that the part of the sentence after the comma
elaborates on the information presented in the first part of the sentence. Because
Verna Harts poster sold out very quickly, it makes sense to say that her work is
appealing; Harts work is clearly attractive to buyers. Additionally, the fact that all
copies of the poster were sold so quickly is an unmistakable indicationa clear
signof the appeal of Harts work.

Explanation for Incorrect Answer A :

Choice (A) is incorrect. A refutation shows that something is false or erroneous. If


something has popularity, it is popular, or commonly liked. The structure of the
sentence suggests that the part of the sentence after the comma elaborates on the
information presented in the first part of the sentence. Verna Harts poster may have
sold out very quickly because her work is popular, or liked by many buyers. However,
the fact that all copies of the poster were sold so quickly is not an unmistakable
refutation of the popularity of Harts work; rather than showing that Harts popularity
is somehow false, the successful sale of the poster demonstrates that Harts work is pop
ular.

Explanation for Incorrect Answer B :

Choice (B) is incorrect. An investigation is a systemic examination. If something has


value, it has relative worth or importance. The structure of the sentence suggests tha

From SAT Online Course CUUS


2007 May Sunday SAT Answers and Explanations

t the part of the sentence after the comma elaborates on the information presented in
the first part of the sentence. Verna Harts poster sold out very quickly, so it makes
sense to say that her work has value, or relative importance, to buyers. However, the
fact that all copies of the poster were sold so quickly is not an unmistakable
investigation; the sale of a poster has nothing to do with an investigation, or a careful
examination, of anything.

Explanation for Incorrect Answer C :

Choice (C) is incorrect. In this context, to demonstrate something is to show it clearly.


If something is mediocre, it is of moderate or low quality. The structure of the sentence
suggests that the part of the sentence after the comma elaborates on the information
presented in the first part of the sentence. The fact that Verna Harts poster sold out
very quickly might be a clear demonstration of something about Harts workfor
example, that buyers find it appealing. But it is somewhat illogical to sugg est that the
successful sale of Harts poster demonstrates the mediocrity of her work; if Harts
work is of moderate or low quality, it likely would not sell quickly.

Explanation for Incorrect Answer D :

Choice (D) is incorrect. A duplication is an identical copy of something. If something


is durable, it can exist for a long time without much deteriorationin other words, it is
long-lasting. The structure of the sentence suggests that the part of the sentence after
the comma elaborates on the information presented in the first part of the sentence.
The terms duplication and durability do not logically complete the sentence. It does
not make sense to describe the sale of something as a duplication, or an identical copy,
and nothing in the sentence indicates that Vera Harts work is durable. Further, the fact
that all copies of Harts poster were sold very quickly does not have anything to do with
the idea that Harts work can exist for a long time.

4 Although Virginia Woolf was a central figure in Londons established literary scene,
she nevertheless perceived herself as being ------- .

(A) renowned
(B) permissive
(C) rational
(D) salutary
(E) marginal

ANSWERS AND EXPLA NATIONS

Explanation for Correct Answer E :

Choice (E) is correct. The term although suggests that Virginia Woolf saw herself in a

From SAT Online Course CUUS


2007 May Sunday SAT Answers and Explanations

way that did not reflect her actual position as a central figure in Londons established
literary scene; therefore, the missing term should have something to do with not being
a central part ofor of primary importance tosomething. The term marginal
logically completes the sentence because it describes someone who is not of central
importance. Woolf perceived herself as marginal to Londons literary scene even though
she was actually a central figure.

Explanation for Incorrect Answer A :

Choice (A) is incorrect. The term although suggests that Virginia Woolf saw herself in
a way that did not reflect her actual position as a central figure in Londons established
literary scene; therefore, the missing term should have something to do with not being
a central part ofor of primary importance tosomething. The term renowned does
not logically complete the sentence, because it describes someone who is celebrated or
famous, and such a person might be considered to be of primary importance. If Woolf
saw herself as a celebrated person, she might have considered herself central to
Londons literary scene.

Explanation for Incorrect Answer B :

Choice (B) is incorrect. The term although suggests that Virginia Woolf saw herself in
a way that did not reflect her actual position as a central figure in Londons established
literary scene; therefore, the missing term should have something to do with not being
a central part ofor of primary importance tosomething. The term permissive does
not logically complete the sentence because it describes someone who does not
maintain great controlthat is, someone who is tolerant or lenient; there is not
necessarily a connection between being permissive and not being of primary
importance.

Explanation for Incorrect Answer C :

Choice (C) is incorrect. The term although suggests that Virginia Woolf saw herself in
a way that did not reflect her actual position as a central figure in Londons established
literary scene; therefore, the missing term should have something to do with not being
a central part ofor of primary importance tosomething. The term rational does not
logically complete the sentence because it describes someone who behaves in a
manner based on reason; there is not necessarily a connection between being rational
and not being of primary importance.

Explanation for Incorrect Answer D :

Choice (D) is incorrect. The term although suggests that Virginia Woolf saw herself in
a way that did not reflect her actual position as a central figure in Londons established
literary scene; therefore, the missing term should have something to do with not being
a central part ofor of primary importance tosomething. The term salutary does no

From SAT Online Course CUUS


2007 May Sunday SAT Answers and Explanations

t logically complete the sentence because it describes something that produces a


beneficial effect or is curative; it is somewhat illogical to describe a person as being
salutary. Further, there is not necessarily a connection between producing a beneficial
effect and not being of primary importance.

5 Although strangers compliment the comedian on her work repeatedly, almost------


-, she is always moved by such enthusiastic encouragement.

(A) tenuously
(B) staunchly
(C) singularly
(D) incessantly
(E) inimitably

ANSWERS AND EXPLA NATIONS

Explanation for Correct Answer D :

Choice (D) is correct. The structure of the sentence indicates that the missing term is
similar in meaning tobut stronger thanthe term repeatedly, and elaborates on the
idea that strangers compliment the comedian on her work over and over again. The
term incessantly logically completes the sentence because something that is
incessant is unceasing or continual; strangers compliment the comedian over and over
again, almost without ceasing.

Explanation for Incorrect Answer A :

Choice (A) is incorrect. Something that is tenuous is shaky and lacks strength. The
structure of the sentence indicates that the missing term is similar in meaning tobut
stronger thanthe term repeatedly, and elaborates on the idea that strangers
compliment the comedian on her work over and over again. The term tenuously does
not logically complete the sentence; being complimented shakily is not similar to being
complimented repeatedly. Further, it is somewhat illogical to suggest that one would
describe weak, tenuous compliments as enthusiastic encouragement.

Explanation for Incorrect Answer B :

Choice (B) is incorrect. Someone who is staunch is extremely loyal or faithful. The
structure of the sentence indicates that the missing term is similar in meaning tobut
stronger thanthe term repeatedly, and elaborates on the idea that strangers
compliment the comedian on her work over and over again. Strangers who appreciate
the comedians work might offer compliments or enthusiastic encouragement as an
expression of their loyalty, but it does not really make sense to describe the manner in
which the comedian is complimented as almost staunch. Further, the term staunchly i

From SAT Online Course CUUS


2007 May Sunday SAT Answers and Explanations

s not similar in meaning to the term repeatedly.

Explanation for Incorrect Answer C :

Choice (C) is incorrect. In this context, something that is singular is unusual or out of
the ordinary. The structure of the sentence indicates that the missing term is similar in
meaning tobut stronger thanthe term repeatedly, and elaborates on the idea that
strangers compliment the comedian on her work over and over again. Strangers
frequently compliment the comedian, so it does not make sense to suggest that these
compliments are almost singular; rather than being out of the ordinary, the strangers
compliments actually are ordinary, or commonly occurring.

Explanation for Incorrect Answer E :

Choice (E) is incorrect. In this context, something that is inimitable is matchless, or has
no equal. The structure of the sentence indicates that the missing term is similar in
meaning tobut stronger thanthe term repeatedly, and elaborates on the idea that
strangers compliment the comedian on her work over and over again. It is possible
that some individuals have given the comedian compliments that have never been
matched by others, but the term inimitably does not logically complete the sentence;
being complimented in a way that cannot be matched is not similar to being
complimented repeatedly.

6 In contrast to the ------- maneuvers of his colleagues, Robertos business relations


were always open and aboveboard.

(A) convivial
(B) clandestine
(C) steadfast
(D) fortuitous
(E) frank

ANSWERS AND EXPLA NATIONS

Explanation for Correct Answer B :

Choice (B) is correct. If something is clandestine, it is done in secrecy. The sentence


indicates that Robertos business relations contrasted with the maneuvers of his
colleagues. Robertos business dealings were open and aboveboardthat is, they
were done openly and straightforwardlyso it makes sense to say that, in contrast, his
colleagues maneuvers were clandestine, or done secretly.

Explanation for Incorrect Answer A :

From SAT Online Course CUUS


2007 May Sunday SAT Answers and Explanations

Choice (A) is incorrect. If something is convivial, it is related to feasting, drinking,


and good company. The sentence indicates that Robertos business relations
contrasted with the maneuvers of his colleagues. Convivial maneuversmaneuvers
conducted with companymight be conducted in a way that is open and aboveboard.
Therefore, Robertos colleagues convivial maneuvers would not necessarily contrast
with his own open and straightforward business dealings.

Explanation for Incorrect Answer C :

Choice (C) is incorrect. If something is steadfast, it is not subject to change. The


sentence indicates that Robertos business relations contrasted with the maneuvers
of his colleagues. Steadfast maneuversmaneuvers that stay the same and do not
changemight be conducted in a way that is open and aboveboard. Therefore,
Robertos colleagues steadfast maneuvers would not necessarily contrast with his own
open and straightforward business dealings.

Explanation for Incorrect Answer D :

Choice (D) is incorrect. If something is fortuitous, it occurs by a lucky chance. The


sentence indicates that Robertos business relations contrasted with the maneuvers
of his colleagues. Maneuvers that are fortuitousthose that occur by chance and turn
out to be luckymight be conducted in a way that is open and aboveboard. Therefore,
Robertos colleagues fortuitous maneuvers would not necessarily contrast with his own
open and straightforward business dealings.

Explanation for Incorrect Answer E :

Choice (E) is incorrect. If something is frank, it is characterized by clarity and


sincerity in expression. The sentence indicates that Robertos business relations
contrasted with the maneuvers of his colleagues. Frank maneuversmaneuvers
characterized by sinceritywould likely be conducted in an open and aboveboard way.
Therefore, Robertos colleagues frank maneuvers would not necessarily contrast with
his own open and straightforward business dealings.

7 This medical study shuns ------- in describing the drugs dangers; the appeal is to
rational evaluation rather than to fea r.

(A) obfuscation
(B) certitude
(C) sensationalism
(D) piety
(E) plausibility

ANSWERS AND EXPLA NATIONS

From SAT Online Course CUUS


2007 May Sunday SAT Answers and Explanations

Explanation for Correct Answer C :

Choice (C) is correct. Sensationalism is the use of sensational subject matter, or


subject matter that produces emotional reactions. The sentence indicates that the
medical study presents the dangers of a drug but does not appeal . . . to fear. Fear is
certainly an emotional reaction, so it makes sense to say that the study shuns
sensationalism, or the use of subject matter that will produce emotional reactions.
Instead of appealing to people's emotions, the study appeals to rational
evaluationappraisals based on reason.

Explanation for Incorrect Answer A :

Choice (A) is incorrect. Obfuscation is the process of making something obscure or


confusing. The structure of the sentence indicates that the idea in the second part of
the sentence is connected to the idea in the first part of the sentencein other words,
the medical study shuns something because it chooses to appeal to rational
evaluation, or appraisals based on reason, rather than to fear. Those conducting a
medical study certainly might shun obfuscation; they would likely want people to
understand their findings, so they probably would not choose to make certain
descriptions confusing. However, there is no logical connection between avoiding
making something confusing and appealing to rational evaluation instead of fear, so the
term obfuscation does not logically complete the sentence.

Explanation for Incorrect Answer B :

Choice (B) is incorrect. Certitude is the state of being or feeling certain. The structure
of the sentence indicates that the idea in the second part of the sentence is connected
to the idea in the first part of the sentencein other words, the medical study shuns
something because it chooses to appeal to rational evaluation, or appraisals based on
reason, rather than to fear. Although the people conducting the study might not yet
be certain of the drugs dangers, it is somewhat illogical to suggest that they would
shun certitude; on the contrary, it is more likely that those conducting the study would
want to feel certain when presenting the drugs dangers. Further, there is no logical
connection between avoiding feeling certain and appealing to rational evaluation
instead of fear.

Explanation for Incorrect Answer D :

Choice (D) is incorrect. Piety is the quality of being pious, or loyally reverent or
virtuous. The structure of the sentence indicates that the idea in the second part of the
sentence is connected to the idea in the first part of the sentencein other words, the
medical study shuns something because it chooses to appeal to rational evaluation, or
appraisals based on reason, rather than to fear. It does not make sense to suggest
that the people conducting the study would shun piety; there is no reason to believe
that they would want to avoid being virtuous. Further, there is no logical connection bet

From SAT Online Course CUUS


2007 May Sunday SAT Answers and Explanations

ween avoiding being pious and appealing to rational evaluation instead of fear.

Explanation for Incorrect Answer E :

Choice (E) is incorrect. Plausibility is the quality of being plausible, or appearing


worthy of belief. The structure of the sentence indicates that the idea in the second part
of the sentence is connected to the idea in the first part of the sentencein other
words, the medical study shuns something because it chooses to appeal to rational
evaluation, or appraisals based on reason, rather than to fear. It is somewhat
illogical to suggest that the people conducting the study would shun plausibility; on the
contrary, it is more likely that those conducting the study want people to understand
the drug s dangers, so they would want their descriptions to be seen as worthy of belief.
Further, there is no logical connection between avoiding being plausible and appealing
to rational evaluation instead of fear.

8 Alexis complained that Jim ------- too quickly when their parents imposed a curfew:
instead of negotiating, he complied without protest.

(A) remonstrated
(B) capitulated
(C) compromised
(D) interceded
(E) equivocated

ANSWERS AND EXPLA NATIONS

Explanation for Correct Answer B :

Choice (B) is correct. To capitulate is to surrender or cease resisting. The structure of


the sentence indicates that the part of the sentence after the colon further explains the
idea presented in the first part. The term complied means acted in accordance with
anothers command, request, or rule. Because Jim complied without protest instead
of negotiating when their parents set a curfew, Alexis complained about his behavior.
If Alexis expected Jim to arrange a compromise with their parents and he instead
submitted to their request without complaint, it makes sense to suggest that Alexis
accused Jim of capitulating, or surrendering, too quickly.

Explanation for Incorrect Answer A :

Choice (A) is incorrect. To remonstrate is to argue in protest or opposition. The


structure of the sentence indicates that the part of the sentence after the colon further
explains the idea presented in the first part. The term complied means acted in
accordance with anothers command, request, or rule. Because Jim complied without
protest instead of negotiating when their parents set a curfew, Alexis complained ab

From SAT Online Course CUUS


2007 May Sunday SAT Answers and Explanations

out his behavior. It does not make sense to suggest that Jim remonstrated too
quickly, because remonstrations are arguments in protest of something and the
sentence clearly indicates that Jim complied without protest.

Explanation for Incorrect Answer C :

Choice (C) is incorrect. To compromise is to come to an agreement by mutual


concession. The structure of the sentence indicates that the part of the sentence after
the colon further explains the idea presented in the first part. The term complied
means acted in accordance with anothers command, request, or rule. Because Jim
complied without protest instead of negotiating when their parents set a curfew,
Alexis complained about his behavior. It does not make sense to suggest that Jim
compromised too quickly, because compromising is similar to negotiating both
involve coming to a mutual agreementand the sentence clearly indicates that Jim did
not negotiate with his parents.

Explanation for Incorrect Answer D :

Choice (D) is incorrect. To intercede is to mediate, or to intervene between parties with


the intent to reconcile differences. The structure of the sentence indicates that the part
of the sentence after the colon further explains the idea presented in the first part. The
term complied means acted in accordance with another s command, request, or rule.
Because Jim complied instead of negotiating when their parents set a curfew, Alexis
complained about his behavior. If Jim had interceded, he likely would have tried to
reconcile the differences between Alexis and their parentsthis might be seen as
negotiating, or trying to reach a compromise, and the sentence indicates that Jim did n
ot negotiate. Further, there is no reason to believe that Alexis would accuse Jim of
interceding because he complied without protest, or submitted to his parents request
without complaint; there is no logical connection between interceding and submitting to
a request.

Explanation for Incorrect Answer E :

Choice (E) is incorrect. To equivocate is to use equivocal languagelanguage that is


subject to multiple interpretations and usually used to mislead or confuse. The
structure of the sentence indicates that the part of the sentence after the colon further
explains the idea presented in the first part. The term complied means acted in
accordance with anothers command, request, or rule. Because Jim complied instead
of negotiating when their parents set a curfew, Alexis complained about his behavior.
Nothing in the sentence suggests that Jim used language intended to mislead or
confuse anyone; Jim merely complied without protestsubmitted to his parents
request without complaintso there is no reason to believe that Alexis would accuse
him of equivocating.

From SAT Online Course CUUS


2007 May Sunday SAT Answers and Explanations

Scientifically speaking, it seems clear why dogs act


like dogs. Like most domesticated animals, dogs are the
descendants of an intensely social species. Their instinct
Line to seek the company of human beings is an attempt to
5 reconstitute an ancestral social structure that is etched in
their genes. But cats defy all normal rules about domes-
ticated animals. The cats wild progenitor, the African
wildcat, is completely solitary in its natural state. I sup-
pose this explains why Ive never seen any Seeing Eye
10 cats, Frisbee-catching cats, or slipper-f etching cats. But
by the same reasoning, there ought to be no lap-sitting
cats or treat-begging catsand certainly no cats like
Shawn, my orange barn cat.

9 Which of the following would be the LEAST appropriate to add to the list in
lines 9-10?

(A) search-and-rescue cats


(B) sled-pulling cats
(C) luggage-sniffing cats
(D) mouse-hunting cats
(E) sheep-herding cats

ANSWERS AND EXPLA NATIONS

Explanation for Correct Answer D :

Choice (D) is correct. The author is making the point that, unlike dogs, cats are solitary
by nature. The author notes that cats are descended from a "completely solitary"
animal, and suggests that this ancestry explains why he or she has never seen any
Seeing Eye cats, Frisbee-catching cats, or slipper-fetching cats." Each of these activities
is a social, human-related activity associated with dogs; the author implies that cats do
not perform them because cats are solitary and independent, not social. Of the choices,
mouse hunting is that only one that is not a social activity associated with dogsand
the only one that cats are known to perform regularly. Mouse-hunting would thus be
inappropriate to add to the list.

Explanation for Incorrect Answer A :

Choice (A) is incorrect. The author is making the point that, unlike dogs, cats are
solitary by nature. The author notes that cats are descended from a "completely
solitary" animal, and suggests that this ancestry explains why he or she has never
seen any Seeing Eye cats, Frisbee-catching cats, or slipper-fetching cats." Each of
these activities is a social, human-related activity associated with dogs; the author impl

From SAT Online Course CUUS


2007 May Sunday SAT Answers and Explanations

ies that cats do not perform them because cats are solitary and independent, not social.
Search and rescue is a social, human-related activity associated with dogs, not cats. It
would thus be quite appropriate to add to the list.

Explanation for Incorrect Answer B :

Choice (B) is incorrect. The author is making the point that, unlike dogs, cats are
solitary by nature. The author notes that cats are descended from a "completely
solitary" animal, and suggests that this ancestry explains why he or she has never
seen any Seeing Eye cats, Frisbee-catching cats, or slipper-fetching cats." Each of
these activities is a social, human-related activity associated with dogs; the author
implies that cats do not perform them because cats are solitary and independent, not
social. Sled pulling is a social, human-related activity associated with dogs, not cats. It
would thus be quite appropriate to add to the list.

Explanation for Incorrect Answer C :

Choice (C) is incorrect. The author is making the point that, unlike dogs, cats are
solitary by nature. The author notes that cats are descended from a "completely
solitary" animal, and suggests that this ancestry explains why he or she has never
seen any Seeing Eye cats, Frisbee-catching cats, or slipper-fetching cats." Each of
these activities is a social, human-related activity associated with dogs; the author
implies that cats do not perform them because cats are solitary and independent, not
social. Luggage sniffing is a social, human-related activity associated with dogs, not
cats. It would thus be quite appropriate to add to the list.

Explanation for Incorrect Answer E :

Choice (E) is incorrect. The author is making the point that, unlike dogs, cats are
solitary by nature. The author notes that cats are descended from a "completely
solitary" animal, and suggests that this ancestry explains why he or she has never
seen any Seeing Eye cats, Frisbee-catching cats, or slipper-fetching cats." Each of
these activities is a social, human-related activity associated with dogs; the author
implies that cats do not perform them because cats are solitary and independent, not
social. Sheep herding is a social, human-related activity associated with dogs, not cats.
It would thus be quite appropriate to add to the list.

10 The author of the passage implies that Shawn is very

(A) curious
(B) independent
(C) aggressive
(D) companionable

From SAT Online Course CUUS


2007 May Sunday SAT Answers and Explanations

(E) mischievous

ANSWERS AND EXPLA NATIONS

Explanation for Correct Answer D :

Choice (D) is correct. In lines 1-10 the author develops a line of reasoning that an
animals ancestry determines its behavior: since cats are descended from a completely
solitary animal, the African wildcat, they are solitary as well. The author then states
that by the same reasoning, there ought to be . . . no cats like Shawn. If the
existence of cats like Shawn poses a challenge to this reasoning, then Shawn must not
be solitary. The reader can infer that Shawn is companionable, or sociable.

Explanation for Incorrect Answer A :

Choice (A) is incorrect. In lines 1-10 the author develops a line of reasoning that an
animals ancestry determines its behavior: since cats are descended from a completely
solitary animal, the African wildcat, they are solitary as well. The author then states
that by the same reasoning, there ought to be . . . no cats like Shawn. If the
existence of cats like Shawn poses a challenge to this reasoning, then Shawn must not b
e solitary. A cat could be both curious and solitary, but the authors main implication is
that the non-solitary Shawn is companionable, or sociable.

Explanation for Incorrect Answer B :

Choice (B) is incorrect. In lines 1-10 the author develops a line of reasoning that an
animals ancestry determines its behavior: since cats are descended from a completely
solitary animal, the African wildcat, they are solitary as well. The author then states
that by the same reasoning, there ought to be . . . no cats like Shawn. If the
existence of cats like Shawn poses a challenge to this reasoning, then Shawn must not b
e solitary. Independent and solitary are synonymous, and Shawn is clearly neither.

Explanation for Incorrect Answer C :

Choice (C) is incorrect. In lines 1-10 the author develops a line of reasoning that an
animals ancestry determines its behavior: since cats are descended from a completely
solitary animal, the African wildcat, they are solitary as well. The author then states
that by the same reasoning, there ought to be . . . no cats like Shawn. If the
existence of cats like Shawn poses a challenge to this reasoning, then Shawn must not b
e solitary. A cat could be both aggressive and solitary, but the authors main implication
is that the non-solitary Shawn is companionable, or sociable.

Explanation for Incorrect Answer E :

Choice (E) is incorrect. In lines 1-10 the author develops a line of reasoning that an
animals ancestry determines its behavior: since cats are descended from a completely

From SAT Online Course CUUS


2007 May Sunday SAT Answers and Explanations

solitary animal, the African wildcat, they are solitary as well. The author then states
that by the same reasoning, there ought to be . . . no cats like Shawn. If the
existence of cats like Shawn poses a challenge to this reasoning, then Shawn must not b
e solitary. A cat could be both mischievous and solitary, but the authors main
implication is that the non-solitary Shawn is companionable, or sociable.

The Equal Employ ment Opportunity Commission


has been investigating the treatment of women in a ma jor
financial services firm since a former executive alleged
Line that she had been underpaid, excluded from outings with
5 clients, and denied promotion because of her gender. If
the commission sues the firm, it would be a rare case of
the federal government s taking up the cause of a highly
compensated prof essional in a workplace dispute.

11 The passage implies that the federal government traditionally intervenes in


workplace disputes that

(A) cannot be resolved in the judicial system


(B) have repercussions for both men and women
(C) involve employees at the lower end of the pay scale
(D) are beyond the authority of state governing bodies
(E) are not likely to result in further lawsuits

ANSWERS AND EXPLA NATIONS

Explanation for Correct Answer C :

Choice (C) is correct. According to the passage, suing a major financial services firm
would be a rare case of the federal governments taking up the cause of a highly
compensated professional. If it is rare, or unusual, for the federal government to
support a highly compensated professionalsomeone who gets paid a high wageit is
logical to conclude that the federal government usually intervenes in cases that involve
employees at the lower end of the pay scaleemployees who are not highly
compensated.

Explanation for Incorrect Answer A :

Choice (A) is incorrect. The passage does not suggest that the federal government
usually gets involved in disputes that cannot be resolved in the judicial system. On the
contrary, the passage indicates that the federal government is involved in a dispute
that might be resolved in the judicial system: the Equal Employment Opportunity
Commission is investigating the treatment of women in a major financial services firm

From SAT Online Course CUUS


2007 May Sunday SAT Answers and Explanations

and might sue the firmin other words, the commission may bring their case against
the firm to court.

Explanation for Incorrect Answer B :

Choice (B) is incorrect. The passage does not suggest that the federal government
usually intervenes in workplace disputes that have repercussions for both men and
women. The passage discusses the investigation of the treatment of women in a major
financial services firm, stating that the federal government might sue the firmin other
words, the commission may bring their case against the firm to court. There is no
discussion of any possible repercussions for either the men or women who are involved.

Explanation for Incorrect Answer D :

Choice (D) is incorrect. The passage does not suggest that the federal government
usually intervenes in workplace disputes that are beyond the authority of state
governing bodies. The passage indicates that the federal government might sue a
major financial services firmthat is, the Equal Employment Opportunity Commission
may bring their case against the firm to court; there is no indication that state
governing bodies do not have the authority to sue the firm. State governing bodies are
not addressed in the passage.

Explanation for Incorrect Answer E :

Choice (E) is incorrect. The passage does not suggest that the federal government
usually intervenes in workplace disputes that are not likely to result in further lawsuits.
The passage discusses the investigation of the treatment of women in a major financial
services firm, stating that the federal government might sue the firmthat is, the Equal
Employment Opportunity Commission may bring their case against the firm to court. It
is possible that there could be further lawsuits against the firm, but the passage does
not address this possibility or suggest that the government would not get involved if
further lawsuits were likely.

12 It can be inferred that the commission views the alleged treatment of the former
executive as

(A) possibly representative of a general pattern


(B) clearly showing the need for legal reform
(C) indicative of declining business ethics
(D) posing a sensitive political dilemma
(E) likely to prove difficult to verify

ANSWERS AND EXPLA NATIONS

From SAT Online Course CUUS


2007 May Sunday SAT Answers and Explanations

Explanation for Correct Answer A :

Choice (A) is correct. The passage states that the Equal Employment Opportunity
Commission has been investigating the treatment of women in a major financial
services firm based on the allegations of a former employee. The fact that the
commission is investigating the treatment of women in the firmnot just the treatment
of the former executivesuggests that the commission believes there may be other
women who have been treated unfairly. One can infer from the passage that the
commission thinks the alleged treatment of the former executive might be
representative of a general pattern.

Explanation for Incorrect Answer B :

Choice (B) is incorrect. The passage states that the Equal Employment Opportunity
Commission might sue a major financial services firm because women there may have
been treated unfairly. By conducting an investigation based on the allegations made by
a former executive, the commission is trying to find out whether or not any of the
firms employees treated women unfairly. Because there have been such allegations, it
is possible that the commission believes that certain laws may need to be stronger or
more strict. However, this idea is not addressed in the passage; there is no indication
that the commission views the alleged treatment of the former executive or other
women in the firm as showing the need for legal reform, or changes to the laws. The
commission likely believes it is the firm, not the laws, that might be in error.

Explanation for Incorrect Answer C :

Choice (C) is incorrect. The passage states that the Equal Employment Opportunity
Commission has been investigating the treatment of women in a major financial
services firm based on the allegations of a former employee. This suggests that the
commission believes other women in the firm may have been treated unfairly; however,
there is no indication that the commission views the alleged treatment of the former
executive or other women in the firm as indicative of a general decline in business
ethics. The passage mentions the commissions investigation of only one firm, and there
is no mention of business ethics or principles.

Explanation for Incorrect Answer D :

Choice (D) is incorrect. The passage states that the Equal Employment Opportunity
Commission has been investigating the treatment of women in a major financial
services firm based on the allegations of a former employee. This suggests that the
commission believes other women in the firm may have been treated unfairly. There is
no indication in the passage that the commission views the alleged treatment of the
former executive as posing a sensitive political dilemma; there is no mention of political
issues that might be relevant to the alleged treatment.

From SAT Online Course CUUS


2007 May Sunday SAT Answers and Explanations

Explanation for Incorrect Answer E :

Choice (E) is incorrect. The passage states that the Equal Employment Opportunity
Commission has been investigating the treatment of women in a major financial
services firm based on the allegations of a former employee. The fact that the
commission is investigating the treatment of women in the firmnot just the treatment
of the former executivesuggests that the commission believes there may be other
women who have been mistreated. It could be difficult for the commission to prove the
former executives allegations, but the passage does not indicate that the commission
believes it will be difficult to verify the allegations.

This passage is adapted from a 1981 book on the history of the blues.

I have some difficulty in describing why I traveled to


West Africa and what I was doing there, since the jour-
ney that became so complicated and took me to so many
Line unexpected places seemed in the beginning to be so
5 simple and so clearly defined. I went to Africa to find the
roots of the blues. It had always been obvious that the
blues sprang from a complex background, with much of
it developing from the music of the long period of African
slavery in the United States and with some of its harmonic
10 forms and instrumental styles derived out of a broad Euro-
pean context. It had always been just as obvious that there
were certain elements in the bluesin the singing style
and in the rhythmic structuresthat were not traceable
to anything in the countryside of the American South.
15 These things, it seemed to me, might have come from
a distantly remembered African background, even if
there had been such a lengthy period between the break
in contact with Africa and the emergence of the blues in
the 1890s.
20 In the beginning I planned simply to record the tribal
singers of West Africa known as griots, since it was these
musicians who seemed to come closest to what we know as
a blues singer. They are from tribes that had many people
taken to the southern states as slaves, and they usually
25 sing alone, accompanying themselves for the most part
on plucked string instruments. Since most African music
is perf ormed by village groups, and is often dominated by
drumming, this practice in itself is enough to set the griots
apart. At the same time I hoped to collect from the singers
30 narrative accounts of the first encounters between the

From SAT Online Course CUUS


2007 May Sunday SAT Answers and Explanations

Africans and the Europeans, told from the African


viewpoint. I felt that this could give me a clearer picture of
one of the factors that had shaped the early Black
experience in the United States.
35 Bef ore leaving for Africa Id spent months taking notes
on the tribal groups and working with as much material
on the griots as I could find. As I traveled I had a definite
idea of where I wanted to go, but at the same time I had
not planned the trip in any way. Ive always felt that to
40 plan a trip too carefully is to make sure you wont find
out anything you dont already know.
I didn t know, however, how much the simple trip I
had begun would change direction once Id come to Africa,
almost as if it took on a lif e and a will of its own. I began
45 to feel like someone who had bought a set of boxes that
fit inside each other in a wooden nest. When I opened
one there was another inside it, and inside that one was
still another. I found so many boxes inside each other that
the simple project I had begun with became a series of
50 new perceptions, each of which was contained within the
perceptionthe boxthat Id just opened. Sometimes,
as I sat on sagging beds engulf ed in mosquito nets, the
space around me seemed to be filled with the myriad boxes
of different sizes that my notebooks and tapes had come to
55 symbolize.
When I opened the box that was the music Id come
to record, I found that the box inside was slavery itself.
There was no way that I could work with the music without
taking into consideration how it had come to the United
60 States. I also realized that this was one of the reasons I had
come to Africa. I was trying to find traces of an experience,
and not only that, I was looking for traces of an experience
that had occurred hundreds of years before. Would what I
found have any reality for me so many years afterward?
65 I understand now that this complex set of questions had
already been there in my mind when I put the microphones
and the tape recorder into my shoulder bag. I had always
tried to have some conception of the slavery that had
brought people from West Africa to the United States, even
70 if I hadnt seen, symbolically, that when I opened the box
decorated with pictures of musicians and instruments inside
it would be the next box, illustrated with old engravings of
slave ships. I had come to Africa to find a kind of song, to
find a kind of music and the people who perf ormed it. But

From SAT Online Course CUUS


2007 May Sunday SAT Answers and Explanations

75 nothing can be taken from a culture without considering its


context.

13 The authors difficulty (line 1) was caused primarily by the

(A) long distances that had to be traveled


(B) unanticipated changes in the project
(C) refusal to question some widespread assumptions
(D) cultural limitations that hindered communication
(E) challenge of mastering a new musical form

ANSWERS AND EXPLA NATIONS

Explanation for Correct Answer B :

Choice (B) is correct. The author s project, which consisted of finding the roots of the
blues in West Africa, resulted in the author visiting many unexpected places and
realizing that the development of the blues was more complex than he or she originally
thought. These unforeseen changes made the author s trip more complicated and
contributed to the difficulty in describing the original reason he or she made the trip to
West Africa. The second half of the passage describes in more detail how "the simple
trip [the author] had begun would change direction once [he or she had] come to
Africa."

Explanation for Incorrect Answer A :

Choice (A) is incorrect. The authors difficulty in describing the nature of the trip to
West Africa was not related to long distances. The difficulty was primarily due to
unexpected changes in the author's project, including his or her realiza tion of the
complex influences that determined the development of the blues style. The second
half of the passage describes in more detail how "the simple trip [the author] had
begun would change direction once [he or she had] come to Africa."

Explanation for Incorrect Answer C :

Choice (C) is incorrect. The passage indicates that the author was very open to
questioning assumptions about how the blues originated. The authors difficulty was
primarily caused by unexpected changes in the project once he or she began the field
work in West Africa. The second half of the passage describes in more detail how "the
simple trip [the author] had begun would change direction once [he or she had] come
to Africa."

Explanation for Incorrect Answer D :

Choice (D) is incorrect. The passage does not indicate that the authors difficulty was ca

From SAT Online Course CUUS


2007 May Sunday SAT Answers and Explanations

used by cultural limitations that hindered his or her ability to communicate. In fact,
once in Africa, the author was better able to understand the complexity of the project
and the impact of West African cultural contexts on the development of the blues. The
authors difficulty was primarily caused by the unexpected changes in the project once
he or she began field work in West Africa. The second half of the passage describes in
more detail how "the simple trip [the author] had begun would change direction once
[he or she had] come to Africa."

Explanation for Incorrect Answer E :

Choice (E) is incorrect. The authors goal was to understand the roots of the blues from
a historical perspective, not to master a new musical form; the author s difficulty was
due to the changes in his or her perception about the formation of the blues and the
forces that influenced its beginnings. The second half of the passage describes in more
detail how "the simple trip [the author] had begun would change direction once [he or
she had] come to Africa."

14 In line 10, broad most nearly means

(A) spacious
(B) progressive
(C) coarse
(D) obvious
(E) general

ANSWERS AND EXPLA NATIONS

Explanation for Correct Answer E :

Choice (E) is correct. When the author refers to a broad European context as an
influence on the blues' "harmonic forms and instrumental styles," he or she alludes to
the European musical tradition in general. In other words, the author does not refer to
a particular European technique, but to a common or general European background.

Explanation for Incorrect Answer A :

Choice (A) is incorrect. The term spacious means roomy and airy. The author does
not refer to size when he or she indicates that the blues derive in part from a broad
European context. Rather, the author refers to a wide-ranging and common European
musical tradition.

Explanation for Incorrect Answer B :

Choice (B) is incorrect. The term progressive means open-minded or forward-thinking .

From SAT Online Course CUUS


2007 May Sunday SAT Answers and Explanations

When the author states that the blues derived from a broad European context, he or
she does not refer to a progressive or forward-thinking European mindset. Rather, the
author refers to a wide-ranging and common European musical tradition.

Explanation for Incorrect Answer C :

Choice (C) is incorrect. The term coarse means crude or rough. The author does not
suggest that European "harmonic forms and instrumental styles"out of which the
blues partially developedare coarse or rough. The author uses the term broad to
refer to a wide-ranging and common European musical tradition.

Explanation for Incorrect Answer D :

Choice (D) is incorrect. The term obvious means apparent or evident. The author
does not imply that the European context that influenced the development of the
blues was obvious or apparent. Rather, the author uses the term broad to indicate
that a wide-ranging and common European musical tradition influenced the blues style.

15 The author mentions the singing style and rhythmic structures (lines 12-13)
primarily in order to

(A) discuss why the blues have remained popular through the years
(B) identify aspects of the blues that present a historical enigma
(C) argue that the American South had a profound influence on the musical
structure of the blues
(D) praise the musical complexity of a little-known art form
(E) cite some blues innovations that have influenced other musical genres

ANSWERS AND EXPLA NATIONS

Explanation for Correct Answer B :

Choice (B) is correct. When the author discusses the "complex background" of blues
music he or she first mentions "the music of the long period of African slavery" and the
"broad European context" as clear influences on the blues. Next, he or she indicates
that there are aspects of the blues, particularly in its singing style and rhythmic
structures, that people have not been able to trace directly to any other musical
tradition. These elements present a historical enigma, or mystery, for the authorone
for which he or she hopes to find an answer in Africa.

Explanation for Incorrect Answer A :

Choice (A) is incorrect. The author does not speculate about why the blues have
remained popular. When the author discusses the "complex background" of blues music

From SAT Online Course CUUS


2007 May Sunday SAT Answers and Explanations

he or she first mentions "the music of the long period of African slavery" and the "broad
European context" as clear influences on the blues. Next, he or she indicates that there
are aspects of the blues, particularly in its singing style and rhythmic structures, that
people have not been able to trace directly to any other musical tradition. These
elements present a historical enigma, or mystery, for the authorone for which he or
she hopes to find an answer in Africa.

Explanation for Incorrect Answer C :

Choice (C) is incorrect. In lines 11-14 the author discusses the "complex background"
of blues music. The author indicates that the United States did have some influence on
the blues (developing from the music of the long period of African slavery in the
United States), but he or she refers to the singing style and . . . the rhythmic
structures as examples of elements of the blues that cannot be traced back to the
American South (not traceable to anything in the countryside of the American South).
The author suggests that these elements present a historical enigma, or mystery, for
the author one for which he or she hopes to find an answer in Africa.

Explanation for Incorrect Answer D :

Choice (D) is incorrect. Nowhere in the passage does the author suggest that the blues
is a little-known art form. When the author discusses the complex background of
blues music he or she first mentions the music of the long period of African slavery
and the broad European context as clear influences on the blues. Next, he or she
indicates that there are aspects of the blues, particularly in its singing style and
rhythmic structures, that people have not been able to trace directly to any other
musical tradition. These elements present a historical enigma, or mystery, for the
authorone for which he or she hopes to find an answer in Africa.

Explanation for Incorrect Answer E :

Choice (E) is incorrect. In lines 11-14 the author discusses the "complex background"
of blues music. He or she indicates that there are aspects of the blues, particularly in
its singing style and rhythmic structures, that were not traceable to anything in the
countryside of the American South. The author does not mention the blues' singing
style and rhythmic structures in order to discuss how the blues influenced other musical
genres, or types of music. Rather, the author indicates that these elements present a
historical enigma, or mystery, for which he or she hopes to find an answer in Africa.

16 The description of the musicians in lines 20-29 (In the . . . apart) primarily
serves to

(A) introduce a new school of African music by comparing it to the music of the griot
s

From SAT Online Course CUUS


2007 May Sunday SAT Answers and Explanations

(B) describe an alternative interpretation of blues music


(C) indicate important differences between the author and other music historians
(D) justify an intended direction in the authors research
(E) illustrate the social status of musicians in West Africa

ANSWERS AND EXPLA NATIONS

Explanation for Correct Answer D :

Choice (D) is correct. In lines 20-29 the author gives a number of reasons to justify the
attention he or she paid to the tribal singers in West Africa, the griots. The author
explains why these musicians come closest to what we know as a blues singer: they
are from tribes from which many slaves were taken, and they sing alone and play
string instruments (blues singers tend to sing alone and play guitar). The autho r seems
to be justifying the choice to study the griots by showing evidence of a connection
between griots and blues singers.

Explanation for Incorrect Answer A :

Choice (A) is incorrect. The author does not present any new school of African music or
compare it to the music of the griots. He or she describes the griots of West Africa and
their links to the blues: they are from tribes from which many slaves were taken, and
they sing alone and play string instruments (blues singers tend to sing alone and play
guitar). The author seems to be justifying the choice to study the griots by showing
evidence of a connection between griots and blues singers.

Explanation for Incorrect Answer B :

Choice (B) is incorrect. The author does not present any interpretation of blues music,
alternative or otherwise. He or she describes the griots of West Africa and their links to
the blues: they are from tribes from which many slaves were taken, and they sing
alone and play string instruments (blues singers tend to sing alone and play guitar).
The author seems to be justifying the choice to study the griots by showing evidence of
a connection between griots and blues singers.

Explanation for Incorrect Answer C :

Choice (C) is incorrect. In lines 20-29 the author does not compare himself or herself
with any other music historians or indicate that he or she differs from other historians.
Rather, in these lines he or she focuses on the West African tribal singers known as the
griots; the griots are from tribes from which many slaves were taken, and they sing
alone and play string instruments (blues singers tend to sing alone and play guitar).
The author seems to be justifying the choice to study the griots by showing evidence of
a connection between griots and blues singers.

From SAT Online Course CUUS


2007 May Sunday SAT Answers and Explanations

Explanation for Incorrect Answer E :

Choice (E) is incorrect. Nothing in lines 20-29 alludes to the social status of musicians
in West Africa. These lines describe the authors intention to research the griots; the gri
ots are from tribes from which many slaves were taken, and they sing alone and play
string instruments (blues singers tend to sing alone and play guitar). The author seems
to be justifying the choice to study the griots by showing evidence of a connection
between griots and blues singers.

17 The author views the narrative accounts (line 30) primarily as

(A) offering a useful perspective on a complex historical situation


(B) lending authenticity to an unusual form of music
(C) contributing to a communitys sense of patriotism
(D) exhibiting the versatile nature of an art form
(E) increasing the appeal of an already popular musical genre

ANSWERS AND EXPLA NATIONS

Explanation for Correct Answer A :

Choice (A) is correct. The author hopes to collect the singers narrative accounts in
order to gain information about their ancestors encounters with the Europeans. The
author believes that these narratives might offer a "clearer picture" of the events that
shaped the early Black experience in the United States. He or she notes throughout
the passage that in order to study the blues, it is necessary to confront a complex
historical situation; the author hopes that the narrative accounts will help him or her
understand part of that history.

Explanation for Incorrect Answer B :

Choice (B) is incorrect. The author does not want to collect the singers narrative
accounts in order to authenticate an unusual form of music. Rather, the author views
the narrative accounts as a way to access a unique perspective on important events
that shaped the early Black experience in the United States. He or she notes
throughout the passage that in order to study the blues, it is necessary to confront a
complex historical situation; the author hopes that the narrative accounts will help him
or her understand part of that history.

Explanation for Incorrect Answer C :

Choice (C) is incorrect. Although the narrative accounts of the griots might contain
elements of patriotism, the author is primarily concerned with examining the
experiences the tribes had in their first encounters with the Europeans. The author

From SAT Online Course CUUS


2007 May Sunday SAT Answers and Explanations

believes that this unique perspective would provide a clearer picture of the factors that
had shaped the early Black experience in the United States. He or she notes
throughout the passage that in order to study the blues, it is necessary to confront a
complex historical situation; the author hopes that the narrative accounts will help him
or her understand part of that history.

Explanation for Incorrect Answer D :

Choice (D) is incorrect. The author does not refer to the narrative accounts as an art
form or mention that he or she plans to consider them as such. The authors primary
purpose for examining these stories is to learn about the tribes early encounters with
the Europeans. The author believes that this perspective will help him or her gain a
better understanding of the events that shaped the early Black experience in the
United States. He or she notes throughout the passage that in order to study the blues,
it is necessary to confront a complex historical situation; the author hopes that the
narrative accounts will help him or her understand part of that history.

Explanation for Incorrect Answer E :

Choice (E) is incorrect. The primary purpose for the author s interest in the singers
narrative accounts is to better understand the factors that helped [shape] the early
Black experience in the United States. The author does not state that he or she
intends to utilize these narratives as a way to increase the popular appeal of a musical
genre. He or she notes throughout the passage that in order to study the blues, it is
necessary to confront a complex historical situation; the author hopes that the
narrative accounts will help him or her understand part of that history.

18 Lines 35-41 (Before . . . know) suggest that the author valued both

(A) frivolity and impulsiveness


(B) preparation and flexibility
(C) scholarship and artistry
(D) thoroughness and subtlety
(E) creativity and generosity

ANSWERS AND EXPLA NATIONS

Explanation for Correct Answer B :

Choice (B) is correct. Lines 35-41 indicate that the author valued both preparation and
flexibility. He or she prepared very carefully, spending "months" researching the tribal
groups, but also showed flexibility by not planning the details of the trip in advance (I
had not planned the trip in any way). The author explains that to plan a trip too
carefully is to make sure you wont find out anything you dont already know.

From SAT Online Course CUUS


2007 May Sunday SAT Answers and Explanations

Explanation for Incorrect Answer A :

Choice (A) is incorrect. Lines 35-41 do suggest that the author might value
impulsiveness, or the tendency to act on sudden inspiration or impulsethe fact that
the author had not planned the trip in any way implies that the author made
decisions about the course of the trip as he or she went along. But lines 35-41 do not
suggest that the author valued frivolity, or unbecoming levity and a lack of seriousness;
the author seems to have been serious when preparing for his or her trip, spending
months researching the tribal groups. Rather than suggesting that the author valued
frivolity and impulsiveness, lines 35-41 suggest that he or she valued both preparation
and flexibility.

Explanation for Incorrect Answer C :

Choice (C) is incorrect. Lines 35-41 do suggest that the author values scholarship, or
the activity of learning; the author spent months researching the tribal groups.
However, although the author may deeply appreciate art, nothing in lines 35-41
suggests that the author values artistry. Rather, these lines suggest that the author
valued both preparation and flexibility; the author researched the tribal groups before
his or her trip but did not plan the details of the trip in advance (I had not planned the
trip in any way).

Explanation for Incorrect Answer D :

Choice (D) is incorrect. Lines 35-41 do indicate that the author valued thoroughness;
the author claims to have worked with as much material on the griots as [he or she]
could find when preparing for the trip. However, nothing in these lines indicates that
the author valued subtlety, or the quality of being difficult to understand or perceive.
Lines 35-41 primarily suggest that the author valued both preparation and flexibility;
the author researched the tribal groups before his or her trip but did not plan the
details of the trip in advance (I had not planned the trip in any way).

Explanation for Incorrect Answer E :

Choice (E) is incorrect. The author may have valued the qualities of being creative and
generous, but nothing in lines 35-41 indicates that the author valued creativity and
generosity. These lines focus on the author s preparation for his or her trip to Africa and
suggest that he or she valued both preparation and flexibility. The author prepared very
carefully, spending months researching the tribal groups, but also showed flexibility
by not planning the details of the trip in advance (I had not planned the trip in any
way).

19 Which statement best describes the function of the sentence in lines 42-44 (I
didnt . . . own)?

From SAT Online Course CUUS


2007 May Sunday SAT Answers and Explanations

(A) It indicates a significant turning point in the author s research.


(B) It suggests that the authors initial hypothesis lacked validity.
(C) It reveals the author s ability to adapt to a new environment.
(D) It highlights the importance of the author s thorough preparation.
(E) It expresses regret about the outcome of the authors trip .

ANSWERS AND EXPLA NATIONS

Explanation for Correct Answer A :

Choice (A) is correct. In lines 42-44, the author explains that he or she "didn't know . . .
how much the simple trip [he or she] had begun would change direction once he or
she arrived in Africa. This sentence indicates that the author's research planto
interview the West African singers known as the griotsbecame much more complex
once he or she arrived in Africa and began working. The author is vague about exactly
how his or her research changed, but it is clear that the simple trip changed direction
and "took on a life and will of its own."

Explanation for Incorrect Answer B :

Choice (B) is incorrect. The statement in lines 42-44 does not invalidate the authors
initial hypothesis that American blues music was partly influenced by African tribal
music. Rather, as the author explains, a simple research plan changed and became
more complex. In lines 42-44, the author notes, "I didn't know . . . how much the
simple trip I had begun would change direction once I'd come to Africa." This sentence
indicates that the author's plan to interview the West African singers known as
the griots began to change once he or she arrived in Africa and began working. The
discussion of "a set of boxes" that follows shows just how complex the research project
became.

Explanation for Incorrect Answer C :

Choice (C) is incorrect. Although the author appears capable of adapting to new
situations, lines 42-44 do not primarily illustrate his or her ability to adapt. In these
lines, the author explains that "[he or she] didn't know . . . how much the simple trip .
. . would change direction once [he or she had] come to Africa." This sentence
indicates that the author's "simple" research planto interview the West African singers
known as the griotsbecame much more complex once he or she arrived in Africa and
began working.

Explanation for Incorrect Answer D :

Choice (D) is incorrect. The sentence in lines 42-44 does not highlight the importance
of thorough preparation. Rather, the sentence highlights the necessity of remaining ope

From SAT Online Course CUUS


2007 May Sunday SAT Answers and Explanations

n to new discoveries and unexpected turns of events. In these lines, the author
explains that "[he or she] didn't know . . . how much the simple trip . . . would change
direction once [he or she had] come to Africa." This sentence indicates that the
autho r's "simple" research planto interview the West African singers known as the gri
otsbecame much more complex once he or she arrived in Africa and began working.

Explanation for Incorrect Answer E :

Choice (E) is incorrect. There is no sense of regret in the author s statement in lines
42-44. The author merely indicates that "[he or she] didn't know . . . how much the
simple trip . . . would change direction once [he or she had] come to Africa." This
sentence explains that the author's "simple" research planto interview the West
African singers known as the griots became much more complex once he or she
arrived in Africa and began working. The author seems to find this situation confusing,
as the discussion of the "set of boxes" shows, but he or she does not express regret or
disappointment.

20 The primary purpose of the fourth paragraph (lines 42-55) is to

(A) demonstrate the authors belief that earlier studies of West African music were
inaccurate
(B) dramatize the authors excitement about some findings
(C) justify the authors method of collecting music samples
(D) convey the increasing challenges of the authors project
(E) describe the authors daily life while conducting research

ANSWERS AND EXPLA NATIONS

Explanation for Correct Answer D :

Choice (D) is correct. The primary purpose of the fourth paragraph is to convey the
increasing challenges of the authors project. The author describes how the simple trip
. . . change[d] direction and seemed to take on a life and a will of its own. By saying
that he or she felt like someone who had bought a set of boxes that fit inside each
other, the author explains how the project became more complex and more
challenging as he or she made new discoveries: the simple project I had begun with
became a series of new perceptions, each of which was contained within the
perceptionthe boxthat Id just opened. Every time the author mastered one
perception, or one concept, there was something elsesomething newto investigate.

Explanation for Incorrect Answer A :

Choice (A) is incorrect. The primary purpose of the fourth paragraph is not to
demonstrate the author s belief that earlier studies of West African music were inaccura

From SAT Online Course CUUS


2007 May Sunday SAT Answers and Explanations

te. Instead, the fourth paragraph illustrates the increasing challenges of the authors
project: the simple project I had begun with became a series of new perceptions, each
of which was contained within the perceptionthe boxthat Id just opened. Ever y
time the author mastered one perception, or one concept, there was something
elsesomething newto investigate.

Explanation for Incorrect Answer B :

Choice (B) is incorrect. The primary purpose of the fourth paragraph is not to dramatize
the author s excitement about some findings. While it is likely that the author felt some
excitement, there is no indication of this; rather, the author seems a bit confused and
overwhelmed by the challenges of his or her project. In the fourth paragraph, the
author describes how the project became more complex and more challenging as it
evolved: the simple project I had begun with became a series of new perceptions,
each of which was contained within the perceptionthe boxthat Id just opened.

Explanation for Incorrect Answer C :

Choice (C) is incorrect. The primary purpose of the fourth paragraph is not to justify
the author s method of collecting music samples. Earlier in the passage, the author
does mention his or her intention to simply record the tribal singers of West Africa.
However, the fourth paragraph is not limited to a discussion or justification of this
method of collecting music samples. This paragraph focuses on the broad range of the
authors project, not just one aspect of it. The author describes how the projec t became
more complex and more challenging as it evolved: the simple project I had begun with
became a series of new perceptions, each of which was contained within the
perceptionthe boxthat Id just opened.

Explanation for Incorrect Answer E :

Choice (E) is incorrect. The primary purpose of the fourth paragraph is not to describe
the author s daily life while conducting research. The author does mention times when
he or she would sit on sagging beds engulfed in mosquito nets, but this is simply a
description of the physical environment, not of his or her daily life. Rather, the fourth
paragraph essentially describes the growing complexity of the authors project: the
simple project I had begun with became a series of new perceptions, each of which was
contained within the perceptionthe boxthat Id just opened. Every time the author
mastered one perception, or one concept, there was something elsesomething
newto investigate.

21 In context, the reference to notebooks and tapes (line 54) primarily serves to

(A) illustrate the technology required by the authors research


(B) underscore the authors growing awareness of the scope of the undertaking

From SAT Online Course CUUS


2007 May Sunday SAT Answers and Explanations

(C) suggest that few people appreciate the difficulty of writing historical narratives
(D) describe the authors success in collecting data that supports oral history
(E) indicate the importance of a methodical approach to every project

ANSWERS AND EXPLA NATIONS

Explanation for Correct Answer B :

Choice (B) is correct. Earlier in the passage, the author discusses his or her plan to
record the tribal singers of West Africa, and mentions spending months taking notes
on the tribal groups before leaving for Africa. As the author gathered data through
notes and musical recordings he or she made new discoveries which led to new
investigations: I began to feel like someone who had bought a set of boxes that fit
inside each other . . . . When I opened one there was another inside it, and inside that
one was still another. In line 54, the author notes that his or her notebooks and tapes
had come to symbolize these boxes; the author's many notebooks and tapes reflect
his or her growing awareness of the scope of the undertaking.

Explanation for Incorrect Answer A :

Choice (A) is incorrect. The reference to notebooks and tapes is not intended to
illustrate the technology required by the authors research; although the author clearly
used notebooks and tapes, there is very little discussion of his or her research methods
or the tools that might be necessary to conduct the research. Rather, the notebooks
and tapes symbolize the myriad boxes of different sizes in other words, they
represent the new perceptions gained by the author, as well as his or her growing
awareness of the increasing complexity of the project. As the author gathered
datathrough notes and musical recordingshe or she made new discoveries which led
to new investigations: I began to feel like someone who had bought a set of boxes
that fit inside each other . . . . When I opened one there was another inside it, and
inside that one was still another.

Explanation for Incorrect Answer C :

Choice (C) is incorrect. There is no indication that the author references his or her
notebooks and tapes to suggest that few people appreciate the difficulty of writing
historical narratives. Rather, the notebooks and tapes symbolize the myriad boxes of
different sizes in other words, they represent the new perceptions gained by the
author, as well as his or her growing awareness of the increasing complexity of the
project. As the author gathered datathrough notes and musical recordingshe or she
made new discoveries which led to new investigations: I began to feel like someone
who had bought a set of boxes that fit inside each other . . . . When I opened one there
was another inside it, and inside that one was still another.

Explanation for Incorrect Answer D :

From SAT Online Course CUUS


2007 May Sunday SAT Answers and Explanations

Choice (D) is incorrect. The author does not necessarily indicate success in collecting
data when he or she references the notebooks and tapes. These notebooks and tapes
symbolize the myriad boxes of different sizes in other words, they represent the new
perceptions gained by the author, as well as his or her growing awareness of the
increasing complexity of the project. As the author gathered datathrough notes and
musical recordingshe or she made new discoveries which led to new investigations: I
began to feel like someone who had bought a set of boxes that fit inside each other . . .
. When I opened one there was another inside it, and inside that one was still another.

Explanation for Incorrect Answer E :

Choice (E) is incorrect. The author never comments on the importance of a methodical
approach to every project. While his or her use of notebooks and tapes might have
been somewhat methodical, these research tools are used to symbolize the myriad
boxes of different sizes in other words, they represent the new perceptions gained by
the author, as well as his or her growing awareness of the increasing complexity of the
project. As the author gathered datathrough notes and musical recordingshe or she
made new discoveries which led to new investigations: I began to feel like someone
who had bought a set of boxes that fit inside each other . . . . When I opened one there
was another inside it, and inside that one was still another.

22 In context, the phrase not only that (line 62) emphasizes the idea that

(A) an event will prove to have surprising repercussions


(B) an objective will be particularly difficult to accomplish
(C) a problem can often be solved by considering its historical context
(D) a research finding will strain the readers credibility
(E) an approach has yielded a number of promising leads

ANSWERS AND EXPLA NATIONS

Explanation for Correct Answer B :

Choice (B) is correct. In lines 60-61, the author states that one of the reasons why [he
or she] had come to Africa was to find traces of an experience. The fact that the
author uses the term traces indicates that there is some difficulty involved in the
taska trace is a small and often barely detectable amount or indication. The phrase
not only that in line 62 then emphasizes the idea that an objective will be particularly
difficult to accomplish: in addition to looking for something that is likely hard to find,
the author is looking for traces of an experience that occurred hundreds of years
before. Since the author is researching something that happened in the distant past,
the task seems even more difficult to accomplish.

Explanation for Incorrect Answer A :

From SAT Online Course CUUS


2007 May Sunday SAT Answers and Explanations

Choice (A) is incorrect. In lines 60-61, the author states that one of the reasons why
[he or she] had come to Africa was to find traces of an experience. The fact that the
author uses the term traces indicates that there is some difficulty involved in the
taska trace is a small and often barely detectable amount or indication. The phrase
not only that in line 62 then emphasizes the idea that an objective will be particularly
difficult to accomplish: in addition to looking for something that is likely hard to find,
the author is looking for traces of an experience that occurred hundreds of years
before. Since the author is researching something that happened in the distant past,
the task seems even more difficult to accomplish. There is no indication that the author
believes his or her findings will have any surprising repercussions.

Explanation for Incorrect Answer C :

Choice (C) is incorrect. While the author may believe that a problem can often be
solved by considering its historical context, the phrase not only that in line 62 does
not emphasize this idea. In lines 60-61, the author states that one of the reasons why
[he or she] had come to Africa was to find traces of an experience. The fact that the
author uses the term traces indicates that there is some difficulty involved in the
taska trace is a small and often barely detectable amount or indication. The phrase
not only that in line 62 then emphasizes the idea that an objective will be particularly
difficult to accomplish: in addition to looking for something that is likely hard to find,
the author is looking for traces of an experience that occurred hundreds of years
before. Since the author is researching something that happened in the distant past,
the task seems even more difficult to accomplish.

Explanation for Incorrect Answer D :

Choice (D) is incorrect. In lines 60-61, the author states that one of the reasons why
[he or she] had come to Africa was to find traces of an experience. The fact that the
author uses the term traces indicates that there is some difficulty involved in the
taska trace is a small and often barely detectable amount or indication. The phrase
not only that in line 62 then emphasizes the idea that an objective will be particularly
difficult to accomplish: in addition to looking for something that is likely hard to find,
the author is looking for traces of an experience that occurred hundreds of years
before. Since the author is researching something that happened in the distant past,
the task seems even more difficult to accomplish. There is no discussion of the readers
response to the authors findings, and the phrase not only that certainly does not
suggest that the readers credibility will be strained as a result of the findings.

Explanation for Incorrect Answer E :

Choice (E) is incorrect. In lines 60-61, the author states that one of the reasons why
[he or she] had come to Africa was to find traces of an experience. The fact that the
author uses the term traces indicates that there is some difficulty involved in the
taska trace is a small and often barely detectable amount or indication. The phrase n

From SAT Online Course CUUS


2007 May Sunday SAT Answers and Explanations

ot only that in line 62 then emphasizes the idea that an objective will be particularly
difficult to accomplish: in addition to looking for something that is likely hard to find,
the author is looking for traces of an experience that occurred hundreds of years
before. Since the author is researching something that happened in the distant past,
the task seems even more difficult to accomplish. The author does not discuss any
promising leads that have come out of the research.

23 The primary obstacle the author faces in seeking the traces of an experience
(line 62) is the

(A) difficulty of maintaining emotional detachment from the topic


(B) inability to find any new evidence in Africa
(C) distortion of actual events when represented in song and story
(D) reluctance of some people to be interviewed by a stranger
(E) distance between the modern observer and past events

ANSWERS AND EXPLA NATIONS

Explanation for Correct Answer E :

Choice (E) is correct. The author is attempting to reconstruct events that occurred
hundreds of years earlier. The author is hopeful that he or she can record the tribal
singers of West Africa and collect from the singers narrative accounts of the first
encounters between the Africans and the Europeans, told from an African viewpoint.
However, since these first encounters took place in the distant past, the author faces
the enormous challenge of successfully understanding those experiences. The author
doubts his or her ability to bridge the distance between the modern observer (the
author) and past events (the encounters), posing the question, Would what I found
have any reality for me so many years afterward?

Explanation for Incorrect Answer A :

Choice (A) is incorrect. There is no discussion of any possible emotional attachment to


the topic or any difficulty in maintaining an emotional detachment. The author is
attempting to reconstruct events that occurred hundreds of years earlier. The author
is hopeful that he or she can record the tribal singers of West Africa and collect from
the singers narrative accounts of the first encounters between the Africans and the
Europeans, told from an African viewpoint. However, since these first encounters
took place in the distant past, the author faces the enormous challenge of successfully
understanding those experiences.

Explanation for Incorrect Answer B :

Choice (B) is incorrect. The author does not express any concern about an inability to fi

From SAT Online Course CUUS


2007 May Sunday SAT Answers and Explanations

nd new evidence in Africa. In fact, in the author s attempt to find traces of an


experience that had occurred hundreds of years before," he or she is hopeful that he or
she can record the tribal singers of West Africa and collect from the singers narrative
accounts of the first encounters between the Africans and the Europeans, told from an
African viewpoint. However, since these first encounters took place in the distant
past, the author faces the enormous challenge of successfully understanding those
experiences. The author doubts his or her ability to bridge the distance between the
modern observer (the author) and past events (the encounters), posing the question,
Would what I found have any reality for me so many years afterward?

Explanation for Incorrect Answer C :

Choice (C) is incorrect. There is no indication that the author feels that there might be
a distortion of actual events represented in songs and stories; if the author did believe
this, he or she would not be relying on recording West African music and listening to
personal narratives as methods of gathering information. The author is attempting to
reconstruct events that occurred hundreds of years earlier. The author is hopeful that
he or she can record the tribal singers of West Africa and collect from the singers
narrative accounts of the first encounters between the Africans and the Europeans, told
from an African viewpoint. However, since these first encounters took place in the
distant past, the author faces the enormous challenge of successfully understanding
those experiences.

Explanation for Incorrect Answer D :

Choice (D) is incorrect. Nowhere in the passage does the author discuss some peoples
reluctance to be interviewed by a stranger. The passage focuses on the authors actions,
thoughts, and feelings throughout the experience of conducting research; it does not
focus on others reactions or responses to the author s methods. The author is
attempting to reconstruct events that occurred hundreds of years earlier. The author
is hopeful that he or she can record the tribal singers of West Africa and collect from
the singers narrative accounts of the first encounters between the Africans and the
Europeans, told from an African viewpoint. However, since these first encounters
took place in the distant past, the author faces the enormous challenge of successfully
understanding those experiences.

24 The author presents the final sentence (lines 74-76) as a

(A) learned lesson


(B) grim admonition
(C) surprising afterthought
(D) scholarly hypothesis
(E) sarcastic commentary

From SAT Online Course CUUS


2007 May Sunday SAT Answers and Explanations

ANSWERS AND EXPLA NATIONS

Explanation for Correct Answer A :

Choice (A) is correct. Throughout the passage, the author explains his or her
experiences in Africa, searching for the roots of the blues. The author describes his or
her intentions, hopes, and concerns; his or her methods of research; and his or her
new and unexpected perceptions. The last sentence in the passage then expresses an
important lesson the author learned through his or her investigative processthat
objects of study need to be understood in their appropriate context. The author
indicates that it is not enough to study a kind of music and the people who performed
it; one needs to examine and understand the cultural context in which the music and
the people played a part.

Explanation for Incorrect Answer B :

Choice (B) is incorrect. Throughout the passage, the author explains his or her
experiences in Africa, searching for the roots of the blues. The last sentence in the
passage then expresses an important lesson the author learned through his or her
investigative processthat objects of study need to be understood in their appropriate
context. The author indicates that it is not enough to study a kind of music and the
people who performed it; one needs to examine and understand the cultural context in
which the music and the people played a part. While the last sentence could be viewed
as an admonition, or cautionary advice or warning, that those who conduct research
should be sure to take cultural context into consideration, there is certainly nothing
grimthat is, fierce or sternabout this piece of advice. The author is simply sharing
something important that he or she learned.

Explanation for Incorrect Answer C :

Choice (C) is incorrect. Throughout the passage, the author explains his or her
experiences in Africa, searching for the roots of the blues. The last sentence in the
passage then expresses an important lesson the author learned through his or her
investigative processthat objects of study need to be understood in their appropriate
context. The author indicates that it is not enough to study a kind of music and the
people who performed it; one needs to examine and understand the cultural context in
which the music and the people played a part. Although the author may not have
originally realized the importance of considering cultural context, the last sentence does
not convey any degree of surprise. The author is simply sharing something important
that he or she learned.

Explanation for Incorrect Answer D :

Choice (D) is incorrect. Throughout the passage, the author explains his or her
experiences in Africa, searching for the roots of the blues. The author describes his or

From SAT Online Course CUUS


2007 May Sunday SAT Answers and Explanations

her intentions, hopes, and concerns; his or her methods of research; and his or her
new and unexpected perceptions. The last sentence in the passage then expresses an
important lesson the author learned through his or her investigative processthat
objects of study need to be understood in their appropriate context. The author
indicates that it is not enough to study a kind of music and the people who performed
it; one needs to examine and understand the cultural context in which the music and
the people played a part. This is a general statement, not a scholarly hypothesis. A
scholarly hypothesis is a specific assumption or explanation about an observation or
phenomenon that serves as a basis for further investigation.

Explanation for Incorrect Answer E :

Choice (E) is incorrect. Throughout the passage, the author explains his or her
experiences in Africa, searching for the roots of the blues. The last sentence in the
passage then expresses an important lesson the author learned through his or her
investigative processthat objects of study need to be understood in their appropriate
context. The author indicates that it is not enough to study a kind of music and the
people who performed it; one needs to examine and understand the cultural context in
which the music and the people played a part. The last sentence does not convey
sarcasmthe author does not use language that is intended to express contempt or
ridicule. The author seems to be genuine, sharing something important that he or she
learned from experience.

25 The primary purpose of the passage is to describe the

(A) authors theory about the origins of slave music in the American South
(B) mental processes of a researcher considering an issue in music history
(C) position of the griots in the musical culture of West Africa
(D) various research techniques utilized by different musicologists
(E) significance of blues music in the social development of West Africa

ANSWERS AND EXPLA NATIONS

Explanation for Correct Answer B :

Choice (B) is correct. The passage primarily serves to illustrate the author s mental
processes as he or she experiences various phases of research and discovery in the
attempt to "find the roots of the blues. The author describes his or her journey to
Africa as one that originally seemed so simple and so clearly defined, and later
became so complicated and took [him or her] to so many unexpected places. The
author then explains how his or her trip to Africa resulted in not only gathering
pertinent information and facing unexpected challenges, but also discovering new
perceptions and issues greater than finding the origin of a type of music. Ultimately,
the author realizes the importance of examining past events as well as the cultural cont

From SAT Online Course CUUS


2007 May Sunday SAT Answers and Explanations

ext in which those events took place, in addition to studying the music itself. Clearly,
the passage conveys a variety of mental processes and emotions that the author
experienced: hope, anticipation, confusion, appreciation, and realiza tion.

Explanation for Incorrect Answer A :

Choice (A) is incorrect. While the author does mention that the blues obviously sprang
from the music of the long period of African slavery in the United States, the primary
purpose of the passage is not to describe the author's theory about "slave music." The
author is attempting to find the roots of the blues. The author describes the journey
to Africa as one that originally seemed so simple and so clearly defined, and later
became so complicated and took [him or her] to so many unexpected places. The
author then explains how his or her trip to Africa resulted in not only gathering
pertinent information and facing unexpected challenges, but also discovering new
perceptions and issues greater than finding the origin of a type of music. Clearly, the
passage conveys a variety of mental processes and emotions that the author
experienced: hope, anticipation, confusion, appreciation, and realiza tion.

Explanation for Incorrect Answer C :

Choice (C) is incorrect. In conducting his or her research, the author recorded the griots
' music and sought material and information on the griotsbut only because of their
connection to the blues. There is no discussion of the griots' position or status in the
musical culture of West Africa. In his or her attempt to find the roots of the blues, the
author describes the journey to Africa as one that originally seemed so simple and so
clearly defined, and later became so complicated and took [him or her] to so many
unexpected places. The author then explains how his or her trip to Africa resulted in
not only gathering pertinent information and facing unexpected challenges, but also
discovering new perceptions and issues greater than finding the origin of a type of
music. Clearly, the passage conveys a variety of mental processes and emotions that
the author experienced: hope, anticipation, confusion, appreciation, and realiza tion.

Explanation for Incorrect Answer D :

Choice (D) is incorrect. The primary purpose of the passage is not to describe the
various research techniques utilized by different musicologists. The passage primarily
serves to describe one researchersthe authorsreflections on the mental processes
he or she experienced in the attempt to "find the roots of the blues. The author
describes his or her journey to Africa as one that originally seemed so simple and so
clearly defined, and later became so complicated and took [him or her] to so many
unexpected places. Clearly, the passage conveys a variety of mental processes and
emotions that the author experienced: hope, anticipation, confusion, appreciation, and
realiza tion. There is no discussion of any other musicologists or researchers.

Explanation for Incorrect Answer E :

From SAT Online Course CUUS


2007 May Sunday SAT Answers and Explanations

Choice (E) is incorrect. The primary purpose of the passage is not to explain the
significance of blues music in the social development of West Africa. On the contrary,
the passage describes the author s realiza tion that the social and cultural experiences
of West African tribes had a significant impact on the development of blues music. In
his or her attempt to find the roots of the blues, the author describes the journey to
Africa as one that originally seemed so simple and so clearly defined, and later
became so complicated and took [him or her] to so many unexpected places. The
passage conveys a variety of mental processes and emotions that the author
experienced: hope, anticipation, confusion, appreciation, and realiza tion.

Section5: Writing

View Answers and Explanations

Online - Practice Test #2

1 On Friday, the local police arrested many demonstrators, especially while picketing
the construction site.

(A) especially while picketing the construction site


(B) especially them picketing the construction site
(C) especially those who were picketing the construction site
(D) especially their picketing of the construction site
(E) and especially the picketing of the construction site

ANSWERS AND EXPLA NATIONS

Explanation for Correct Answer C :

Choice (C) is correct. It avoids the modification error of the original by providing an
appropriate subject to complete the action of picketing the construction site.

Explanation for Incorrect Answer A :

Choice (A) involves improper modification. The sentence illogically implies that the
police were picketing the construction site while arresting demonstrators.

Explanation for Incorrect Answer B :

Choice (B) results in an improper pronoun case. The pronoun them, which is in the
objective case, is inappropriate in this context. The plural demonstrative pronoun

From SAT Online Course CUUS


2007 May Sunday SAT Answers and Explanations

those should be used instead.

Explanation for Incorrect Answer D :

Choice (D) results in an illogical statement. It does not make sense to say that local
police arrested . . . their picketing of the construction site.

Explanation for Incorrect Answer E :

Choice (E) results in improper coordination. The clause especially the picketing of the
construction site is dependent and cannot be joined to the preceding independent
clause with only a comma and the conjunction and. In addition, this clause does not
logically modify demonstrators.

2 An increased concentration of carbon dioxide and certain other gases in the Earths
atmosphere, contributing to global warming.

(A) atmosphere, contributing to


(B) atmosphere, they contribute to
(C) atmosphere, which contributes to
(D) atmosphere contributes to
(E) atmosphere and contributes to

ANSWERS AND EXPLA NATIONS

Explanation for Correct Answer D :

Choice (D) is correct. It avoids the sentence-fragment error of the original by providing
a main verb (contributes) to carry out the action of the sentence.

Explanation for Incorrect Answer A :

Choice (A) involves a sentence fragment. There is no main verb to carry out the action
of the sentence, only the participle contributing.

Explanation for Incorrect Answer B :

Choice (B) results in a vague pronoun. There is nothing in the sentence to which the
plural pronoun they can logically refer.

Explanation for Incorrect Answer C :

Choice (C) results in a sentence fragment. There is no main verb to carry out the action
of the sentence, only the modifying phrase which contributes to global warming.

From SAT Online Course CUUS


2007 May Sunday SAT Answers and Explanations

Explanation for Incorrect Answer E :

Choice (E) involves improper coordination and results in faulty sentence structure.
There is no main verb to carry out the action of the clause that precedes the
conjunction and.

3 Millions of dollars are spent each year to get rid of fleas on dogs and cats causing a
variety of diseases.

(A) to get rid of fleas on dogs and cats causing a variety of diseases
(B) because they can cause a variety of diseases if dogs and cats are not rid of fleas
(C) to rid dogs and cats of fleas, which can cause a variety of diseases
(D) to be getting dogs and cats rid of fleas, which can cause a variety of diseases
(E) because a variety of diseases can be gotten if you do not rid dogs and cats of
fleas

ANSWERS AND EXPLA NATIONS

Explanation for Correct Answer C :

Choice (C) is correct. It avoids the modification error of the original by placing the noun
fleas closest to the phrase that modifies it (which can cause a variety of diseases).

Explanation for Incorrect Answer A :

Choice (A) involves a modification error. It does not make sense to say that dogs and
cats are causing a variety of diseases.

Explanation for Incorrect Answer B :

Choice (B) results in a vague pronoun. It is not clear to what the plural pronoun they
is referring.

Explanation for Incorrect Answer D :

Choice (D) results in unidiomatic phrasing. The phrase to be getting dogs and cats rid
of fleas is awkward and unidiomatic. The phrase to rid dogs and cats of fleas should
be used instead.

Explanation for Incorrect Answer E :

Choice (E) results in awkward and imprecise phrasing. The phrase can be gotten is
awkward, and it is not clear who or what can get a variety of diseases.

From SAT Online Course CUUS


2007 May Sunday SAT Answers and Explanations

4 While some marine scientists lament the spread of new diseases that are killing
coral reefs, others would have noted that the species of coral that do survive are
disease-resistant.

(A) reefs, others would have noted


that
(B) reefs; others would note if
(C) reefs, others note that
(D) reefs; others who note
(E) reefs, but others should note that

ANSWERS AND EXPLA NATIONS

Explanation for Correct Answer C :

Choice (C) is correct. It avoids the verb-tense error of the original by providing the
simple present-tense verb note.

Explanation for Incorrect Answer A :

Choice (A) involves an error in verb tense. The conjunction While links the two
clauses (some marine scientists lament the spread of new diseases . . . and others
[note] that the species of coral . . .) and indicates that they occur at the same time.
Therefore, it would be illogical for the first clause to be in present tense (lament) and
the second clause to be conditional (would have noted).

Explanation for Incorrect Answer B :

Choice (B) results in an error in verb tense. The conjunction While links the two
clauses (some marine scientists lament the spread of new diseases . . . and others .
. . note . . . the species of coral . . .) and indicates that they occur at the same time.
Therefore, it would be illogical for the first clause to use a present-tense verb (lament)
and the second clause to use a modal that implies an action to occur in the future
(would note).

Explanation for Incorrect Answer D :

Choice (D) results in faulty sentence structure. Two dependent clauses (While some
marine scientists lament the spread of new diseases . . . reefs and others who note
the species . . . are disease-resistant) are joined with a semicolon, resulting in a
sentence with no main verb.

Explanation for Incorrect Answer E :

Choice (E) results in redundancy and awkward phrasing. The conjunction While links

From SAT Online Course CUUS


2007 May Sunday SAT Answers and Explanations

the two clauses (some marine scientists lament the spread of new diseases . . . and
others . . . note that the species of coral . . .) and indicates contrast, making the
conjunction but unnecessary. In addition, it does not make sense to use the modal
should to illogically indicate that other scientists are under an obligation.

5 Josephine Baker rose to fame as a dancer, then using her resources to adopt many
needy children of all nationalities.

(A) Josephine Baker rose to fame as a dancer, then using


(B) Josephine Baker rising to fame as a dancer while using
(C) Josephine Baker, who rose to fame as a dancer, also using
(D) Josephine Baker, she rose to fame as a dancer and then used
(E) Josephine Baker, having risen to fame as a dancer, used

ANSWERS AND EXPLA NATIONS

Explanation for Correct Answer E :

Choice (E) is correct. It avoids the faulty sentence structure of the original by providing
a main verb (used) to carry out the action Josephine Baker . . . used her resources.

Explanation for Incorrect Answer A :

Choice (A) involves improper subordination. The subordinate phrase then using
should be replaced with and then used.

Explanation for Incorrect Answer B :

Choice (B) results in a sentence fragment. There is no main verb to carry out the action
of the sentence, only the participles rising and using.

Explanation for Incorrect Answer C :

Choice (C) results in a sentence fragment. There is no main verb to carry out the action
of the sentence, only the relative clause who rose to fame as a dancer and the
participle using.

Explanation for Incorrect Answer D :

Choice (D) results in improper coordination. The subject Josephine Baker is named,
so the comma and pronoun she should be deleted.

6 Roberto is from Italy, he can speak four languages: Italian, French, Spanish, and
English.

From SAT Online Course CUUS


2007 May Sunday SAT Answers and Explanations

(A) Roberto is from Italy, he can speak four languages


(B) Roberto, from Italy, who can speak four languages
(C) Roberto is from Italy, he is able to speak four languages
(D) Roberto, who is Italian and able to speak four languages
(E) Roberto, who is from Italy, can speak four languages

ANSWERS AND EXPLA NATIONS

Explanation for Correct Answer E :

Choice (E) is correct. It avoids the comma splice of the original by recasting the first
independent clause (Roberto is from Italy) as the relative clause who is from Italy.

Explanation for Incorrect Answer A :

Choice (A) involves a comma splice. Two independent clauses (Roberto is from Italy
and he can speak four languages . . .) are joined by only a comma.

Explanation for Incorrect Answer B :

Choice (B) results in a sentence fragment. There is no main verb to carry out the action
of the sentence, only the relative clause beginning with who can speak four
languages.

Explanation for Incorrect Answer C :

Choice (C) results in a comma splice. Two independent clauses (Roberto is from Italy
and he is able to speak four languages . . .) are joined by only a comma.

Explanation for Incorrect Answer D :

Choice (D) results in a sentence fragment. There is no main verb to carry out the action
of the sentence, only the relative clause beginning with who is Italian and able to
speak four languages.

7 The care exercised by the company in investigating side effects of various kinds of
drugs have won them their enviable reputation among scientists.

(A) have won them their


(B) have won them an
(C) have won it its
(D) has won for it their
(E) has won it an

From SAT Online Course CUUS


2007 May Sunday SAT Answers and Explanations

ANSWERS AND EXPLA NATIONS

Explanation for Correct Answer E :

Choice (E) is correct. It avoids the disagreement in number of the original by providing
a singular verb (has) for the singular subject care and a singular a singular pronoun
(it) to refer to the singular noun company.

Explanation for Incorrect Answer A :

Choice (A) involves disagreement in number. The singular subject care does not agree
with the plural verb have, and the plural pronouns them and their do not agree
with the singular noun company.

Explanation for Incorrect Answer B :

Choice (B) results in disagreement in number. The singular subject care does not
agree with the plural verb have, and the plural pronoun them does not agree with
the singular noun company.

Explanation for Incorrect Answer C :

Choice (C) results in subject-verb disagreement. The singular subject care does not
agree with the plural verb have.

Explanation for Incorrect Answer D :

Choice (D) results in a pronoun error. There is nothing in the sentence to which the
plural possessive pronoun their can logically refer.

8 Armed with new tools for the manipulation of genes and proteins, vaccines are
devised by scientists fighting everything from food poisoning to cancer.

(A) vaccines are devised by scientists fighting


(B) scientists, devising vaccines to fight
(C) scientists are the ones who are devising vaccines in order to fight
(D) scientists are devising vaccines to fight
(E) the fight by scientists who devise vaccines is for

ANSWERS AND EXPLA NATIONS

Explanation for Correct Answer D :

Choice (D) is correct. It avoids the modification error of the original by placing the
subject scientists closest the phrase modifying it (Armed with new tools for the mani

From SAT Online Course CUUS


2007 May Sunday SAT Answers and Explanations

pulation of genes and proteins ).

Explanation for Incorrect Answer A :

Choice (A) involves a modification error. The sentence illogically implies that vaccines
are Armed with new tools for the manipulation of genes and proteins.

Explanation for Incorrect Answer B :

Choice (B) results in a sentence fragment. There is no main verb to carry out the action
of the sentence, only the participle devising.

Explanation for Incorrect Answer C :

Choice (C) results in wordiness. The phrase are the ones who is not necessary and
should be deleted.

Explanation for Incorrect Answer E :

Choice (E) results in a modification error. The sentence illogically implies that the
fight is Armed with new tools for the manipulation of genes and proteins.

9 New York State has passed a law requiring that all legal contracts be written in
simple, understandable English.

(A) all legal contracts be written


(B) all legal contracts being written
(C) all legal contracts have to be written
(D) the writing of all legal contracts is
(E) when writing legal contracts, it should be

ANSWERS AND EXPLA NATIONS

Explanation for Correct Answer A :

Choice (A) is correct. It avoids the errors of the other options by appropriately
providing a verb phrase, be written, in the subjunctive mood.

Explanation for Incorrect Answer B :

Choice (B) results in faulty sentence structure. There is no main verb to carry out the
action of the clause all legal contracts . . . , only the present participle phrase being
written.

Explanation for Incorrect Answer C :

From SAT Online Course CUUS


2007 May Sunday SAT Answers and Explanations

Choice (C) results in redundancy. The verb requiring indicates that what the law
states is mandatory, so the phrasal modal have to is unnecessary.

Explanation for Incorrect Answer D :

Choice (D) results in an error in verb form. The phrase beginning with that is part of a
subordinate clause in the subjunctive mood, so the verb phrase should be in the
subjunctive form be, not the present tense is.

Explanation for Incorrect Answer E :

Choice (E) results in a vague pronoun. There is nothing in the sentence to which the
pronoun it can logically refer.

10 Carefully reading the manuscript, numerous grammatical errors were detected by


the copy editor.

(A) Carefully reading the manuscript, numerous grammatical errors were detected
by the copy editor.
(B) Carefully reading the manuscript, the copy editor detected numerous
grammatical errors.
(C) The reading of the manuscript by the copy editor carefully detected numerous
grammatical errors.
(D) When carefully reading the manuscript, numerous grammatical errors were
detected by the copy editor.
(E) The careful reading of the manuscript by the copy editor enabled detecting
numerous grammatical errors.

ANSWERS AND EXPLA NATIONS

Explanation for Correct Answer B :

Choice (B) is correct. It avoids the modification error of the original by placing the
subject (the copy editor ) closest to the phrase modifying it (Carefully reading the
manuscript).

Explanation for Incorrect Answer A :

Choice (A) involves improper modification. It does not make sense to say that
numerous grammatical errors were Carefully reading the manuscript.

Explanation for Incorrect Answer C :

Choice (C) results in an illogical statement. It does not make sense to say that The
reading of the manuscript was able to perform an action (detecting errors) carefully.

From SAT Online Course CUUS


2007 May Sunday SAT Answers and Explanations

Explanation for Incorrect Answer D :

Choice (D) results in improper modification. It does not make sense to say that
numerous grammatical errors were carefully reading the manuscript.

Explanation for Incorrect Answer E :

Choice (E) results in an error in verb form. The present participle detecting
inappropriately serves as the direct object of the verb enabled and should be replaced
with the noun phrase the detection of.

11 The Arts and Crafts movement at the turn of the twentieth century was a revolt
against the uniformity of objects mass-produced by the factories of the Industrial
Revolution.

(A) century was a revolt against the uniformity of objects mass-produced


(B) century revolted against objects that were mass-produced and
uniform
(C) century, being a revolt against uniform, mass-produced objects
(D) century was revolting against mass-produced, uniform objects
(E) century has revolted against uniformity in mass-producing objects

ANSWERS AND EXPLA NATIONS

Explanation for Correct Answer A :

Choice (A) is correct. It avoids the errors of the other options by providing the correct
form of the verb phrase was a revolt.

Explanation for Incorrect Answer B :

Choice (B) results in awkward phrasing. The phrase objects that were . . . uniform by
the factories is awkward and illogical.

Explanation for Incorrect Answer C :

Choice (C) results in a sentence fragment. There is no main verb to carry out the action
of the sentence, only the participle being.

Explanation for Incorrect Answer D :

Choice (D) results in awkward phrasing. The phrase mass-produced, uniform objects
by the factories is awkward.

Explanation for Incorrect Answer E :

From SAT Online Course CUUS


2007 May Sunday SAT Answers and Explanations

Choice (E) results in an error in verb form and awkward phrasing. The Arts and Craft
movement is described as occurring at the beginning of the twentieth century, yet the
verb phrase has revolted indicates a past action that ended at an indefinite time or
continues to the present. In addition, the phrase uniformity in mass-producing objects
by the factories is awkward and confusing.

12

While Sylvia is justifiably proud of the award that she won for placing first in the

science contest, but Carol should not feel disappointed about taking second

prize . No error

ANSWERS AND EXPLA NATIONS

Corrected Sentence: While Sylvia is justifiably proud of the award that she won for
placing first in the science contest, Carol should not feel disappointed about taking
second prize.

Explanation for Correct Answer C :

The error in this sentence occurs at (C). The conjunction While, occurring at the
beginning of the sentence, indicates that there is a contrast between the clauses
Sylvia is justifiably proud . . . and Carol should not feel disappointed . . . , so the
conjunction but is redundant and should be deleted.

Explanation for Incorrect Answer A :

There is no error at (A). The adverb justifiably appropriately modifies the adjective
proud, which modifies Sylvia. The preposition of idiomatically follows proud and
introduces the prepositional phrase of the award.

Explanation for Incorrect Answer B :

There is no error at (B). The pronoun she appropriately refers to Sylvia, and the
past-tense verb won refers to an action completed in the past.

Explanation for Incorrect Answer D :

There is no error at (D). The adjective disappointed appropriately follows the auxiliary
verb feel. The preposition about idiomatically follows the adjective disappointed
and introduces the prepositional phrase about taking second prize.

From SAT Online Course CUUS


2007 May Sunday SAT Answers and Explanations

Explanation for Incorrect Answer E : There is an error in the sentence.

13

Recently trolleys have been brought back to solve the problems of congestioncause

d by the automobile, the very vehicle that causes the trolleys demise years ago.

No error

ANSWERS AND EXPLA NATIONS

Corrected Sentence: Recently trolleys have been brought back to solve the problems
of congestion caused by the automobile, the very vehicle that caused the trolleys
demise years ago.

Explanation for Correct Answer D :

The error in the sentence occurs at (D). The phrase years ago indicates an action
completed in the past, so the present-tense verb causes should be replaced with the
past-tense verb caused.

Explanation for Incorrect Answer A :

There is no error at (A). The infinitive verb phrase to solve appropriately functions as
an adverbial phrase modifying the verb phrase have been brought back.

Explanation for Incorrect Answer B :

There is no error at (B). The plural noun problems serves as a direct object for the
transitive verb solve. The preposition of idiomatically follows the noun problems
and introduces the prepositional phrase of congestion.

Explanation for Incorrect Answer C :

There is no error at (C). The past participle caused appropriately modifies the noun
congestion, and the preposition by idiomatically follows caused and introduces the
prepositional phrase by the automobile.

Explanation for Incorrect Answer E : There is an error in the sentence.

From SAT Online Course CUUS


2007 May Sunday SAT Answers and Explanations

14

Jocelyn, my friend since junior high school, believes she is more well suited to a

career in the sciences than to the business career her parents have urged her to

pursue. No error

ANSWERS AND EXPLA NATIONS

Corrected Sentence: Jocelyn, my friend since junior high school, believes she is
better suited to a career in the sciences than to the business career her parents have
urged her to pursue.

Explanation for Correct Answer B :

The error in the sentence occurs at (B). The phrase more well suited is not idiomatic;
more well should be replaced with the idiomatic comparative adverb better.

Explanation for Incorrect Answer A :

There is no error at (A). The preposition since appropriately introduces the


prepositional phrase since junior high school, which modifies the noun friend.

Explanation for Incorrect Answer C :

There is no error at (C). The conjunction than appropriately links the parallel phrases
to a career in the sciences and to the business career.

Explanation for Incorrect Answer D :

There is no error at (D). The plural present prefect verb phrase have urged agrees
with the plural subject parents and indicates an action that was completed at a
nonspecific time in the past. The pronoun her appropriately refers to Jocelyn and
serves as the object of the verb phrase.

Explanation for Incorrect Answer E : There is an error in the sentence.

15

One of the playwrights hopes was that the play would shock theatergoers; instead,

it succeeded only in confusing them. No error

From SAT Online Course CUUS


2007 May Sunday SAT Answers and Explanations

ANSWERS AND EXPLA NATIONS

Corrected Sentence: One of the playwrights hopes was that the play would shock
theatergoers; instead, it succeeded only in confusing them.

Explanation for Correct Answer E : There is no error in this sentence.

Explanation for Incorrect Answer A :

There is no error at (A). The singular verb was agrees with the singular subject
One. The conjunction that appropriately introduces the clause the play would shock
theatergoers.

Explanation for Incorrect Answer B :

There is no error at (B). The verb phrase would shock appropriately indicates a
predicted action.

Explanation for Incorrect Answer C :

There is no error at (C). The past-tense verb succeeded appropriately indicates an


action that was completed in the past.

Explanation for Incorrect Answer D :

There is no error at (D). The preposition in idiomatically follows the verb succeeded
and appropriately introduces the prepositional phrase in confusing them.

16

Bill simultaneously listened to music and did homework at the same time because

he believed that music enabled him to relax while he studied. No error

ANSWERS AND EXPLA NATIONS

Corrected Sentence: Bill simultaneously listened to music and did homework because
he believed that music enabled him to relax while he studied.

Explanation for Correct Answer B :

The error in the sentence occurs at (B). The adverb simultaneously indicates how Bill
listened to music and did homework, so the phrase at the same time is redundant

From SAT Online Course CUUS


2007 May Sunday SAT Answers and Explanations

and should be deleted.

Explanation for Incorrect Answer A :

There is no error at (A). The past-tense verb did appropriately indicates an action
completed in the past, as indicated by the past-tense verbs listened, believed, and
enabled, and is the idiomatic verb for the noun homework.

Explanation for Incorrect Answer C :

There is no error at (C). The pronoun him appropriately refers to Bill in the
objective case.

Explanation for Incorrect Answer D :

There is no error at (D). The infinitive verb to relax appropriately serves as the
indirect object of the verb enabled, and the conjunction while links the actions of
enabled him to relax and studied.

Explanation for Incorrect Answer E : There is an error in the sentence.

17

Vaudeville was an early training ground for many great American artists who

serveapprenticeships there before going on to successful careers in theater, film,

or television. No error

ANSWERS AND EXPLA NATIONS

Corrected Sentence: Vaudeville was an early training ground for many great
American artists who served apprenticeships there before going on to successful
careers in theater, film, or television.

Explanation for Correct Answer C :

The error in the sentence occurs at (C). The verb was indicates that the actions of the
sentence occurred in the past, making the present-tense verb serve inappropriate. It
should be replaced with the past-tense verb served.

Explanation for Incorrect Answer A :

There is no error at (A). The adjective early appropriately modifies the noun phrase t

From SAT Online Course CUUS


2007 May Sunday SAT Answers and Explanations

raining ground.

Explanation for Incorrect Answer B :

There is no error at (B). The present participle training appropriately modifies the
noun ground. The preposition for appropriately follows the noun phrase training
ground and introduces the prepositional phrase for many great American artists.

Explanation for Incorrect Answer D :

There is no error at (D). The preposition before appropriately introduces the idiomatic
prepositional phrase before going on to successful careers.

Explanation for Incorrect Answer E : There is an error in the sentence.

18

Biologists which have studied coyotes claim that living in packs enables the animals

to defend their food supplies against marauders. No error

ANSWERS AND EXPLA NATIONS

Corrected Sentence: Biologists who have studied coyotes claim that living in packs
enables the animals to defend their food supplied against marauders.

Explanation for Correct Answer A :

The error in the sentence occurs at (A). The relative pronoun which unidiomatically
refers to people and inappropriately indicates a nonrestrictive clause; it should be
replaced with the relative pronoun who.

Explanation for Incorrect Answer B :

There is no error at (B). The plural verb claim agrees with the plural subject
Biologists. The conjunction that appropriately introduces the clause living in packs
enables . . .

Explanation for Incorrect Answer C :

There is no error at (C). The singular verb enables agrees with the singular subject
living in packs.

Explanation for Incorrect Answer D :

From SAT Online Course CUUS


2007 May Sunday SAT Answers and Explanations

There is no error at (D). The plural possessive pronoun their appropriately refers to
the plural noun animals.

Explanation for Incorrect Answer E : There is an error in the sentence.

19

Even when Barbara Jordan put questions toward a political nominee, her

elegantdiction evoked in listeners memories of her eloquent political speeches. N

o error

ANSWERS AND EXPLA NATIONS

Corrected Sentence: Even when Barbara Jordan put questions to a political nominee,
her elegant diction evoked in listeners memories of her eloquent political speeches.

Explanation for Correct Answer B :

The error in the sentence occurs at (B). The preposition toward is unidiomatic in this
context and should be replaced with the preposition to.

Explanation for Incorrect Answer A :

There is no error at (A). The conjunction when links the actions of Barbara Jordan
put questions [to] a political nominee and her elegant diction evoked in listeners . . .
speeches. In addition, the adverb even appropriately modifies the phrase when
Barbara Jordan put questions [to] a political nominee

Explanation for Incorrect Answer C :

There is no error at (C). The past-tense verb evoked appropriately indicates an action
completed in the past by the subject diction.

Explanation for Incorrect Answer D :

There is no error at (D). The plural noun memories serves as the direct object for the
verb evoked. The preposition of idiomatically follows the noun memories and
appropriately introduces the prepositional phrase of her eloquent political speeche s.

Explanation for Incorrect Answer E : There is an error in the sentence.

From SAT Online Course CUUS


2007 May Sunday SAT Answers and Explanations

20

With the current difficulty in filling seats on juries, some argue that jurors should

be compensated for lost wages, particularly given the increasing length of trials.

No error

ANSWERS AND EXPLA NATIONS

Corrected Sentence: With the current difficulty in filling seats on juries, some argue
that jurors should be compensated for lost wages, particularly given the increasing
length of trials.

Explanation for Correct Answer E : There is no error in this sentence.

Explanation for Incorrect Answer A :

There is no error at (A). The gerund filling is an appropriate part of the prepositional
phrase in filling seats.

Explanation for Incorrect Answer B :

There is no error at (B). The indefinite pronoun some appropriately refers to a


nonspecific group of people.

Explanation for Incorrect Answer C :

There is no error at (C). The preposition for idiomatically follows the past participle
compensated and introduces the prepositional phrase for lost wages, in which the
adjective lost appropriately modifies the noun wages.

Explanation for Incorrect Answer D :

There is no error at (D). The present participle increasing appropriately modifies the
noun phrase length of trials.

21

No one who travels on the expressways that circle our cities can help but

noticingthe growing number of junkyards that defile the beauty of the countryside.

From SAT Online Course CUUS


2007 May Sunday SAT Answers and Explanations

No error

ANSWERS AND EXPLA NATIONS

Corrected Sentence: No one who travels on the expressways that circle our cities can
help but notice the growing number of junkyards that defile the beauty of the
countryside.

Explanation for Correct Answer B :

The error in the sentence occurs at (B). The phrase help but noticing is unidiomatic
and should be replaced with the phrase help but notice.

Explanation for Incorrect Answer A :

There is no error at (A). The singular verb travels agrees with the singular subject
No one. The preposition on idiomatically follows the verb travels and appropriately
introduces the prepositional phrase on the expressways.

Explanation for Incorrect Answer C :

There is no error at (C). The present participle growing appropriately modifies the
noun number.

Explanation for Incorrect Answer D :

There is no error at (D). The relative clause beginning with that modifies junkyards.
The plural verb defile, which appears in the relative clause, agrees with the plural
subject junkyards.

Explanation for Incorrect Answer E : There is an error in the sentence.

22

Muffins made from whole-grain graham flour are coarser in texture butmore

flavorful than white flour. No error

ANSWERS AND EXPLA NATIONS

Corrected Sentence: Muffins made from whole-grain graham flour are coarser in
texture but more flavorful than those made from white flour.

From SAT Online Course CUUS


2007 May Sunday SAT Answers and Explanations

Explanation for Correct Answer D :

The error in this sentence occurs at (D). The sentence makes an illogical comparison
between muffins and white flour. Option (D) should be replaced with the phrase
than those made from white flour so that the sentence compares muffins to muffins.

Explanation for Incorrect Answer A :

There is no error at (A). The past participle made appropriately modifies muffins,
and the preposition from idiomatically follows made.

Explanation for Incorrect Answer B :

There is no error at (B). The plural verb are agrees with the plural subject muffins.

Explanation for Incorrect Answer C :

There is no error at (C). The comparative adverb more appropriately compares one
thing (muffins made from whole-grain graham flour) with one other (muffins made
from white flour). The adjective flavorful modifies muffins.

Explanation for Incorrect Answer E : There is an error in the sentence.

23

In their search for ways to extend the human life span and warding off diseases,

scientists find themselves focusing not on expanding the diet but rather on limit ing

it . No error

ANSWERS AND EXPLA NATIONS

Corrected Sentence: In their search for ways to extend the human life span and to
ward off diseases, scientists find themselves focusing not on expanding the diet, but
rather on limiting it.

Explanation for Correct Answer A :

The error in the sentence occurs at (A). The verb phrase warding off is not parallel
with the preceding verb phrase to extend and should be replaced with to ward off.

Explanation for Incorrect Answer B :

From SAT Online Course CUUS


2007 May Sunday SAT Answers and Explanations

There is no error at (B). The plural verb find agrees with the plural subject
scientists, which is the antecedent of the plural reflexive pronoun themselves.

Explanation for Incorrect Answer C :

There is no error at (C). The conjunction but links the actions of expanding the diet
and limiting it, and the adverb rather appropriately modifies the phrase on limiting
it.

Explanation for Incorrect Answer D :

There is no error at (D). The verb phrase limiting it is parallel to the preceding verb
phrase expanding the diet. The pronoun it appropriately refers to the noun phrase
the diet.

Explanation for Incorrect Answer E : There is an error in the sentence.

24

The frustrated director held auditions longer than expected because sheis unable

to find someone who could convincingly portray a character as complex as

Dardanella. No error

ANSWERS AND EXPLA NATIONS

Corrected Sentence: The frustrated director held auditions longer than expected
because she was unable to find someone who could convincingly portray a character as
complex as Dardanella.

Explanation for Correct Answer A :

The error in the sentence occurs at (A). The actions of the sentence occurred in the
past, as indicated by the past-tense verb held, so the present-tense verb is is
inappropriate and should be replaced with the past-tense verb was.

Explanation for Incorrect Answer B :

There is no error at (B). The relative pronoun who appropriately refers to the
indefinite pronoun someone and introduces the restrictive clause who could
convincingly portray . . . Dardanella.

Explanation for Incorrect Answer C :

From SAT Online Course CUUS


2007 May Sunday SAT Answers and Explanations

There is no error at (C). The adverb convincingly appropriately modifies the verb
portray, which is part of the idiomatic verb phrase could . . . portray.

Explanation for Incorrect Answer D :

There is no error at (D). The correlative conjunction as . . . as appropriately links the


adjective complex with the noun Dardanella.

Explanation for Incorrect Answer E : There is an error in the sentence.

25

Neither a percolator nor a drip coffeepot are likely to produce good coffee if the

water used for brewing is overly chlorinated. No error

ANSWERS AND EXPLA NATIONS

Corrected Sentence: Neither a percolator nor a drip coffeepot is likely to produce


good coffee if the water used for brewing is overly chlorinated.

Explanation for Correct Answer B :

The error in the sentence occurs at (B). When the correlative conjunction Neither . . .
nor is used, the subject closest to the verb determines whether the verb is singular or
plural. The plural verb are does not agree with the singular subject coffeepot.

Explanation for Incorrect Answer A :

There is no error at (A). The conjunction nor is appropriately paired with Neither to
form the correlative conjunction Neither . . . no r.

Explanation for Incorrect Answer C :

There is no error at (C). The past participle used appropriately modifies the noun
water. The preposition for idiomatically follows used and introduces the
prepositional phrase for brewing.

Explanation for Incorrect Answer D :

There is no error at (D). The singular verb is agrees with the singular subject water.

Explanation for Incorrect Answer E : There is an error in the sentence.

From SAT Online Course CUUS


2007 May Sunday SAT Answers and Explanations

26

Long neglected by restaurants in the United States, Portuguese cookingis

attracting attention with their colorful, highly flavored, and complex dishes. No

error

ANSWERS AND EXPLA NATIONS

Corrected Sentence: Long neglected by restaurants in the United States, Portuguese


cooking is attracting attention with its colorful, highly flavored, and complex dishes.

Explanation for Correct Answer C :

The error in the sentence occurs at (C). The plural possessive pronoun their does not
agree with the singular subject Portuguese cooking and should be replaced with the
singular possessive pronoun its.

Explanation for Incorrect Answer A :

There is no error at (A). In the context of the sentence, long is appropriately used as
an adverb to modify the past participle neglected, which modifies the noun phrase
Portuguese cooking.

Explanation for Incorrect Answer B :

There is no error (B). The singular present progressive verb phrase is attracting
agrees with the singular subject Portuguese cooking and is appropriate to describe a
currently occurring action that will continue.

Explanation for Incorrect Answer D :

There is no error (D). The adverb highly appropriately modifies the adjective
flavored.

Explanation for Incorrect Answer E : There is an error in the sentence.

27

Though Rivers had done all the work, Peters had taken all the credit, and sohe held

him in contempt . No error

From SAT Online Course CUUS


2007 May Sunday SAT Answers and Explanations

ANSWERS AND EXPLA NATIONS

Corrected Sentence: Though Rivers had done all the work, Peters had taken all the
credit, and so Rivers held him in contempt.

Explanation for Correct Answer C :

The error in the sentence occurs at (C). The use of two vague pronouns (he and
him ) does not clearly indicate whether Rivers held Peters in contempt or Peters held
Rivers in contempt. One of the pronouns should be replaced with a proper noun.

Explanation for Incorrect Answer A :

There is no error at (A). The past perfect verb phrase had done appropriately
indicates an action that was completed before another action in the past (held him in
contempt). The adjective all appropriately modifies the noun phrase the work.

Explanation for Incorrect Answer B :

There is no error at (B). The adjective all appropriately modifies the noun phrase the
credit, and the phrase all the credit functions as the direct object of the verb phrase
had taken.

Explanation for Incorrect Answer D :

There is no error at (D). The adverbial phrase in contempt appropriately indicates


how the direct object him was held.

Explanation for Incorrect Answer E : There is an error in the sentence.

28

These novels move willing readers away from their humdrum lives and into a

worldthat is at once fantastic and mysterious. No error

ANSWERS AND EXPLA NATIONS

Corrected Sentence: These novels move willing readers away from their humdrum
lives and into a world that is at once fantastic and mysterious.

Explanation for Correct Answer E : There is no error in this sentence.

From SAT Online Course CUUS


2007 May Sunday SAT Answers and Explanations

Explanation for Incorrect Answer A :

There is no error at (A). The adjective willing appropriately modifies the noun
readers.

Explanation for Incorrect Answer B :

There is no error at (B). The plural possessive pronoun their appropriately refers to
the plural noun readers. The adjective humdrum modifies the plural noun lives.

Explanation for Incorrect Answer C :

There is no error at (C). The preposition into appropriately introduces the


prepositional phrase into a world that is at once fantastic and mysterious, which
modifies the verb move.

Explanation for Incorrect Answer D :

There is no error at (D). The idiom at once, meaning at the same time,
appropriately precedes the adjectives fantastic and mysterious.

29

Only by the afterglow of the sunset could the hikers find their path to the campsite

at so late an h our. No error

ANSWERS AND EXPLA NATIONS

Corrected Sentence: Only by the afterglow of the sunset could the hikers find their
path to the campsite at so late an hour.

Explanation for Correct Answer E : There is no error in this sentence.

Explanation for Incorrect Answer A :

There is no error at (A). The adverb only appropriately introduces the prepositional
phrase by the afterglow of sunset and modifies the verb phrase could . . . find.

Explanation for Incorrect Answer B :

There is no error at (B). The modal verb could appropriately indicates that the
subjects, the hikers, were able to complete an action.

From SAT Online Course CUUS


2007 May Sunday SAT Answers and Explanations

Explanation for Incorrect Answer C :

There is no error at (C). The plural possessive pronoun their appropriately refers to
the plural noun hikers.

Explanation for Incorrect Answer D :

There is no error at (D). The adverb so appropriately modifies the adjective late,
which modifies the noun phrase an hour.

(1) In Los Angeles, California, painted on the side of a cement-lined flood


channel, there is a mural. (2) This mural is called The Great Wall of Los Angeles , it
is ten feet high and stretches for over a half mile. (3) Twenty-five artists designed
The Wall's forty panels. (4) Over a period that spanned seven summers, they
supervised the hundreds of teenagers from various communities in Los Angeles
who painted the panels. (5) The project, inspired by the famous murals of Mexico
City, was directed by artist Judith Baca. (6) Ms. Baca has been in charge of the
work on the mural since it began in the 1970's.

(7) Mexico City's murals were painted fifty years ago exclusively by renowned
artists in the city's grandest buildings. (8) In Los Angeles, however, over 2,000
murals have been painted in the last thirty years by ordinary citizens and
community activists as well as artists. (9) The murals range from small creations
painted by one or two artists to gigantic projects like The Wall .

(10) Each section of The Wall has a story to tell about the peoples of California.
(11) Community issues are depicted on The Wall. (12) Unfortunately, The
Wall ends in the mid-1950's. (13) One panel, Division of the Barrios and Chavez
Ravine, portrays the way freeways divided families and neighborhoods within the
city. (14) Ms. Baca hopes that the communities of Los Angeles will add new
panels and bring The Wall up-to-date.

30 Which of the following is the best version of the underlined part of sentence 2
(reproduced below) ?

This mural is called The Great Wall of Los Angeles, it is ten feet high and stretches
for over a half mile.

(A) (as it is now)


(B) Angeles, therefore
(C) Angeles because it is

From SAT Online Course CUUS


2007 May Sunday SAT Answers and Explanations

(D) Angeles; moreover, it is


(E) Angeles, having been built

ANSWERS AND EXPLA NATIONS

Explanation for Correct Answer C :

Choice (C) is correct. It avoids the comma-splice error of the original by appropriately
combining the two independent clauses (This mural is called The Great Wall of Los
Angeles and it is ten feet high and stretches for over a half mile) with the
conjunction because, a conjunction that indicates the murals name comes from its
immense size.

Explanation for Incorrect Answer A :

Choice (A) is unsatisfactory. As it is now, the sentence has a comma-splice error. It


joins two independent clauses (This mural is called The Great Wall of Los Angeles and
it is ten feet high and stretches for over a half mile) with only a comma.

Explanation for Incorrect Answer B :

Choice (B) is unsatisfactory. The conjunction therefore here reverses the cause/effect
relationship between the two ideas in the sentence. The sentence should say that the
therefore, it is called The
mural is ten feet high and stretches for over a half mile; therefore
Great Wall of Los Angeles. In addition, joining the two clauses with the conjunction
therefore requires a semicolon, not just a comma.

Explanation for Incorrect Answer D :

Choice (D) is unsatisfactory. The conjunction moreover does not appropriately


establish the cause/effect relationship between the two ideas in the sentence. The
mural is called The Great Wall of Los Angeles as a result of its being ten feet high and
over half a mile long, not in addition to its size.

Explanation for Incorrect Answer E :

Choice (E) is unsatisfactory. Joining the two ideas in the sentence with the phrase
having been built is illogical. The sentence is describing a mural or painting called The
Great Wall of Los Angeles, not an actual wall, so the verb built does not make sense
in this context. Furthermore, the verb phrase having been built is not parallel with the
following verb stretches.

31 In context, which is the best way to combine the underlined part of sentences 5
and 6 (reproduced below) ?

From SAT Online Course CUUS


2007 May Sunday SAT Answers and Explanations

The project, inspired by the famous murals of Mexico City, was directed by artist
Judith Baca. Ms. Baca has been in charge of the work on the mural since it began
in the 1970s.

(A) is directed by artist Judith Baca, who was


(B) was directed by artist Judith Baca, who has been
(C) artist Judith Baca directing, who has been
(D) was directed and in the charge of Judith Baca, an artist,
(E) was directed by artist Judith Baca, and Ms. Baca was

ANSWERS AND EXPLA NATIONS

Explanation for Correct Answer B :

Choice (B) is correct. The relative clause who has been in charge . . . since it began in
the 1970s appropriately modifies Judith Baca.

Explanation for Incorrect Answer A :

Choice (A) is unsatisfactory. There is a verb-tense shift between the present-tense verb
phrase is directed and the past-tense verb was in was in charge. To imply that Ms.
Baca is still in charge (as the first present-tense verb does), the present perfect has
been in charge should be used.

Explanation for Incorrect Answer C :

Choice (C) is unsatisfactory. Combining sentences 5 and 6 in this manner creates a


sentence fragment. There is no main verb to carry out the action of the sentence.

Explanation for Incorrect Answer D :

Choice (D) is unsatisfactory. It is not idiomatic to say that the project was directed and
in the charge of Judith Baca. The verb phrase was directed requires the preposition
by to follow it. In addition, it is redundant to say that the project was . . . in the
charge of Judith Baca, an artist, in charge of the work.

Explanation for Incorrect Answer E :

Choice (E) is unsatisfactory. The two clauses (The project . . .was directed by artist
Judith Baca and Ms. Baca was in charge . . .since it began in the 1970s) are not of
equal importance. It makes more sense for the second clause about Ms. Baca to appear
as relative clause modifying artist Judith Baca.

From SAT Online Course CUUS


2007 May Sunday SAT Answers and Explanations

32 Which of the following would most improve sentence 7 (reproduced below) ?

Mexico Citys murals were painted fifty years ago exclusively by renowned artists
in the citys grandest buildings.

(A) Insert amazing before murals.


(B) Change were painted to have been painted.
(C) Change the city s to their.
(D) Change grandest to the most grand.
(E) Move in the city s grandest buildings after painted.

ANSWERS AND EXPLA NATIONS

Explanation for Correct Answer E :

Choice (E) is correct. Moving in the citys grandest buildings after painted makes
clear the fact that in the citys grandest buildings modifies painted, not artists.

Explanation for Incorrect Answer A :

Choice (A) is unsatisfactory. Inserting the adjective amazing before murals does not
improve the clarity of sentence 7. It remains unclear whether in the citys grandest
buildings modifies artists or painted.

Explanation for Incorrect Answer B :

Choice (B) is unsatisfactory. Changing the past-tense verb phrase were painted,
which indicates an action completed in the past, to the present perfect tense verb
phrase have been painted, which indicates an action completed at an unspecified time
before now, is illogical. Sentence 7 clearly states that the murals were painted fifty
years ago.

Explanation for Incorrect Answer C :

Choice (C) is unsatisfactory. Changing the citys to their would create an improper
pronoun reference, inappropriately implying that the buildings belonged to the artists,
not to Mexico City.

Explanation for Incorrect Answer D :

Choice (D) is unsatisfactory. Changing grandest to most grand does not improve the
clarity of sentence 7. It remains unclear whether in the citys . . . buildings modifies
artists or painted.

From SAT Online Course CUUS


2007 May Sunday SAT Answers and Explanations

33 What is the primary purpose of sentence 7?

(A) To provide a meaningful contrast


(B) To emphasize the main point of the passage
(C) To demonstrate a standard that is difficult to achieve
(D) To leave the reader with a feeling of pride
(E) To repeat a claim made earlier in the passage

ANSWERS AND EXPLA NATIONS

Explanation for Correct Answer A :

Choice (A) is correct. Sentence 7 explains that the murals of Mexico City were painted
by renowned artists and in the city s grandest buildings. These qualities are in
contrast to the murals of Los Angeles that were painted in places such as on the side
of a cement-lined flood channel by ordinary citizens and community activists.

Explanation for Incorrect Answer B :

Choice (B) is unsatisfactory. The main point of the passages is the murals in Los
Angeles, specifically The Great Wall of Los Angeles. It does not make sense to say that
the purpose of sentence 7, which is about the murals of Mexico City, is to emphasize
the main point of the passage.

Explanation for Incorrect Answer C :

Choice (C) is unsatisfactory. Sentence 7 is about the murals of Mexico City that were
painted in the citys grandest buildings by renowned artists. The murals of Los
Angeles were inspired by the famous murals of Mexico City. The murals of Mexico City
are not a standard that the Los Angeles murals are trying to achieve.

Explanation for Incorrect Answer D :

Choice (D) is unsatisfactory. Sentence 7 describes the murals of Mexico City as a way
to provide contrast between those murals and the murals of Los Angeles. It does not
make sense to say that the purpose of sentence 7 is to leave the reader with a feeling
of pride.

Explanation for Incorrect Answer E :

Choice (E) is unsatisfactory. It does not make sense to say that the purpose of
sentence 7 is to repeat a claim made earlier in the passage. The murals of Mexico City

From SAT Online Course CUUS


2007 May Sunday SAT Answers and Explanations

are mentioned only once before in the passage and are described as the inspiration for
the mural project in Los Angeles.

34 In context, which is the best revision of the underlined part of sentence 11


(reproduced below) ?

Community issues are depicted on The Wall.

(A) issues, however, are


(B) issues affecting everyone
(C) issues, in contrast, are
(D) issues, in particular, are
(E) issues, then, were

ANSWERS AND EXPLA NATIONS

Explanation for Correct Answer D :

Choice (D) is correct. Sentence 10 explains that each section of the mural has a story
to tell about the peoples of California. It is appropriate for sentence 11 to make the
slightly more specific statement that Community issues, in particular, are depicted on T
he Wall.

Explanation for Incorrect Answer A :

Choice (A) is unsatisfactory. Revising sentence 11 to include the conjunction however


inappropriately implies that there is a contrast between the information in sentence 11
(Community issues are depicted on The Wall) and the information before it (each
section of the mural has a story to tell about the peoples of California). The
information in sentences 10 and 11 is not in contrast.

Explanation for Incorrect Answer B :

Choice (B) is unsatisfactory. Revising sentence 11 in this manner would create the
following ungrammatical sentence: Community issues affecting everyone depicted on T
he Wall. The linking verb are must precede depicted.

Explanation for Incorrect Answer C :

Choice (C) is unsatisfactory. Revising sentence 11 to include the phrase in contrast is


illogical. It does not make sense to say that Each section of The Wall has a story to tell
about the peoples of California. Community issues, in contrast, are depicted on The

From SAT Online Course CUUS


2007 May Sunday SAT Answers and Explanations

Wall. Community issues are a type of story about the people of California, not
something in contrast to the stories.

Explanation for Incorrect Answer E :

Choice (E) is unsatisfactory. Sentence 11 provides a detail to support the statement


made in sentence 10 about how each section of the mural has a story to tell about the
peoples of California. Revising sentence 11 to Community issues, then, were depicted
on The Wall inappropriately implies that sentence 11 is the next event in a series
instead of an additional detail.

35 Where should sentence 12 go?

(A) Where it is now


(B) After sentence 1
(C) After sentence 8
(D) After sentence 13
(E) After sentence 14

ANSWERS AND EXPLA NATIONS

Explanation for Correct Answer D :

Choice (D) is correct. Sentences 10 and 11 explain that the mural is made up of
sections and that the sections depict various aspects of community life. Sentence 13
describes one particular panel in detail. It makes sense at that point in the paragraph
to note that the mural depicts community events only until the 1950s, especially since
the next sentence describes Ms. Bacas hopes to add contemporary panels to the mural.

Explanation for Incorrect Answer A :

Choice (A) is unsatisfactory. It does not make sense to note that the murals depiction
of community events ends in the mid-1950s and then go on to describe a particular
panel as is done in sentence 13. It would make more sense to describe a particular
panel in detail first, and then note that the panels end in the mid-1950s, followed by
the discussion of Ms. Bacas desire to add more contemporary panels to the mural.

Explanation for Incorrect Answer B :

Choice (B) is unsatisfactory. It does not make sense to include a sentence describing a
particular panel of The Great Wall of Los Angeles immediately after the mural is first
mentioned in sentence 1 and before the murals name is mentioned in sentence 2.

From SAT Online Course CUUS


2007 May Sunday SAT Answers and Explanations

Explanation for Incorrect Answer C :

Choice (C) is unsatisfactory. The second paragraph, in which sentence 8 appears, is


primarily about the various murals in Los Angeles, not specifically The Great Wall of Los
Angeles. It would not make sense to include a sentence that describes a specific part of
a specific mural in a paragraph that is about Los Angeles murals in general.

Explanation for Incorrect Answer E :

Choice (E) is unsatisfactory. It does not make sense to include the sentence that states
that the events the mural depicts end in the mid-1950s after the sentence that states
Ms. Baca hopes that the communities of Los Angeles will add new panels and bring The
Wall up-to-date. For Ms. Bacas hopes to make sense, the reader must know thatThe
Wall ends in the mid-1950s first.

Section7: Critical Reading

View Answers and Explanations

Online - Practice Test #2

1 Once thought to be ------- , the dawn redwood was rediscovered in the 1940s, at
which time its continued survival seemed ------- ; today, however, it is commonly
cultivated.

(A) immense . . natural


(B) adaptable . . astonishing
(C) defunct . . inevitable
(D) perennial . . doubtful
(E) extinct . . precarious

ANSWERS AND EXPLA NATIONS

Explanation for Correct Answer E :

Choice (E) is correct. "Extinct" means no longer in existence. Precarious" means


uncertain. The sentence indicates that the dawn redwood was "rediscovered," which
suggests that it had previously been lost or had disappeared; therefore, it makes sense
to say that people once thought the dawn redwood was extinct, or no longer existed.
The term "however" near the end of the sentence indicates a contrast with what has
come before. If the tree is commonly cultivated or grown today, it is logical to conclude

From SAT Online Course CUUS


2007 May Sunday SAT Answers and Explanations

that the tree must have once been very uncommon or rare; the survival of such a
treeespecially one that had once been thought to be extinctlikely seemed
precarious, or uncertain, at that time.

Explanation for Incorrect Answer A :

Choice (A) is incorrect. "Immense" means huge. In this context, natural" means
obvious or taken for granted. The sentence indicates that the dawn redwood was
"rediscovered," which suggests that it had previously been lost or had disappeared. The
tree's size is not an issue, and the sentence does not indicate that the tree is immense,
or huge. Further, the term "however" near the end of the sentence indicates a contrast
with what has come before. The term natural does not support this contrast, because
one would expect a tree whose "survival seemed natural" to be "commonly cultivated,"
or commonly grown.

Explanation for Incorrect Answer B :

Choice (B) is incorrect. "Adaptable" means able to change or adapt. "Astonishing"


means surprising. The sentence indicates that the dawn redwood was rediscovered,
which suggests that it had previously been lost or had disappeared. It is somewhat
illogical to say that a tree once thought to be adaptable would have to be rediscovered,
or that the continued survival of such a tree would be astonishing; a tree that is
adaptable likely would be expected to survive because it is able to change as needed.

Explanation for Incorrect Answer C :

Choice (C) is incorrect. "Defunct" means no longer functioning. "Inevitable" means


unavoidable or automatic. The sentence indicates that the dawn redwood was
rediscovered, which suggests that it had previously been lost or had disappeared;
therefore, it might make sense to say that the tree was once thought to be defunct. But
the term "however" near the end of the sentence indicates a contrast with what has
come before, and the term inevitable does not support this contrast; one would
expect a tree whose "survival seemed inevitable" to be "commonly cultivated," or
commonly grown.

Explanation for Incorrect Answer D :

Choice (D) is incorrect. In this context, "perennial" means present throughout the year.
"Doubtful" means unlikely. The sentence indicates that the dawn redwood was
rediscovered, which suggests that it had previously been lost or had disappeared. It
is somewhat illogical to say that a tree once thought to be perennial would have to be
rediscovered, or that the continued survival of such a tree would seem doubtful; there
is no reason to doubt that a tree that is always present would continue to survive.

From SAT Online Course CUUS


2007 May Sunday SAT Answers and Explanations

2 As the name suggests, a geochronologist is a scientist who ------- terrestrial


materials in order to ------- them.

(A) combines . . sequence


(B) gathers . . display
(C) analyzes . . date
(D) examines . . excavate
(E) studies . . synthesize

ANSWERS AND EXPLA NATIONS

Explanation for Correct Answer C :

Choice (C) is correct. When determining the meaning of an unfamiliar word like
"geochronologist," it is useful to consider similar and more familiar words such as
"geologist" (a person who studies the Earth's history) and "chronological" (arranged in
a time sequence). It is also helpful to know the common roots "geo," meaning earth,
and "chrono," meaning time. It is evident from these clues that a geochronologist is
someone who analyzes, or studies, the Earth and focuses on time or dates. Such a
person analyzes "terrestrial," or earthly, materials in order to date them.

Explanation for Incorrect Answer A :

Choice (A) is incorrect. When determining the meaning of an unfamiliar word like
"geochronologist," it is useful to consider similar and more familiar words such as
"geologist" (a person who studies the Earth's history) and "chronological" (arranged in
a time sequence). It is also helpful to know the common roots "geo," meaning earth,
and "chrono," meaning time. It is evident from these clues that a geochronologist is
someone who studies the Earth and focuses on time or dates. There is no indication
that a geochronologist is someone who combines, or blends, "terrestrial," or earthly,
materials in order to sequence them, or arrange them in succession.

Explanation for Incorrect Answer B :

Choice (B) is incorrect. When determining the meaning of an unfamiliar word like
"geochronologist," it is useful to consider similar and more familiar words such as
"geologist" (a person who studies the Earth's history) and "chronological" (arranged in
a time sequence). It is also helpful to know the common roots "geo," meaning earth,
and "chrono," meaning time. It is evident from these clues that a geochronologist is
someone who studies the Earth and focuses on time or dates. There is no indication
that a geochronologist is someone who gathers, or collects "terrestrial," or earthly,
materials in order to display them, or put them on view.

Explanation for Incorrect Answer D :

From SAT Online Course CUUS


2007 May Sunday SAT Answers and Explanations

Choice (D) is incorrect. When determining the meaning of an unfamiliar word like
"geochronologist," it is useful to consider similar and more familiar words such as
"geologist" (a person who studies the Earth's history) and "chronological" (arranged in
a time sequence). It is also helpful to know the common roots "geo," meaning earth,
and "chrono," meaning time. It is evident from these clues that a geochronologist is
someone who studies the Earth and focuses on time or dates. Although a
geochronologist would examine, or closely inspect, terrestrial, or earthly, materials, it
does not make sense to suggest that he or she would examine them in order to
excavate them, or dig them upthe materials would likely need to be excavated befor
e they could be examined.

Explanation for Incorrect Answer E :

Choice (E) is incorrect. When determining the meaning of an unfamiliar word like
"geochronologist," it is useful to consider similar and more familiar words such as
"geologist" (a person who studies the Earth's history) and "chronological" (arranged in
a time sequence). It is also helpful to know the common roots "geo," meaning earth,
and "chrono," meaning time. It is evident from these clues that a geochronologist is
someone who studies the Earth and focuses on time or dates. Although a
geochronologist would engage in the study of terrestrial, or earthly, materials, there
is no indication that such a scientist would synthesize, or combine into a single whole,
the various materials he or she studies.

3 Contemporary Inuit sculpture merges traditional carving techniques with current


subject matter; thus, it ------- an Inuit practice while ------- a modern cultural
identity.

(A) invalidates . . manifesting


(B) disregards . . invigorating
(C) reappraises . . dissolving
(D) supersedes . . negating
(E) reaffirms . . fashioning

ANSWERS AND EXPLA NATIONS

Explanation for Correct Answer E :

Choice (E) is correct. To reaffirm is to validate or confirm something again. Fashioning"


refers to the act of molding, shaping, or making something. The structure of the
sentence indicates that the part of the sentence after the semicolon elaborates on or
restates the information presented in the first part of the sentence. The first part of the
sentence essentially states that Inuit sculptors maintain carving traditionsthey carve
the same way their ancestors carvedwhile addressing currentnewsubject
matter or themes in their work. In other words, Inuit sculptors reaffirm a practice by u

From SAT Online Course CUUS


2007 May Sunday SAT Answers and Explanations

sing traditional carving techniques and also fashion, or shape, a modern cultural
identity by addressing new subjects.

Explanation for Incorrect Answer A :

Choice (A) is incorrect. To invalidate something is to nullify it or destroy its validity.


Manifesting" means showing or displaying. The structure of the sentence indicates that
the part of the sentence after the semicolon elaborates on or restates the information
presented in the first part of the sentence. The first part of the sentence essentially
states that Inuit sculptors maintain carving traditions they carve the same way their
ancestors carvedwhile addressing currentnewsubject matter or themes in
their work. It might make sense to suggest that by addressing new subjects, Inuit
sculptors are manifesting, or displaying, a new cultural identity. But these sculptors also
maintain, or uphold, traditional Inuit carving techniques, so it does not make sense to
suggest that they are invalidating, or nullifying, the practice.

Explanation for Incorrect Answer B :

Choice (B) is incorrect. To disregard something is to ignore it. Invigorating" means


giving energy or life to something. The structure of the sentence indicates that the part
of the sentence after the semicolon elaborates on or restates the information presented
in the first part of the sentence. The first part of the sentence essentially states that
Inuit sculptors maintain carving traditions they carve the same way their ancestors
carvedwhile addressing currentnewsubject matter or themes in their work. It
might make sense to suggest that by addressing new subjects, Inuit sculptors are
invigorating, or giving life to, their new cultural identity. But these sculptors also
maintain, or uphold, traditional Inuit carving techniques, so it does not make sense to
suggest that they are disregarding, or ignoring, the practice.

Explanation for Incorrect Answer C :

Choice (C) is incorrect. To reappraise is to evaluate the worth of something again. In


this context, dissolving" means undoing or making disappear. The structure of the
sentence indicates that the part of the sentence after the semicolon elaborates on or
restates the information presented in the first part of the sentence. The first part of the
sentence essentially states that Inuit sculptors maintain carving traditionsthey carve
the same way their ancestors carvedwhile addressing currentnewsubject
matter or themes in their work. It might make sense to say that as Inuit sculptors
maintain, or uphold, traditional Inuit carving techniques they also reappraise those
techniques. But it does not make sense to suggest that by addressing new subjects,
these sculptors are dissolving a modern cultural identity; on the contrary, the sculptors
seem to be fashioning, or shaping, a modern identity.

Explanation for Incorrect Answer D :

From SAT Online Course CUUS


2007 May Sunday SAT Answers and Explanations

Choice (D) is incorrect. To supersede is to replace something. Negating" means


denying the truth or existence of something. The structure of the sentence indicates
that the part of the sentence after the semicolon elaborates on or restates the
information presented in the first part of the sentence. The first part of the sentence
essentially states that Inuit sculptors maintain carving traditions they carve the same
way their ancestors carvedwhile addressing currentnewsubject matter or
themes in their work. Inuit sculptors are maintaining, or upholding, traditional carving
techniques, so it does not make sense to suggest that they are superseding, or
replacing, a traditional Inuit practice. Further, it does not make sense to suggest that
by addressing new subjects, these sculptors are negating, or denying, a modern
cultural identity; on the contrary, the sculptors seem to be fashioning, or shaping, a
modern identity.

4 Although many novelists write for artistic reasons, others are lured by the money,
because popular success in writing can be so ------- .

(A) acquisitive
(B) aesthetic
(C) diverting
(D) lucrative
(E) fulfilling

ANSWERS AND EXPLA NATIONS

Explanation for Correct Answer D :

Choice (D) is correct. The sentence suggests that popular success in writing can
generate moneythat, in fact, some novelists write because they are lured by the
money. If some writers make money, it makes sense to say that popular success in
writing can be lucrative, or profitable.

Explanation for Incorrect Answer A :

Choice (A) is incorrect. To be acquisitive is to value money and possessions


excessively. The sentence suggests that popular success in writing can generate
moneythat, in fact, some novelists write because they are lured by the money.
Therefore, the missing term must have something to do with making money. A writer w
ho wants to make money might be described as acquisitive, but it does not make sense
to say that the success a writer can achieve values or desires anything.

Explanation for Incorrect Answer B :

Choice (B) is incorrect. "Aesthetic" means relating to notions of beauty. The sentence
suggests that popular success in writing can generate moneythat, in fact, some nov

From SAT Online Course CUUS


2007 May Sunday SAT Answers and Explanations

elists write because they are lured by the money. Therefore, the missing term must
have something to do with making money. The term aesthetic is not related to
making money; additionally, although a writer might write for aesthetic reasons, it does
not make sense to say that the success a writer can achieve is related to notions of
beauty.

Explanation for Incorrect Answer C :

Choice (C) is incorrect. In this context, "diverting" means fun or amusing. The sentence
suggests that popular success in writing can generate moneythat, in fact, some
novelists write because they are lured by the money. Therefore, the missing term
must have something to do with making money. Although a writer might find popular
success diverting, or fun, the term diverting is not related to making money.

Explanation for Incorrect Answer E :

Choice (E) is incorrect. "Fulfilling" means satisfying. The sentence suggests that
popular success in writing can generate moneythat, in fact, some novelists write
because they are lured by the money. Therefore, the missing term must have
something to do with making money. A writer might find popular successand the
money that comes with itsatisfying, but the term fulfilling is not necessarily related
to making money; something could be fulfilling without being profitable. Indeed, the
term fulfilling is usually used to describe activities or experiences undertaken for non-
monetary reasons, such as volunteer work, family life, and creative expression.

5 The company manager was known for both his ------- and his ------- : he lied
frequently, but did so with amazing fla ir.

(A) ambivalence . . extravagance


(B) duplicity . . panache
(C) evasiveness . . irascibility
(D) mendacity . . corruption
(E) brashness . . charisma

ANSWERS AND EXPLA NATIONS

Explanation for Correct Answer B :

Choice (B) is correct. The structure of the sentence indicates that the missing terms
describe someone who lies frequently and has amazing flair. To be duplicitous is to be
two-faced or dishonest, and panache is flamboyance or flair; someone who lies
frequently with amazing flair would likely be known for his duplicity and his panache.

Explanation for Incorrect Answer A :

From SAT Online Course CUUS


2007 May Sunday SAT Answers and Explanations

Choice (A) is incorrect. The structure of the sentence indicates that the missing terms
describe someone who lies frequently and has amazing flair. The terms ambivalence
and extravagance do not necessarily describe such a person; ambivalence, or
simultaneous and contradictory feelings about something, and extravagance, or
excessive luxury, have nothing to do with lying with flair.

Explanation for Incorrect Answer C :

Choice (C) is incorrect. The structure of the sentence indicates that the missing terms
describe someone who lies frequently and has amazing flair. The terms evasiveness
and irascibility do not necessarily describe such a person. Someone who is evasive, or
avoids addressing something, might lie, but there is no logical connection between
being irascible, or having a quick temper, and lying with flair.

Explanation for Incorrect Answer D :

Choice (D) is incorrect. The structure of the sentence indicates that the missing terms
describe someone who lies frequently and has amazing flair. A manager who lies
frequently could certainly be known for his mendacity, or dishonesty, and his
corruption, or involvement in dishonest business dealings. But (D) is not the best
choice because neither mendacity nor corruption describes someone who has
amazing flair.

Explanation for Incorrect Answer E :

Choice (E) is incorrect. The structure of the sentence indicates that the missing terms
describe someone who lies frequently and has amazing flair. The terms brashness
and charisma do not necessarily describe such a person; brashness, or aggressively
self-assertive behavior, and charisma, or special charm or appeal, have nothing to do
with lying with flair.

Passage 1

The bias against lif e on Venus is rooted in human


presumpt ion. From our limited observation of lif e on
Earth we inf er that liquid water, pref erably lots of it,
Line is essential for lif e everywhere. So, when searching
5 for extraterrestrial lif e, we obsess over "riv ulets" on
Mars' faceapparently carved by ancient gushes of
waterand delight in hints of permaf rost just under-
neath the planet's surf ace. (By comparison, Venus
isn't even that interesting to look at: A boring cue ball
10 for backyard astronomers.) Attention and then funding

From SAT Online Course CUUS


2007 May Sunday SAT Answers and Explanations

follow the water: More space landers will depart


for Mars, and serious plans for further missions hover in
the future.

Passage 2

Is there lif e on Venus? Could there be lif e on Venus?


15 The standard answers are "No" and "NO!" Venus is usu-
ally dismissed in a paragraph or two before an extensive
discussion of the prospects for lif e on Mars and other
planets. This thinking presupposes knowledge of the
universal nature of lif e and the general characteristics
20 of inhabited planetsknowledge that we do not yet
possess. Lif e is usually assumed to require organic
molecules dissolved in liquid water. Discussions of
other habitable places in the universe exclusively
focus on planets with climates and atmospheric
25 conditions that are "just right" for us. Venus, with
its 850-900 degree Fahrenheit surf ace temperatures,
is obviously sterile. Or is it?

6 The author of Passage 2 suggests that the bias mentioned in line 1, Passage 1,
arises because people

(A) find Venus less interesting to look at than other planets


(B) often ignore crucial information provided by experts
(C) assume that they know what they do not actually know
(D) are unaware of the atmospheric conditions on Venus
(E) believe that Earth has a unique status in the universe

ANSWERS AND EXPLA NATIONS

Explanation for Correct Answer C :

Choice (C) is correct. The author of Passage 1 refers to the bias against life on
Venuspeoples reluctance to consider that there may be life on Venus. The author of
Passage 2 also refers to this bias, noting that the standard [answer] is that there is
no life on Venus and that there cannot be life on Venus. He or she states that this
biased thinking presupposes knowledge of the universal nature of life and the general
characteristics of inhabited planets knowledge that we do not yet possess." In other
words, it stems from peoples false assumption that they know what is necessary for
life to exist and what is required for a planet to be inhabitable. The author of Passage 2
suggests that This thinking arises because people assume that they know what they
do not actually know.

From SAT Online Course CUUS


2007 May Sunday SAT Answers and Explanations

Explanation for Incorrect Answer A :

Choice (A) is incorrect. The author of Passage 1 refers to the bias against life on
Venuspeoples reluctance to consider that there may be life on Venus. The author of
Passage 2 also refers to this bias, noting that the standard [answer] is that ther e is
no life on Venus and that there cannot be life on Venus. Although the author of Passage
1 refers to Venus as a boring cue ball for backyard astronomers, the author of
Passage 2 does not discuss the idea that people find Venus less interesting to look at
than other planets. Rather, the author suggests that the bias against life on Venus
stems from peoples assumption that they know what is necessary for life to exist and
what is required for a planet to be inhabitable people think they have knowledge that
they actually do not yet possess.

Explanation for Incorrect Answer B :

Choice (B) is incorrect. The author of Passage 1 refers to the bias against life on
Venuspeoples reluctance to consider that there may be life on Venus. The author of
Passage 2 also refers to this bias, noting that the standard [answer] is that ther e is
no life on Venus and that there cannot be life on Venus. Rather than suggesting that
the bias arises because people often ignore crucial information provided by experts, the
author of Passage 2 suggests that it arises because some people assume that they
know what they do not actually know. The author states that this biased thinking
presupposes knowledge of the universal nature of life and the general characteristics
of inhabited planetsknowledge of what is necessary for life to exist and what is
required for a planet to be inhabitableand goes on to indicate that this is knowledge
that we do not yet possess.

Explanation for Incorrect Answer D :

Choice (D) is incorrect. The author of Passage 1 refers to the bias against life on
Venuspeoples reluctance to consider that there may be life on Venus. The author of
Passage 2 also refers to this bias, noting that the standard [answer] is that ther e is
no life on Venus and that there cannot be life on Venus. He or she states that this
biased thinking presupposes knowledge of the universal nature of life and the general
characteristics of inhabited planets in other words, it stems from peoples assumption
that they know what is necessary for life to exist and what is required for a planet to be
inhabitable. Rather than suggesting that the bias against life on Venus arises because
people are unaware of Venus atmospheric conditions, the author suggests that the bias
arises in part because people are aware of the atmospheric conditions: people think
that Venus is obviously sterile because of its 850-900 degree Fahrenheit surface
temperatures.

Explanation for Incorrect Answer E :

Choice (E) is incorrect. The author of Passage 1 refers to the bias against life on Venus

From SAT Online Course CUUS


2007 May Sunday SAT Answers and Explanations

peoples reluctance to consider that there may be life on Venus. The author of
Passage 2 also refers to this bias, noting that the standard [answer] is that there is
no life on Venus and that there cannot be life on Venus. The author does imply that
Earths climates and atmospheric conditions are just right for us, but he or she
does not suggest that the bias against life on Venus arises because people believe that
Earth has a unique status in the universe. Indeed, the author claims that there may be
planets with similar conditions to Earth (Discussions of other habitable places in the
universe); this suggests that Earth may not be unique.

7 On which point do the authors agree?

(A) Earth is not the only planet in the universe where life exists.
(B) Life is more likely to be discovered on Venus than on Mars.
(C) People consider the possibility of life on Venus from too narrow a perspective.
(D) Humans have a natural inclination to explore other planets .
(E) Earth-based observation of Venus has given astronomers a false impression.

ANSWERS AND EXPLA NATIONS

Explanation for Correct Answer C :

Choice (C) is correct. The two authors agree that people consider the possibility of life
on Venus from too narrow a perspective. The author of Passage 1 indicates that our
limited observation of life on Earth has caused us to infer that liquid water . . . is
essential for life everywhere; as a result, we obsess over rivulets on Mars face and
have a bias against life on Venus. The author suggests that people make judgments
about the possibility of life on Venus based on what they know of life on
Earthknowledge that is limited in scope. The author of Passage 2 also addresses this
idea, stating that the thinking that there is no life on Venus presupposes knowledge of
the universal nature of life and the general characteristics of inhabited planetsin
other words, it stems from peoples assumption that they know what is required for life
to exist and what makes a planet inhabitable. The author suggests that this view
results in a narrow perspective: when considering other habitable places in the
universe, people exclusively focus on planets with climates and atmospheric
conditions that are just right for us and disregard Venus because they think it is
obviously sterile, due to its 850-900 degree Fahrenheit surface temperatures.

Explanation for Incorrect Answer A :

Choice (A) is incorrect. Both authors suggest that there might be life on other planets in
the universe, but neither passage contains any reference to proof that there is life on
any other planet in the universe. Rather, the authors are primarily concerned with
people's reluctance to even consider the possibility of life on Venus.

From SAT Online Course CUUS


2007 May Sunday SAT Answers and Explanations

Explanation for Incorrect Answer B :

Choice (B) is incorrect. Both authors imply that many people expect to find life on Mars
and not on Venus (bias against life on Venus; we obsess over rivulets on Mars face;
Venus is usually dismissed in a paragraph or two before an extensive discussion of the
prospects for life on Mars and other planets). While cautioning that these people may
be wrong, and that the possibility of life on Venus has been dismissed too quickly,
neither author goes as far as to say that life is more likely to be discovered on Venus
than on Mars.

Explanation for Incorrect Answer D :

Choice (D) is incorrect. The author of Passage 1 does acknowledge that people are
exploring other planets, noting that as indications of water on Mars are discovered,
More space landers will depart for Marsimplying that space landers have already
departed for Mars. But he or she does not address the idea that humans have a natural
inclination to explore other planets. The author of Passage 2 does refer to Venus 850-
900 degree Fahrenheit surface temperaturesinformation that likely came from some
sort of exploration of Venusbut he or she does not directly address the exploration of
other planets.

Explanation for Incorrect Answer E :

Choice (E) is incorrect. The author of Passage 1 does refer to observations made on
Earth and to astronomers; however, he or she refers to observations of life on Earth,
not to observations of Venus, and does not indicate that astronomerseven backyard
astronomers, or non-specialists who pursue astronomy only as a hobbyhave a false
impression of Venus. The author of Passage 2 suggests that people dismiss the idea of
life on Venus too quickly, but he or she does not suggest that this dismissal is the result
of false impressions gained from Earth-based observation of Venus; the author refers to
Discussions of other habitable places in the universe, but not to direct observation of
any other planet.

8 Unlike Passage 2, Passage 1 makes reference to

(A) receptivity to the idea of life on Mars


(B) the prospect of future planetary explorations
(C) the distance between Earth and Venus
(D) Venus inhospitable climate
(E) the abundance of habitable places in the universe

ANSWERS AND EXPLA NATIONS

Explanation for Correct Answer B :

From SAT Online Course CUUS


2007 May Sunday SAT Answers and Explanations

Choice (B) is correct. Passage 1 makes reference to the prospect of future planetary
explorations. The author says that as indications of water are discovered on Mars,
Attention and then funding follow the water: More space landers will depart for Mars,
and serious plans for further missions hover in the future. The author of Passage 2,
however, does not refer to the prospect of future planetary explorations. He or she only
addresses Discussions of . . . habitable places in the universe and perceptions of the
universal nature of life and the general characteristics of inhabitable planets.

Explanation for Incorrect Answer A :

Choice (A) is incorrect. Both passages discuss receptivity to the idea of life on Mars.
The author of Passage 1 indicates that people infer that . . . water . . . is essential for
life, and suggests that, as a result, they obsess over rivulets on Mars face . . . and
delight in hints of permafrost just underneath the planets surface because these are
indications that life could exist on Mars. The author of Passage 2 indicates that people
are receptive to the idea of life on Mars, stating that Venus is usually dismissed in a
paragraph or two before an extensive discussion of the prospects for life on Mars and
other planets.

Explanation for Incorrect Answer C :

Choice (C) is incorrect. Neither passage contains any reference to the distance between
Earth and Venus; both passages simply discuss ideas related to the topic of finding life
on Venus.

Explanation for Incorrect Answer D :

Choice (D) is incorrect. Passage 2 does make reference to Venus inhospitable climate;
the author suggests that many assume Venus is sterile because of its inhospitable
850-900 degree Fahrenheit surface temperatures. It is Passage 1 that does not make
reference to Venus climate; the author only notes that Venus isnt . . . that interesting
to look at.

Explanation for Incorrect Answer E :

Choice (E) is incorrect. Neither passage makes reference to the abundance, or ample
quantity, of habitable places in the universe. The author of Passage 1 suggests that
people think there might be life on Mars because indications of water have been found,
but he or she does not state that there are, in fact, any other habitable places in the
universe. The author of Passage 2 suggests that there might be some habitable places
in the universeother planets with climates and atmospheric conditions that are just
right for usbut does not indicate that there is an abundance of such places.

9 Which of the following is implied in both passages but explicitly stated in neither?

From SAT Online Course CUUS


2007 May Sunday SAT Answers and Explanations

(A) Venus surface temperature is higher than that of Earth.


(B) Scientists generally dismiss the possibility of extraterrestrial life.
(C) There can be no life without organic molecules.
(D) Mars rivulets are not what they appear to be.
(E) There is no evidence of liquid water on Venus.

ANSWERS AND EXPLA NATIONS

Explanation for Correct Answer E :

Choice (E) is correct. Both passages implysuggest without stating directlythat there
is no evidence of liquid water on Venus. The author of Passage 1 explains that the bias
against life on Venus stems from the idea, based on observation of life on Earth, that
liquid water . . . is essential for life. This explanation suggests that there is no
evidence of liquid water on Venusif water had been discovered on Venus, people
would not disregard the possibility of finding life there. The author of Passage 2 states
that Life is usually assumed to require organic molecules dissolved in liquid water,
and that Venus is usually dismissed in a paragraph or two before an extensive
discussion of the prospects for life on Mars and other planets. The authors discussion
suggests that evidence of liquid watera prerequisite for lifehas not been found on
Venus.

Explanation for Incorrect Answer A :

Choice (A) is incorrect. The author of Passage 2 does suggest that Venus surface
temperature is higher than that of Earth: unlike planets with climates and atmospheric
conditions that are just right for usplanets with conditions like those on
EarthVenus has 850-900 degree Fahrenheit surface temperatures and may be
sterile. But the author of Passage 1 does not imply or explicitly state that Venus
surface temperature is higher than that of Earth; the author does not address Venus
surface temperature at all.

Explanation for Incorrect Answer B :

Choice (B) is incorrect. Neither author implies or explicitly states that scientists
generally dismiss the possibility of extraterrestrial life. The author of Passage 1 may be
speaking for himself or herself, as well as scientists, when he or she states that when
searching for extraterrestrial life, we obsess over rivulets on Mars face . . . . The
author of Passage 1 also refers to serious plans for . . . missions to Mars, suggesting
that scientists are engaged in the search for extraterrestrial life. The author of Passage
2 also may be referring to scientists when he or she mentions extensive discussion of
the prospects for life on Mars and other planets and Discussions of other habitable
places in the universe. There is no indication that scientists dismiss the possibility of
life on other planets.

From SAT Online Course CUUS


2007 May Sunday SAT Answers and Explanations

Explanation for Incorrect Answer C :

Choice (C) is incorrect. The author of Passage 2 explicitly states that Life is usually
assumed to require organic molecules dissolved in liquid waterin other words, there
can be no life without organic molecules. But the author of Passage 1 neither implies
nor explicitly states that life cannot exist without organic molecules; he or she only
states that people infer that liquid water . . . is essential for lifewater is not an
organic molecule.

Explanation for Incorrect Answer D :

Choice (D) is incorrect. The author of Passage 1 states that Mars rivulets were
apparently carved by ancient gushes of water." If this could be proved, it would
provide evidence that Mars once had water. While there is no indication in the passage
that any proof exists, the author does not imply that these rivulets are inauthentic or
that they are authentic. The author's point is simply that, "when searching for
extraterrestrial life," observers focus on questions of water. The author of Passage 2
does not address Mars rivulets at all.

This passage is taken from a novel set in the 1950s at the Cedar Grove Elementary
School, where Miss Dove has taught geography for more than a generation.

Miss Dove's rules were as fixed as the signs of the


zodiac. And they were known. Miss Dove rehearsed
them at the beginning of each school year, stating them
Line as calmly and dispassionately as if she were describing
5 the atmospheric effects of the Gulf Stream. The penalties
for inf ractions of the rules were also known. If a student's
posture was incorrect, he had to go and sit for a while upon
a stool without a backrest. If a page in his notebook was
untidy, he had to copy it over. If he emitted an uncovered
10 cough, he was expected to rise immediately and fling open
a window, no matter how cold the weather, so that a blast
of fresh air could protect his fellows from the contamina-
tion of his germs. And if he felt obliged to disturb the
class routine by leaving the room for a drink of water
15 (Miss Dove loftily ignored any other necessity), he did
so to an accompaniment of dead silence. Miss Dove would
look at him that was allfollowing his departure and
greeting his return with her perf ectly expressionless gaze,
and the whole class would sit idle and motionless until
20 he was back in the fold again. It was easiereven if
one had eaten salt fish for breakf astto remain and

From SAT Online Course CUUS


2007 May Sunday SAT Answers and Explanations

suffer.
Of course, there were flagrant offenses that were
dealt with in private. Sometimes prof anity sullied the
25 air of the geography room. Sometimes, though rarely,
open rebellion was displayed. In those instances, the
delinquent was detained, minus the comf ort of his com-
rades, in awful seclusion with Miss Dove. What happened
between them was never fully known. (Did she threaten
30 him with legal prosecution? Did she terrorize him with
her long map-pointer?) The culprit, himself, was unlikely
to be communicative on the subject or, if he were, would
tend to overdo the business with a tale that revolved to
an incredible degree around his own heroism. After-
35 wards, as was duly noted, his classroom attitude was
subdued and chastened.
Miss Dove had no rule relating to prevarication.
A child's word was taken at face value. If it happened
to be falsewell, that was the child's problem. A lie,
40 unattacked and undistorted by defense, remained a lie
and was apt to be recognized as such by its author.
Occasionally a group of progressive mothers would
contemplate organized revolt. "She's been teaching too
long," they would cry. "Her pedagogy hasn't changed
45 since we were in Cedar Grove. She rules the children
through fear!" They would turn to the boldest one
among themselves. "You go," they would say. "You
go talk to her!"
The bold one would go, but somehow she never did
50 much talking. For there in the geography room, she would
being to feelthough she wore her handsomest tweeds
and perhaps a gardenia for couragethat she was about
ten years old and her petticoat was showing. Her throat
would tickle. She would wonder desperately if she had a
55 clean handkerchief in her bag. She would also feel thirsty.
Without firing a shot in the cause of freedom she would
retreat ingloriously from the field of battle.
And on that unassaulted fieldin that room where no
leeway was given to the personality, where a thing was
60 black or white, right or wrong, polite or rude, simply
because Miss Dove said it was, there was a curiously
soothing quality. The children left it refreshed and
restored, ready for fray or frolic. For within its walls
they enjoyed what was allowed them nowhere else
65 a complete suspension of will.

From SAT Online Course CUUS


2007 May Sunday SAT Answers and Explanations

10 The narrator refers to the signs of the zodiac (lines 1-2) as an example of things
that are

(A) mysterious
(B) countless
(C) unchanging
(D) ominous
(E) unattainable

ANSWERS AND EXPLA NATIONS

Explanation for Correct Answer C :

Choice (C) is correct. In the first sentence of the passage, the narrator describes Miss
Doves rules as being as fixed as the signs of the zodiac. In this context, the term
fixed means firmly set and not subject to change. The narrator refers to the signs of
the zodiac 12 constellations in the sky that are attributed symbolic significance by
astrologersas an example of things that are fixed or unchanging.

Explanation for Incorrect Answer A :

Choice (A) is incorrect. In the first sentence of the passage, the narrator describes Miss
Doves rules as being as fixed as the signs of the zodiac. In this context, the term
fixed means firmly set and not subject to change. The narrator refers to the signs of
the zodiac 12 constellations in the sky that are attributed symbolic significance by
astrologersas an example of things that are fixed or unchanging. He or she does not
suggest that the zodiac signs are an example of things that are mysterious, or beyond
understanding; the zodiac signs, like Miss Doves rules, seem to be known.

Explanation for Incorrect Answer B :

Choice (B) is incorrect. In the first sentence of the passage, the narrator describes Miss
Doves rules as being as fixed as the signs of the zodiac. In this context, the term
fixed means firmly set and not subject to change. The narrator refers to the signs of
the zodiac 12 constellations in the sky that are attributed symbolic significance by
astrologersas an example of things that are fixed or unchanging. He or she does not
suggest that the zodiac signs are an example of things that are countless, or too
numerous to be counted; there are just 12 zodiac signs.

Explanation for Incorrect Answer D :

Choice (D) is incorrect. In the first sentence of the passage, the narrator describes Miss
Doves rules as being as fixed as the signs of the zodiac. In this context, the term
fixed means firmly set and not subject to change. The narrator refers to the signs of t

From SAT Online Course CUUS


2007 May Sunday SAT Answers and Explanations

he zodiac 12 constellations in the sky that are attributed symbolic significance by


astrologersas an example of things that are fixed or unchanging. There is no
indication that the narrator finds the signs of the zodiac to be ominoussuggesting or
foreshadowing some evilso there is no reason to believe that he or she refers to the
zodiac signs as an example of things that are ominous.

Explanation for Incorrect Answer E :

Choice (E) is incorrect. In the first sentence of the passage, the narrator describes Miss
Doves rules as being as fixed as the signs of the zodiac. In this context, the term
fixed means firmly set and not subject to change. The narrator refers to the signs of
the zodiac 12 constellations in the sky that are attributed symbolic significance by
astrologersas an example of things that are fixed or unchanging. There is no
indication that the narrator finds the signs of the zodiac to be unattainable, or
impossible to attain or achieve, so there is no reason to believe that he or she refers to
the zodiac signs as an example of things that are unattainable. In fact, it is somewhat
illogical to even describe the zodiac signs as unattainable.

11 In line 2, rehearsed most nearly means

(A) directed
(B) listed
(C) practiced
(D) perfected
(E) demonstrated

ANSWERS AND EXPLA NATIONS

Explanation for Correct Answer B :

Choice (B) is correct. In the first paragraph, the narrator describes Miss Doves rules as
being fixed, or unchanging. He or she states that Miss Dove rehearsed them at the
beginning of each school year, stating them . . . calmly and dispassionately. In this
context, the term rehearsed means recounted in order, or listed. The narrators
description of Miss Dove rehearsing her rules calmly and dispassionately suggests that
each year, Miss Dovewith no emotionsimply listed for her students the rules of her
classroom.

Explanation for Incorrect Answer A :

Choice (A) is incorrect. In the first paragraph, the narrator describes Miss Doves rules
as being fixed, or unchanging. He or she states that Miss Dove rehearsed them at
the beginning of each school year, stating them . . . calmly and dispassionately. A
director of a play might run a rehearsal, but as it is used in line 2, the term rehearsed

From SAT Online Course CUUS


2007 May Sunday SAT Answers and Explanations

does not mean directed. Rather, the term is used to mean listed, or recounted in order;
the narrator suggests that each year, Miss Dove listed for her students the rules of her
classroom.

Explanation for Incorrect Answer C :

Choice (C) is incorrect. In the first paragraph, the narrator describes Miss Doves rules
as being fixed, or unchanging. He or she states that Miss Dove rehearsed them at
the beginning of each school year, stating them . . . calmly and dispassionately. In
some contexts, the term rehearsed can mean practiced, but the term is not used in
this way in line 2. One might practice something in order to memorize it, but the
narrator states that Miss Doves rules were known. Additionally, since Miss Doves
rules did not change from year to year, Miss Dove would likely be so familiar with them
that she would not need to practice reciting them. Rather, the narrator suggests that
each year, Miss Dove listed for her students the rules of her classroomshe recounted
the rules in order.

Explanation for Incorrect Answer D :

Choice (D) is incorrect. In the first paragraph, the narrator describes Miss Doves rules
as being fixed, or unchanging. He or she states that Miss Dove rehearsed them at
the beginning of each school year, stating them . . . calmly and dispassionately.
Someone might rehearse something in order to perfect it, but the narrator does not
suggest that Miss Dove perfected, or improved and refined, her rules each year.
Perfecting the rules would involve changing them, and the narrator tells the reader that
Miss Doves rules were unchanging. Rather, the narrator suggests that each year, Miss
Dove listed for her students the rules of her classroomshe recounted the rules in
order.

Explanation for Incorrect Answer E :

Choice (E) is incorrect. In the first paragraph, the narrator describes Miss Doves rules
as being fixed, or unchanging. He or she states that Miss Dove rehearsed them at
the beginning of each school year, stating them . . . calmly and dispassionately. The
narrator does not suggest that Miss Dove demonstrated, or showed by example, her
rules; there is no indication that Miss Dove acted out any rules. Rather, the narrator
suggests that each year, Miss Dove listed for her students the rules of her
classroomshe recounted the rules in order.

12 The series of statements in lines 6-16 (If a students . . . silence) is best


described as a

(A) series of anecdotes about memorable events


(B) collection of advice regarding student conduct

From SAT Online Course CUUS


2007 May Sunday SAT Answers and Explanations

(C) string of accusations and excuses


(D) list of punishments for various infractions
(E) rationalizatio n for some unfair penalties

ANSWERS AND EXPLA NATIONS

Explanation for Correct Answer D :

Choice (D) is correct. In lines 5-6, the narrator states that the penalties for infractions
of [Miss Doves] rules were . . . known. He or she then goes on to list some of the
punishments for violating the rules. For example, a student with incorrect posture had
to sit . . . upon a stool without a backrest, a student with a messy page in his
notebook had to copy it over, and a student who emitted an uncovered cough had
to rise immediately and fling open a window, no matter how cold the weather.

Explanation for Incorrect Answer A :

Choice (A) is incorrect. In lines 5-6, the narrator states that the penalties for
infractions of [Miss Doves] rules were known. He or she then goes on to list some of
the punishments for violating the rules: a student might be made to
sit . . . upon a stool without a backrest, to copy . . . over a messy notebook page,
or to rise immediately and fling open a window, no matter how cold the weather, after
emitt[ing] an uncovered cough. The narrator may consider these events to be
memorable, but the statements in lines 6-16 are not a series of anecdotes; an
anecdote is a short narrative of an interesting, amusing, or biographical incident.
Rather than relating a series of narratives, the author simply lists the punishments for
various infractions.

Explanation for Incorrect Answer B :

Choice (B) is incorrect. In lines 5-6, the narrator states that the penalties for
infractions of [Miss Doves] rules were known. He or she then goes on to list some of
the punishments for violating the rules: a student might be made to
sit . . . upon a stool without a backrest, to copy . . . over a messy notebook page,
or to rise immediately and fling open a window, no matter how cold the weather, after
emitt[ing] an uncovered cough. Rather than giving advice regarding how students
should behave, the author lists the punishments students would receive when Miss
Dove found their behavior unacceptable.

Explanation for Incorrect Answer C :

Choice (C) is incorrect. In lines 5-6, the narrator states that the penalties for
infractions of [Miss Doves] rules were known. He or she then goes on to list some of
the punishments for violating the rules: a student might be made to
sit . . . upon a stool without a backrest, to copy . . . over a messy notebook page,
or to rise immediately and fling open a window, no matter how cold the weather, after

From SAT Online Course CUUS


2007 May Sunday SAT Answers and Explanations

emitt[ing] an uncovered cough. Miss Dove may have accused the students of
violating certain rulesby having messy notebooks or not sitting up straightbut the
narrators statements in lines 6-16 are not a string of accusations and explanations.
The narrator neither accuses anyone of anything nor makes excuses for anyone. Rather,
he or she simply lists the punishments students would receive when they broke Miss
Doves rules.

Explanation for Incorrect Answer E :

Choice (E) is incorrect. In lines 5-6, the narrator states that the penalties for
infractions of [Miss Doves] rules were known. He or she then goes on to list some of
the punishments for violating the rules: a student might be made to
sit . . . upon a stool without a backrest, to copy . . . over a messy notebook page,
or to rise immediately and fling open a window, no matter how cold the weather, after
emitt[ing] an uncovered cough. The narrator is recalling and describing the penalties,
but there is no indication that he or she feels the penalties were unfair, or that it was
necessary to rationalize, or justify, the penalties. Rather, he or she simply lists the
punishments students would receive when they broke Miss Doves rules.

13 In line 17, the narrator inserts the phrase that was all in order to emphasize that

(A) the infraction was not a serious one


(B) a spoken reproach was unnecessary
(C) the student openly acknowledged wrongdoing
(D) only one offense had been committed
(E) even a single offense brought punishment

ANSWERS AND EXPLA NATIONS

Explanation for Correct Answer B :

Choice (B) is correct. In the first paragraph of the passage, the narrator explains that
Miss Dove strictly enforced the rules of her classroom and then gives examples of the
penalties for infractions of the rules. The narrator recalls that when a student would
disturb the class routine by leaving the room for a drink of water, the student would
leave to an accompaniment of dead silence; Miss Dove would look at himthat was
all. The phrase that was all indicates that a spoken reproach, or expression of
disapproval, was unnecessary; all Miss Dove had to do was look at a student with her
perfectly expressionless gaze , and the student would know that Miss Dove was
displeased with his or her behavior.

Explanation for Incorrect Answer A :

Choice (A) is incorrect. In the first paragraph of the passage, the narrator explains that

From SAT Online Course CUUS


2007 May Sunday SAT Answers and Explanations

Miss Dove strictly enforced the rules of her classroom and then gives examples of the
penalties for infractions of the rules. The narrator recalls that when a student would
disturb the class routine by leaving the room for a drink of water, the student would
leave to an accompaniment of dead silence; Miss Dove would look at himthat was
all. If the phrase that was all was part of the statement about disturbing the class
routine, the narrator could be using it to indicate that the infraction was not a serious
one. But the phrase is used in connection with Miss Doves behavior, not with the
behavior of the student who violated a rule. The phrase indicates that Miss Doves
perfectly expressionless gaze was so powerful that a spoken reproach, or expression
of disapproval, was unnecessary when a student disturbed the class.

Explanation for Incorrect Answer C :

Choice (C) is incorrect. In the first paragraph of the passage, the narrator explains that
Miss Dove strictly enforced the rules of her classroom and then gives examples of the
penalties for infractions of the rules. The narrator recalls that when a student would
disturb the class routine by leaving the room for a drink of water, the student would
leave to an accompaniment of dead silence; Miss Dove would look at himthat was
all. The phrase that was all is used in connection with Miss Doves behavior, not with
the behavior of the student who violated a rule; the phrase does not indicate that the
student openly acknowledged wrongdoing. Rather, the phrase indicates that Miss
Doves perfectly expressionless gaze was so powerful that a spoken reproach, or
expression of disapproval, was unnecessary when a student disturbed the class.

Explanation for Incorrect Answer D :

Choice (D) is incorrect. In the first paragraph of the passage, the narrator explains that
Miss Dove strictly enforced the rules of her classroom and then gives examples of the
penalties for infractions of the rules. The narrator recalls that when a student would
disturb the class routine by leaving the room for a drink of water, the student would
leave to an accompaniment of dead silence; Miss Dove would look at himthat was
all. If the phrase that was all was part of the statement about disturbing the class
routine, the narrator could be using it to indicate that only one offense had been
committed. But the phrase is used in connection with Miss Doves behavior, not with the
behavior of the student who violated a rule. The phrase indicates that Miss Doves
perfectly expressionless gaze was so powerful that a spoken reproach, or expression
of disapproval, was unnecessary when a student disturbed the class.

Explanation for Incorrect Answer E :

Choice (E) is incorrect. In the first paragraph of the passage, the narrator explains that
Miss Dove strictly enforced the rules of her classroom and then gives examples of the
penalties for infractions of the rules. The narrator recalls that when a student would
disturb the class routine by leaving the room for a drink of water, the student would
leave to an accompaniment of dead silence; Miss Dove would look at himthat was

From SAT Online Course CUUS


2007 May Sunday SAT Answers and Explanations

all. If the phrase that was all was part of the statement about disturbing the class
routine, the narrator could be using it to emphasize that even a single offense brought
punishmentthat the student only did one thing wrong. But the phrase is used in
connection with Miss Doves behavior, not with the behavior of the student who violated
a rule. The phrase indicates that Miss Doves perfectly expressionless gaze was so
powerful that a spoken reproach, or expression of disapproval, was unnecessary when
a student disturbed the class.

14 In line 41, author refers to a

(A) child who told a lie


(B) child who reported a liar
(C) teacher who criticized a liar
(D) mother of Miss Doves students
(E) writer of fiction

ANSWERS AND EXPLA NATIONS

Explanation for Correct Answer A :

Choice (A) is correct. In lines 37-41, the narrator states that Miss Dove took a childs
word at face value, and that if a child was lying, "that was the child's problem." A lie
"was apt to be recognized as such by its authorin other words, Miss Dove knew that
a lie was likely to come back to hurt the person who told it. In line 41, the term
author refers to a child who told a lie; that child is the author, or creator, of the lie.

Explanation for Incorrect Answer B :

Choice (B) is incorrect. In lines 37-41, the narrator states that Miss Dove took a childs
word at face value, and that if a child was lying, "that was the child's problem." A lie
"was apt to be recognized as such by its authorin other words, Miss Dove knew that
a lie was likely to come back to hurt the person who told it. There is no discussion of a
child reporting a lia r. When the narrator uses the term author in line 41, he or she
uses it to refer to a child who told a lie; that child is the autho r, or creator, of the lie.

Explanation for Incorrect Answer C :

Choice (C) is incorrect. In lines 37-41, the narrator states that Miss Dove took a child s
word at face value, and that if a child was lying, "that was the child's problem." A lie
"was apt to be recognized as such by its authorin other words, Miss Dove knew that
a lie was likely to come back to hurt the person who told it. Rather than using the term
author to refer to a teacher who criticized a lia r, the narrator uses the term to refer to
a child who told a lie; that child is the author, or creator, of the lie.

From SAT Online Course CUUS


2007 May Sunday SAT Answers and Explanations

Explanation for Incorrect Answer D :

Choice (D) is incorrect. In lines 37-41, the narrator states that Miss Dove took a childs
word at face value, and that if a child was lying, "that was the child's problem." A lie
"was apt to be recognized as such by its authorin other words, Miss Dove knew that
a lie was likely to come back to hurt the person who told it. Later in the passage, the
narrator discusses a group of progressive mothers, but he or she does not mention
these mothers in the discussion of lying. Rather than using the term author to refer to
a mother of Miss Doves students, the narrator uses the term to refer to a child who
told a lie; that child is the author, or creator, of the lie.

Explanation for Incorrect Answer E :

Choice (E) is incorrect. In lines 37-41, the narrator states that Miss Dove took a child s
word at face value, and that if a child was lying, "that was the child's problem." A lie
"was apt to be recognized as such by its authorin other words, Miss Dove knew that
a lie was likely to come back to hurt the person who told it. A writer of fiction is an
author, but the term author does not only refer to someone who is a writer; the term
can also be used to describe someone who creates something. In line 41, the narrator
uses the term author to refer to a child who told a lie; that child is the author, or
creator, of the lie.

15 Lines 56-57 (Without . . . battle) gently mock the situation by describing it in


terms of a

(A) sporting competition


(B) political debate
(C) popularity contest
(D) courtroom trial
(E) military event

ANSWERS AND EXPLA NATIONS

Explanation for Correct Answer E :

Choice (E) is correct. In lines 42-57, the narrator describes a group of progressive
mothers who would occasionally challenge Miss Dove; these mothers felt she had
been teaching too long and that she rule[d] the children through fear! One
motherthe boldest one among [them]would enter Miss Doves geography room,
but, once there, she would lose her resolve, or determination. Ultimately, without
firing a shot in the cause of freedom she would retreat ingloriously from the field of
battle. The narrator gently mocks the situation by describing it in terms of a military
event, suggesting that Miss Doves classroom is a field of battle and that the mothers
complaints are shots fired in the cause of freedomthe students freedom from Miss Do

From SAT Online Course CUUS


2007 May Sunday SAT Answers and Explanations

ves strict rules.

Explanation for Incorrect Answer A :

Choice (A) is incorrect. In lines 42-57, the narrator describes a group of progressive
mothers who would occasionally challenge Miss Dove; these mothers felt she had
been teaching too long and that she rule[d] the children through fear! One
motherthe boldest one among [them]would enter Miss Doves geography room,
but, once there, she would lose her resolve, or determination. Ultimately, without
firing a shot in the cause of freedom she would retreat ingloriously from the field of
battle. The phrases firing a shot, retreat ingloriously, and field of battle all relate
to a military eventa battle not a sporting competition.

Explanation for Incorrect Answer B :

Choice (B) is incorrect. In lines 42-57, the narrator describes a group of progressive
mothers who would occasionally challenge Miss Dove; these mothers felt she had
been teaching too long and that she rule[d] the children through fear! One
motherthe boldest one among [them]would enter Miss Doves geography room,
but, once there, she would lose her resolve, or determination. Ultimately, without
firing a shot in the cause of freedom she would retreat ingloriously from the field of
battle. The phrases firing a shot, retreat ingloriously, and field of battle all relate
to a military eventa battle not a political debate.

Explanation for Incorrect Answer C :

Choice (C) is incorrect. In lines 42-57, the narrator describes a group of progressive
mothers who would occasionally challenge Miss Dove; these mothers felt she had
been teaching too long and that she rule[d] the children through fear! One
motherthe boldest one among [them]would enter Miss Doves geography room,
but, once there, she would lose her resolve, or determination. Ultimately, without
firing a shot in the cause of freedom she would retreat ingloriously from the field of
battle. The phrases firing a shot, retreat ingloriously, and field of battle all relate
to a military eventa battle not a popularity contest.

Explanation for Incorrect Answer D :

Choice (D) is incorrect. In lines 42-57, the narrator describes a group of progressive
mothers who would occasionally challenge Miss Dove; they felt she had been teaching
too long and that she rule[d] the children through fear! One motherthe boldest
one among [them]would enter Miss Doves geography room, but, once there, she
would lose her resolve, or determination. Ultimately, without firing a shot in the cause
of freedom she would retreat ingloriously from the field of battle. The phrases firing a
shot, retreat ingloriously, and field of battle all relate to a military eventa
battle not a courtroom trial.

From SAT Online Course CUUS


2007 May Sunday SAT Answers and Explanations

16 In the final paragraph, the narrator suggests that the students in Miss Doves
classroom benefited from

(A) an animated exchange of conflicting views


(B) an atmosphere of demanding challenges
(C) the example set by an outstanding role model
(D) the application of rigorous academic standards
(E) the subordination of individual opinion and desire

ANSWERS AND EXPLA NATIONS

Explanation for Correct Answer E :

Choice (E) is correct. In Miss Dove's classroom, the students' opinions and desires were
"subordinated," or made to submit to, the teacher's power and authority. In the final
paragraph, the narrator states that in Miss Doves classroom no leeway was given to
the personality; a thing was black or white, right or wrong, polite or rude, simply
because Miss Dove said it was. The narrator notes that Miss Doves classroom was the
only place where the children could enjoy a complete suspension of willin other
words, Miss Doves classroom was the only place where the children did not have to
make conscious choices or consider their own intentions. This subordination of
individual opinion and desire benefited the children, leaving them feeling refreshed
and restored, ready for fray or frolic.

Explanation for Incorrect Answer A :

Choice (A) is incorrect. Nowhere in the passage does the narrator suggest that Miss
Doves students engaged in animated exchanges of conflicting views. In fact, in the
final paragraph, the narrator explicitly states that in Miss Doves classroom a thing was
black or white, right or wrong, polite or rude, simply because Miss Dove said it was.
This suggests that there were no conflicting views in Miss Doves classroomthere was
only Miss Doves view.

Explanation for Incorrect Answer B :

Choice (B) is incorrect. In the final paragraph, the narrator states that in Miss Doves
classroom no leeway was given to the personality; a thing was black or white, right
or wrong, polite or rude, simply because Miss Dove said it was. The narrator notes that
Miss Doves classroom was the only place where the children could enjoy a complete
suspension of willin other words, Miss Doves classroom was the only place where
the children did not have to make conscious choices or consider their own inte ntions.
This suggests that the classroom did not have an atmosphere of demanding challenges;
rather than being challenged, the students simply had to follow Miss Doves rules.

From SAT Online Course CUUS


2007 May Sunday SAT Answers and Explanations

Explanation for Incorrect Answer C :

Choice (C) is incorrect. In the final paragraph, the narrator states that in Miss Doves
classroom no leeway was given to the personality; a thing was black or white, right
or wrong, polite or rude, simply because Miss Dove said it was. The narrator notes that
Miss Doves classroom was the only place where the children could enjoy a complete
suspension of willin other words, Miss Doves classroom was the only place where
the children did not have to make conscious choices or consider their own inte ntions.
Miss Dove may have been an outstanding role model for her students, but the narrator
does not address this idea in the final paragraph. The narrator only suggests that the
students benefitedthey felt refreshed and restored, ready for fray or frolicafter
experiencing in Miss Doves classroom the subordination, or complete submission, of
individual opinion and desire.

Explanation for Incorrect Answer D :

Choice (D) is incorrect. In the final paragraph, the narrator states that in Miss Doves
classroom no leeway was given to the personality; a thing was black or white, right
or wrong, polite or rude, simply because Miss Dove said it was. The narrator notes that
Miss Doves classroom was the only place where the children could enjoy a complete
suspension of willin other words, Miss Doves classroom was the only place where
the children did not have to make conscious choices or consider their own inte ntions.
Miss Dove may have had rigorous academic standards, but this idea is not addressed in
the final paragraph. Indeed, the entire passage focuses on Miss Doves rigorous
standards for classroom behavior, not on her academic standards.

17 The primary purpose of the passage is to

(A) analyze a commonplace problem


(B) criticize a type of teacher
(C) analyze a particular lifestyle
(D) present a character sketch
(E) describe a childhood memory

ANSWERS AND EXPLA NATIONS

Explanation for Correct Answer D :

Choice (D) is correct. A character sketch is a descriptive or narrative passage that is


intended to present the character, or essential nature, of an individual. The primary
purpose of the passage is to present a character sketch of Miss Dove. The reader learns
that Miss Dove had strict rules, that students as well as parents were intimidated by her,
and that in her classroom a thing was black or white, right or wrong, polite or rude,
simply because [she] said it was.

From SAT Online Course CUUS


2007 May Sunday SAT Answers and Explanations

Explanation for Incorrect Answer A :

Choice (A) is incorrect. The primary purpose of the passage is not to analyze a
commonplace problem, or a problem that is frequently encountered. Some of the
progressive mothers may have thought Miss Dove caused problems (She rules the
children through fear!), but the narrator does not seem to consider Miss Doves
methods a problem; on the contrary, the narrator points out that Miss Doves students
enjoyed . . . a complete suspension of will that left them refreshed and restored.
Further, the passage focuses on a specific teacher at a specific school; it is illogical to
characterize Miss Dove as a commonplace problem.

Explanation for Incorrect Answer B :

Choice (B) is incorrect. The primary purpose of the passage is not to criticize a type of
teacher. The narrator does discuss a teacher, Miss Dove, who penalized students for
breaking her rules and was capable of causing a students mother to retreat
ingloriously from her classroom. Although some might criticize Miss Dove for her
behavior, the narrator does not explicitly criticize Miss Dove or other teachers with
similar methods. On the contrary, the narrator ultimately praises the effects of Miss
Doves strict rules: Miss Doves students enjoyed . . . a complete suspension of will
and left her classroom feeling refreshed and restored.

Explanation for Incorrect Answer C :

Choice (C) is incorrect. The primary purpose of the passage is not to analyze a
particular lifestyle. The narrator discusses Miss Doves character as a teacher but does
not address her lifestyle, or her way of life. There is no description of Miss Doves
behavior or way of life outside of the classroom. The narrator focuses only on Miss
Doves behavior in the classroom, noting that the teacher had strict rules and that in
her classroom a thing was black or white, right or wrong, polite or rude, simply
because [she] said it was.

Explanation for Incorrect Answer E :

Choice (E) is incorrect. Although the passage makes reference to the narrators
childhood, the primary purpose of the passage is not to describe a childhood memory;
the narrator does not describe a specific event or incident in detail. Rather, the narrator
focuses on presenting a character sketch of a teacher, Miss Dove. The reader learns
that Miss Dove had strict rules, that students as well as parents were intimidated by her,
and that in her classroom a thing was black or white, right or wrong, polite or rude,
simply because [she] said it was.

This passage discusses the search during the 1950s for a vaccine to fight polio, an
infectious disease affecting the muscular system.

From SAT Online Course CUUS


2007 May Sunday SAT Answers and Explanations

In little over a year, a small laboratory experiment had


become a national event of a size and complexity never
seen before. The testing of the Salk vaccine was the largest
Line field trial ever held, the greatest peacetime mobilization
5 of civilians in United States history, and the most eagerly
observed and heavily publicized scientific program until
the space launches a decade later. Jonas Salk became an
instant hero and an enduring celebrity, the idol and icon
of his age. The announcement that his vaccine worked was
10 a landmark in twentieth-century history, and one of the few
events that burned itself into the consciousness of the world
because the news was good.
Privately funded, privately organized and supervised,
and quite publicly and conspicuously successful, the dis-
15 covery and testing of the Salk vaccine was in many ways
a crowning example of democratic self-help, the mass
organization of indiv idual citizens in a united effort for the
public good. It was also an effort marred by bitter disputes
over procedure, vicious struggles for power and prestige, a
20 small but tragic residue of avoidable injury, and a complete
failure to make an orderly transition from experimentation
to implementation and from laboratory prototypes to large-
scale commercial production.
The conflicts that surrounded the Salk vaccine field
25 trial were not between good and evil, right and wrong,
but between different ideas about what constituted the
good and the right. For over twenty years, several groups
had been attacking the problems of polio along entirely
separate lines. The National Foundation for Inf antile
30 Paralysis created a national network of concerned vol-
unteers, community "experts" on polio who raised money
for medical costs at the same time that they raised aware-
ness of the malady itself. Research scientists, many
supported in whole or part by the National Foundation,
35 studied problems in virology and immunology. Doctors
learned to recognize the elusive symptoms of polio and
prescribe ever better treatments and therapies. The Public
Health Service charted epidemics and quarantined patients.
The public worried, hoped, and waited.
40 Early in 1953, when reports began to appear that a polio
vaccine might be at hand, each of these groups, and each
indiv idual within each group, was firmly in possession of
a set of assumpt ions about how matters would proceed.

From SAT Online Course CUUS


2007 May Sunday SAT Answers and Explanations

Unf ortunately, they were rarely the same assumpt ions.


45 Everyone agreed that it would be a great thing to eliminate
polio. After that the consensus started to break down. Many
involved in organizing the Salk vaccine field trial seemed
to operate on the theory that, if they moved ahead as fast
as possible, their own course of action would develop
50 unstoppable momentum, enough to bring all opposition
into line. This wasnt true, but for a time it led to the ragged
spectacle of a program mov ing briskly in several directions
at once, like an unruly parade in which each section has its
own band and travels its own route, banging its own drums
55 as loudly as possible. Through most of 1953 and 1954, the
march against polio was accompanied by the loud clash of
agendas, the shrill noise of shattering assumpt ions, and the
whoosh of fond dreams punctured and expectations dashed.

18 The first paragraph (lines 1-12) reveals which of the following about the Salk
vaccine?

(A) Its creation was dependent on a number of other events .


(B) Its success was jeopardized by those who were skeptical of the project.
(C) It made other avenues of scientific research possible.
(D) It represented a vital achievement in its time.
(E) It was developed by using the most important new technology of the twentieth
century.

ANSWERS AND EXPLA NATIONS

Explanation for Correct Answer D :

Choice (D) is correct. According to the first paragraph, the Salk vaccine was a vital, or
hugely important, achievement in its time; the announcement of the vaccine was a
landmark in twentieth-century history," and Salk, the inventor of the vaccine, "became
an instant hero and an enduring celebrity, the idol and icon of his age." Clearly, the
development of the vaccine was an event of great importance to people living in the
United States in the 1950s.

Explanation for Incorrect Answer A :

Choice (A) is incorrect. According to the first paragraph, the Salk vaccine was a vital, or
hugely important, achievement in its time; the announcement of the vaccine was a
landmark in twentieth-century history," and Salk, the inventor of the vaccine, "became
an instant hero and an enduring celebrity, the idol and icon of his age." The first
paragraph reveals nothing about other events on which the creation of the Salk vaccine
was dependent; its focus is on the vaccine's prominence to the people of the time.

From SAT Online Course CUUS


2007 May Sunday SAT Answers and Explanations

Explanation for Incorrect Answer B :

Choice (B) is incorrect. According to the first paragraph, the Salk vaccine was a vital, or
hugely important, achievement in its time; the announcement of the vaccine was a
landmark in twentieth-century history," and Salk, the inventor of the vaccine, "became
an instant hero and an enduring celebrity, the idol and icon of his age." The first
paragraph does not suggest that the success of the vaccine was jeopardized, or put at
risk, by opponents; its focus is on the vaccine's prominence to the people of the time.

Explanation for Incorrect Answer C :

Choice (C) is incorrect. According to the first paragraph, the Salk vaccine was a vital, or
hugely important, achievement in its time; the announcement of the vaccine was a
landmark in twentieth-century history," and Salk, the inventor of the vaccine, "became
an instant hero and an enduring celebrity, the idol and icon of his age." The first
paragraph reveals nothing about other avenues of research related to the vaccine; its
focus is on the vaccine's prominence to the people of the time.

Explanation for Incorrect Answer E :

Choice (E) is incorrect. According to the first paragraph, the Salk vaccine was a vital, or
hugely important, achievement in its time; the announcement of the vaccine was a
landmark in twentieth-century history," and Salk, the inventor of the vaccine, "became
an instant hero and an enduring celebrity, the idol and icon of his age." The first
paragraph does not mention any new technologies; its focus is on the vaccine's
prominence to the people of the time.

19 The discussion of the Salk vaccine in lines 1-7 (In . . . later) employs which of
the following?

(A) A historical timeline citing key events leading to the testing of the vaccine
(B) Comparisons between the vaccines development and other important
accomplishments
(C) A listing of groups and individuals most directly affected by the vaccines
creation
(D) Various attitudes about the vaccine gathered from different sources
(E) The juxtaposition of a critical and an admiring view of the vaccine

ANSWERS AND EXPLA NATIONS

Explanation for Correct Answer B :

Choice (B) is correct. The two sentences in lines 1-7 compare the development of the
Salk vaccine to other accomplishments some identified and some not. The testing of

From SAT Online Course CUUS


2007 May Sunday SAT Answers and Explanations

the vaccine is described in comparison with all other projects that preceded it: the
event was "of a size and complexity never seen before"; it was "the largest field trial
ever held"; it was "the greatest peacetime mobiliza tion of citizens " in the nation's
history. Moreover, it was "the most eagerly observed and heavily publicized scientific
program until the space launches." The two sentences form an extended comparison.

Explanation for Incorrect Answer A :

Choice (A) is incorrect. The two sentences in lines 1-7 compare the development of the
Salk vaccine to other accomplishments some identified and some not. The testing of
the vaccine is described in comparison with all other projects that preceded it: the
event was "of a size and complexity never seen before"; it was "the largest field trial
ever held"; it was "the greatest peacetime mobiliza tion of citizens " in the nation's
history. No historical timeline of key events is presented in these lines.

Explanation for Incorrect Answer C :

Choice (C) is incorrect. The two sentences in lines 1-7 compare the development of the
Salk vaccine to other accomplishments some identified and some not. The testing of
the vaccine is described in comparison with all other projects that preceded it: the
event was "of a size and complexity never seen before"; it was "the largest field trial
ever held"; it was "the greatest peacetime mobiliza tion of citizens " in the nation's
history. No list of groups or individuals affected by the vaccines creation is provided in
these lines.

Explanation for Incorrect Answer D :

Choice (D) is incorrect. The two sentences in lines 1-7 compare the development of the
Salk vaccine to other accomplishments some identified and some not. The testing of
the vaccine is described in comparison with all other projects that preceded it: the
event was "of a size and complexity never seen before"; it was "the largest field trial
ever held"; it was "the greatest peacetime mobiliza tion of citizens " in the nation's
history. The author does not refer to various attitudes about the vaccine; the only
attitude revealed here is the enthusiastic attitude of the author.

Explanation for Incorrect Answer E :

Choice (E) is incorrect. The two sentences in lines 1-7 compare the development of the
Salk vaccine to other accomplishments some identified and some not. The testing of
the vaccine is described in comparison with all other projects that preceded it: the
event was "of a size and complexity never seen before"; it was "the largest field trial
ever held"; it was "the greatest peacetime mobiliza tion of citizens " in the nation's
history. There is no juxtaposition, or contrast of opposing views, in these lines; the only
view revealed is that of the author.

From SAT Online Course CUUS


2007 May Sunday SAT Answers and Explanations

20 Lines 9-12 (The announcement . . . good) imply that

(A) the reaction to the news of the Salk vaccine was somewhat overblown
(B) the media had a great deal of influence on the public response to the Salk
vaccine
(C) the twentieth century had more medical achievements than did any previous
period
(D) people usually overlook problems until they become impossible to ignore
(E) disasters were usually more widely known and better remembered than were
successes

ANSWERS AND EXPLA NATIONS

Explanation for Correct Answer E :

Choice (E) is correct. The last sentence of the first paragraph states that the
development of the Salk vaccine "was one of the few events that burned itself into the
consciousness of the world because the news was good." The author clearly implies
that most events that "burn themselves into the consciousness of the world" are bad:
natural disasters, wars, economic catastrophes, and so on. The news of a life-saving
vaccine, "one of the few" successes to become widely known and well remembered,
forms a happy contrast to the disasters and tragedies that people tend to remember
best.

Explanation for Incorrect Answer A :

Choice (A) is incorrect. The last sentence of the first paragraph states that the
development of the Salk vaccine "was one of the few events that burned itself into the
consciousness of the world because the news was good." The author clearly implies
that most events that "burn themselves into the consciousness of the world" are bad:
natural disasters, wars, economic catastrophes, and so on. The author does not
suggest that the reaction to the news of the vaccine was overblown, or excessive. He or
she simply indicates that the life-saving vaccine was "one of the few" successes to
become widely known and well remembered.

Explanation for Incorrect Answer B :

Choice (B) is incorrect. The last sentence of the first paragraph states that the
development of the Salk vaccine "was one of the few events that burned itself into the
consciousness of the world because the news was good." The author clearly implies
that most events that "burn themselves into the consciousness of the world" are bad:
natural disasters, wars, economic catastrophes, and so on. The author does not
mention that the media had anything to do with the response to the vaccine. He or she
simply indicates that the life-saving vaccine was "one of the few" successes to become
widely known and well remembered.

From SAT Online Course CUUS


2007 May Sunday SAT Answers and Explanations

Explanation for Incorrect Answer C :

Choice (C) is incorrect. The last sentence of the first paragraph states that the
development of the Salk vaccine "was one of the few events that burned itself into the
consciousness of the world because the news was good." The author clearly implies
that most events that "burn themselves into the consciousness of the world" are bad:
natural disasters, wars, economic catastrophes, and so on. The author does not
compare the twentieth century's achievements, in general, to those of previous
centuries. He or she simply indicates that the life-saving vaccine was "one of the few"
successes to become widely known and well remembered.

Explanation for Incorrect Answer D :

Choice (D) is incorrect. The last sentence of the first paragraph states that the
development of the Salk vaccine "was one of the few events that burned itself into the
consciousness of the world because the news was good." The author clearly implies
that most events that "burn themselves into the consciousness of the world" are bad:
natural disasters, wars, economic catastrophes, and so on. The author does not
comment on people's tendency to overlook certain problems. He or she simply indicates
that the life-saving vaccine was "one of the few" successes to become widely known
and well remembered.

21 The primary purpose of lines 24-27 (The conflicts . . . right ) is to

(A) expand upon a point made in the previous paragraph


(B) argue in favor of a relatively unpopular position
(C) offer an argument that runs counter to the one made in the first paragraph
(D) relate an anecdote to dramatize the social significance of a breakthrough
(E) call into question the relevance of the examples that follow

ANSWERS AND EXPLA NATIONS

Explanation for Correct Answer A :

Choice (A) is correct. The previous paragraph (lines 13-24) makes the point that, while
the testing of the Salk vaccine was a success, it was "marred by bitter disputes over
procedure" and "vicious struggles for power and prestige." Lines 24-27 expand upon
that point, explaining the nature of the disputes and struggles: "the conflicts . . . were
not between good and evil, right and wrong, but between different ideas."

Explanation for Incorrect Answer B :

Choice (B) is incorrect. The previous paragraph (lines 13-24) makes the point that,
while the testing of the Salk vaccine was a success, it was "marred by bitter disputes

From SAT Online Course CUUS


2007 May Sunday SAT Answers and Explanations

over procedure" and "vicious struggles for power and prestige." Lines 24-27 expand
upon that point, explaining the nature of the disputes and struggles: "the conflicts . . .
were not between good and evil, right and wrong, but between different ideas." The
author of the passage is not arguing in favor of an unpopular position, or of any
position at all. He or she is merely giving additional information about, and analysis of,
the conflicts mentioned in the previous paragraph.

Explanation for Incorrect Answer C :

Choice (C) is incorrect. The previous paragraph (lines 13-24) makes the point that,
while the testing of the Salk vaccine was a success, it was "marred by bitter disputes
over procedure" and "vicious struggles for power and prestige." Lines 24-27 expand
upon that point, explaining the nature of the disputes and struggles: "the conflicts . . .
were not between good and evil, right and wrong, but between different ideas." The
author of the passage is not countering the argument made in the first paragraph of
the passage, which introduces the Salk vaccine as an important success. He or she is
simply offering a closer look at some of the conflicts that occurred during the
development of the vaccine.

Explanation for Incorrect Answer D :

Choice (D) is incorrect. The previous paragraph (lines 13-24) makes the point that,
while the testing of the Salk vaccine was a success, it was "marred by bitter disputes
over procedure" and "vicious struggles for power and prestige." Lines 24-27 expand
upon that point, explaining the nature of the disputes and struggles: "the conflicts . . .
were not between good and evil, right and wrong, but between different ideas." The
author of the passage is not providing an anecdote, or brief story. He or she is merely
giving additional information about, and analysis of, the conflicts mentioned in the
previous paragraph.

Explanation for Incorrect Answer E :

Choice (E) is incorrect. The previous paragraph (lines 13-24) makes the point that,
while the testing of the Salk vaccine was a success, it was "marred by bitter disputes
over procedure" and "vicious struggles for power and prestige." Lines 24-27 expand
upon that point, explaining the nature of the disputes and struggles: "the conflicts . . .
were not between good and evil, right and wrong, but between different ideas." The
author of the passage is not calling into question the relevance of any examples. He or
she is merely giving additional information about, and analysis of, the conflicts
mentioned in the previous paragraph.

22 Lines 29-38 (The National . . . patients) serve primarily to

(A) clarify a little-understood philosophy

From SAT Online Course CUUS


2007 May Sunday SAT Answers and Explanations

(B) undermine an argument put forth by an authority


(C) illustrate a general statement using specific examples
(D) praise the scope of a powerful international effort
(E) offer evidence of scientific improprieties

ANSWERS AND EXPLA NATIONS

Explanation for Correct Answer C :

Choice (C) is correct. In lines 29-38, the author of the passage indicates that several
different groups contributed to the fight against polio, including "the National
Foundation for Infantile Paralysis," "research scientists," "doctors," and "the Public
Health Service." In so doing, the author provides specific examples in support of the
general statement that "for over twenty years, several groups had been attacking the
problems of polio along entirely separate lines." Lines 29-38 list the groups and note
their activities.

Explanation for Incorrect Answer A :

Choice (A) is incorrect. In lines 29-38, the author of the passage indicates that several
different groups contributed to the fight against polio, including "the National
Foundation for Infantile Paralysis," "research scientists," "doctors," and "the Public
Health Service." In so doing, the author provides specific examples in support of the
general statement that "for over twenty years, several groups had been attacking the
problems of polio along entirely separate lines." Lines 29-38 do not clarify a philosophy.

Explanation for Incorrect Answer B :

Choice (B) is incorrect. In lines 29-38, the author of the passage indicates that several
different groups contributed to the fight against polio, including "the National
Foundation for Infantile Paralysis," "research scientists," "doctors," and "the Public
Health Service." In so doing, the author provides specific examples in support of the
general statement that "for over twenty years, several groups had been attacking the
problems of polio along entirely separate lines." Lines 29-38 do not undermine an
argument made by an authority; rather, they support an argument made by the author
concerning the fight against polio.

Explanation for Incorrect Answer D :

Choice (D) is incorrect. In lines 29-38, the author of the passage indicates that several
different groups contributed to the fight against polio, including "the National
Foundation for Infantile Paralysis," "research scientists," "doctors," and "the Public
Health Service." In so doing, the author provides specific examples in support of the
general statement that "for over twenty years, several groups had been attacking the
problems of polio along entirely separate lines." While the reader can infer that the

From SAT Online Course CUUS


2007 May Sunday SAT Answers and Explanations

fight against polio was broad in scope, these lines do not offer praise. Moreover, the
focus of these linesand the passage as a wholeis national, not international.

Explanation for Incorrect Answer E :

Choice (E) is incorrect. In lines 29-38, the author of the passage indicates that several
different groups contributed to the fight against polio, including "the National
Foundation for Infantile Paralysis," "research scientists," "doctors," and "the Public
Health Service." In so doing, the author provides specific examples in support of the
general statement that "for over twenty years, several groups had been attacking the
problems of polio along entirely separate lines." Lines 29-38 do not discuss scientific
improprieties, or things done improperly by scientists.

23 In line 39, the attitude of the public can best be described as

(A) belligerent
(B) cynical
(C) apathetic
(D) anxious
(E) critical

ANSWERS AND EXPLA NATIONS

Explanation for Correct Answer D :

Choice (D) is correct. "Anxious" means uneasy or worried. The passage is about the
search for a vaccine to fight polio, a disease whose effects included infantile paralysis.
The author explains that as different groups pursued different strategies in the fight
against polio, "the public worried, hoped, and waited"the public was anxious.

Explanation for Incorrect Answer A :

Choice (A) is incorrect. "Belligerent" means hostile or combative. The passage is about
the search for a vaccine to fight polio, a disease whose effects included infantile
paralysis. The reader can infer that members of the public, especially those with young
children, were very concerned about this disease, but there is no indication that they
ever acted in a way that could be described as belligerent. The author explains that as
different groups pursued different strategies in the fight against polio, "the public
worried, hoped, and waited"the public was anxious.

Explanation for Incorrect Answer B :

Choice (B) is incorrect. "Cynical" means negative and distrustful. The passage is about
the search for a vaccine to fight polio, a disease whose effects included infantile

From SAT Online Course CUUS


2007 May Sunday SAT Answers and Explanations

paralysis. The reader can infer that members of the public, especially those with young
children, were very concerned about this disease, but there is no indication that they
were cynical. In fact, even though the public worried . . . and waited as different
groups pursued different strategies in the fight against polio, the public also hoped.

Explanation for Incorrect Answer C :

Choice (C) is incorrect. "Apathetic" means indifferent or lacking interest or concern. The
passage is about the search for a vaccine to fight polio, a disease whose effects
included infantile paralysis. The reader can infer that members of the public, especially
those with young children, were very concerned about this disease; there is no
indication that they were apathetic. In fact, as different groups pursued different
strategies in the fight against polio, "the public worried, hoped, and waited."

Explanation for Incorrect Answer E :

Choice (E) is incorrect. In this context, "critical" means finding fault with something.
The passage is about the search for a vaccine to fight polio, a disease whose effects
included infantile paralysis. The reader can infer that members of the public, especially
those with young children, were very concerned about this disease. There is no
indication that they were critical of efforts to combat the disease. Rather, as different
groups pursued different strategies in the fight against polio, the public worried, hoped,
and waited."

24 Which is true of the groups and individuals described in lines 40-46 (Early . . .
down)?

(A) They agreed on the same goal but not on the way to achieve it.
(B) They had similar doubts about how effective a vaccine for polio could be.
(C) They were eager to work together but unsure of how to proceed.
(D) They were willing to tolerate different levels of risk when working with a
contagious disease.
(E) They were uncertain that the public would continue to wait patiently for a cure.

ANSWERS AND EXPLA NATIONS

Explanation for Correct Answer A :

Choice (A) is correct. Lines 45-46 sum up the situation nicely: "Everyone agreed that it
would be a great thing to eliminate polio. After that the consensus started to break
down." The groups and individuals agreed on the same goal, but differed on virtually
everything else, including the way to achieve that goal.

Explanation for Incorrect Answer B :

From SAT Online Course CUUS


2007 May Sunday SAT Answers and Explanations

Choice (B) is incorrect. Lines 45-46 sum up the situation nicely: "Everyone agreed that
it would be a great thing to eliminate polio. After that the consensus started to break
down." The groups and individuals agreed on the same goal, but differed on virtually
everything else, including the way to achieve that goal. The author does not discuss
doubts about the effectiveness of a polio vaccine.

Explanation for Incorrect Answer C :

Choice (C) is incorrect. Lines 45-46 sum up the situation nicely: "Everyone agreed that
it would be a great thing to eliminate polio. After that the consensus started to break
down." The groups and individuals agreed on the same goal, but differed on virtually
everything else, including the way to achieve that goal. The author does not indicate
that the groups were eager to work together. Rather, he or she presents the "ragged
spectacle" of each group working on its own to solve the problem.

Explanation for Incorrect Answer D :

Choice (D) is incorrect. Lines 45-46 sum up the situation nicely: "Everyone agreed that
it would be a great thing to eliminate polio. After that the consensus started to break
down." The groups and individuals agreed on the same goal, but differed on virtually
everything else, including the way to achieve that goal. The author does not indicate
the levels of risk the groups were willing to tolerate.

Explanation for Incorrect Answer E :

Choice (E) is incorrect. Lines 45-46 sum up the situation nicely: "Everyone agreed that
it would be a great thing to eliminate polio. After that the consensus started to break
down." The groups and individuals agreed on the same goal, but differed on virtually
everything else, including the way to achieve that goal. The author does not discuss the
groups' attitude toward the public.

Section8: Critical Reading

View Answers and Explanations

Online - Practice Test #2

1 Crucial to the expeditions ------- will be accurate assumptions about the weather;
without them, the safety of the mountain climbers may be ------- .

(A) prosperity . . enhanced

From SAT Online Course CUUS


2007 May Sunday SAT Answers and Explanations

(B) survival . . demonstrated


(C) futility . . compromised
(D) plausibility . . ensured
(E) success . . jeopardized

ANSWERS AND EXPLA NATIONS

Explanation for Correct Answer E :

Choice (E) is correct. Success is a favorable or desired outcome. To jeopardize


something is to expose it to danger or risk. According to the part of the sentence before
the semicolon, accurate assumptions about the weather are crucial, or essential, to a
certain outcome of the expedition. The part of the sentence after the semicolon
indicates that a lack of accurate assumptions would result in a situation that could
prevent the outcome described in the first part. Without accurate assumptions about
the weather, the safety of the mountain climbers could certainly be jeopardized that
is, the mountain climbers could be exposed to danger or risk. This situation would likely
prevent the success of the expedition, or lead to an unfavorable outcome; therefore,
accurate assumptions about the weather are essential for the expedition to be
successful.

Explanation for Incorrect Answer A :

Choice (A) is incorrect. Prosperity is the condition of being successful or thriving. To


enhance something is to increase or improve its value, quality, desirability, or
attractiveness. According to the part of the sentence before the semicolon, accurate
assumptions about the weather are crucial, or essential, to a certain outcome of the
expedition. The part of the sentence after the semicolon indicates that a lack of
accurate assumptions would result in a situation that could prevent the outcome
described in the first part. Accurate assumptions about the weather could certainly
result in the prosperity of the expedition. However, it is illogical to suggest that the
mountain climbers safety would be enhanced by not having accurate assumptions
about the weathertheir safety would not increase or improve. On the contrary, the
climbers safety would be jeopardized, or put at risk. Further, enhancing the mountain
climbers safety would contribute to the prosperity of the expedition, not prevent it.

Explanation for Incorrect Answer B :

Choice (B) is incorrect. Survival is the continuation of life or existence. To


demonstrate means to prove or make clear by reasoning or evidence. According to the
part of the sentence before the semicolon, accurate assumptions about the weather are
crucial, or essential, to a certain outcome of the expedition. The part of the sentence
after the semicolon indicates that a lack of accurate assumptions would result in a
situation that could prevent the outcome described in the first part. Accurate
assumptions about the weather could certainly result in the survival of the expedition.

From SAT Online Course CUUS


2007 May Sunday SAT Answers and Explanations

However, it does not make sense to say that the safety of the mountain climbers would
be demonstrated as a result of not having accurate assumptions.

Explanation for Incorrect Answer C :

Choice (C) is incorrect. Futility is the quality or state of being useless or completely
ineffective. In this context, the term compromised means exposed to danger.
According to the part of the sentence before the semicolon, accurate assumptions
about the weather are crucial, or essential, to a certain outcome of the expedition.
The part of the sentence after the semicolon indicates that a lack of accurate
assumptions would result in a situation that could prevent the outcome described in the
first part. Without accurate assumptions about the weather, the safety of the mountain
climbers could certainly be compromisedthat is, the mountain climbers could be
exposed to danger or risk. However, this situation would not prevent the futility of the
expedition; on the contrary, it would likely result in the expedition being useless or
ineffective. Further, accurate assumptions would be crucial to the expedition being
useful, not useless.

Explanation for Incorrect Answer D :

Choice (D) is incorrect. Something that is plausible appears worthy of belief. Ensured
means made sure, certain, or safe. According to the part of the sentence before the
semicolon, accurate assumptions about the weather are crucial, or essential, to a
certain outcome of the expedition. The part of the sentence after the semicolon
indicates that a lack of accurate assumptions would result in a situation that could
prevent the outcome described in the first part. It does not make sense to describe an
expeditiona group of people on a journeyas plausible, or to suggest that
assumptions about the weather could cause others to regard the expedition as worthy
of belief. Further, without accurate assumptions about the weather, the mountain
climbers safety would not be ensured, or certain; rather, their safety would be
jeopardized, or put at risk.

2 The universitys aggressive transformation from a teaching-centered college to a


major research institution has brought ------- that teaching is now being ------- .

(A) suspicions . . promoted


(B) recommendations . . discussed
(C) accusations . . neglected
(D) insinuations . . praised
(E) recriminations . . emphasized

ANSWERS AND EXPLA NATIONS

Explanation for Correct Answer C :

From SAT Online Course CUUS


2007 May Sunday SAT Answers and Explanations

Choice (C) is correct. An accusation is a charge of wrongdoing. To neglect something is


to give it little attention or respect. The sentence indicates that the university is no
longer a teaching-centered collegeit has become an institution focused on research
rather than teaching. People who believe that this aggressive transformation is wrong
would likely make accusations that teaching is now being neglected.

Explanation for Incorrect Answer A :

Choice (A) is incorrect. A suspicion is an instance of suspecting something, especially


something wrong, on little evidence or no proof. To promote something is to advance it
in station, rank, or honor. The sentence indicates that the university is no longer a
teaching-centered collegeit has become an institution focused on research rather
than teaching. People who believe that this aggressive transformation is wrong could
certainly be suspicious of the universitys actions. However, it is not logical to suggest
that teaching would be promoted at a university that is no longer teaching-centered.

Explanation for Incorrect Answer B :

Choice (B) is incorrect. A recommendation is an act of recommending or advising


something. To discuss something is to talk about it or to present it in detail for
examination or consideration. The sentence indicates that the university is no longer a
teaching-centered collegeit has become an institution focused on research rather
than teaching. People who believe this aggressive transformation is wrong would
likely recommend that teaching should be discussed; however, it does not make
sense to suggest that teaching is now being discussed at a university that is no longer
teaching-centered.

Explanation for Incorrect Answer D :

Choice (D) is incorrect. To insinuate is to imply, or to introduce gradually or in a subtle,


indirect, or covert way. To praise something is to express a favorable judgment of it.
The sentence indicates that the university is no longer a teaching-centered collegeit
has become an institution focused on research rather than teaching. It is somewhat
illogical to suggest that people would make insinuations that teaching is now being
praised at a university that is no longer teaching-centered.

Explanation for Incorrect Answer E :

Choice (E) is incorrect. A recrimination is a retaliatory accusation, or a charge of


wrongdoing that counters another charge of wrongdoing. To emphasize something is to
place emphasis on it, or to give it special attention. The sentence indicates that the
university is no longer a teaching-centered collegeit has become an institution
focused on research rather than teaching. It does not make sense to suggest that
teaching is now being emphasized at a university that is no longer teaching-centered.
Further, it is unlikely that people would make recriminations about teaching being em

From SAT Online Course CUUS


2007 May Sunday SAT Answers and Explanations

phasized; there is no indication that anyone is responding to an initial charge of


wrongdoing, and it is unlikely that an emphasis on teaching would be considered a
wrongdoing.

3 Certain plant seeds are able to remain ------- for years, appearing lifeless when in
fact they are merely inactive.

(A) conspicuous
(B) verdant
(C) pliant
(D) dormant
(E) stunted

ANSWERS AND EXPLA NATIONS

Explanation for Correct Answer D :

Choice (D) is correct. Dormant means inactive but capable of being activated. The
structure of the sentence suggests that the text after the comma explains the meaning
of the missing term. The term dormant accurately describes something that appears
lifeless but in fact is merely inactive.

Explanation for Incorrect Answer A :

Choice (A) is incorrect. Something that is conspicuous is noticeable, or obvious to the


eye or mind. The structure of the sentence suggests that the text after the comma
explains the meaning of the missing termtherefore, the missing term should describe
something that might appear lifeless but in fact is merely inactive. Some plant
seeds might be obvious to the eye, or easily seen. However, the term conspicuous
does not necessarily describe something that seems lifeless but actually is inactive;
there is no direct connection between being inactive and being conspicuous.

Explanation for Incorrect Answer B :

Choice (B) is incorrect. In this context, verdant means green with growing plants or
vegetation. The structure of the sentence suggests that the text after the comma
explains the meaning of the missing termtherefore, the missing term should describe
something that might appear lifeless but in fact is merely inactive. The term
verdant does not describe something that seems lifeless but actually is inactive; on
the contrary, something that is green with growing plants is certainly active and likely
appears to be full of life.

Explanation for Incorrect Answer C :

From SAT Online Course CUUS


2007 May Sunday SAT Answers and Explanations

Choice (C) is incorrect. In this context, pliant means adaptable to varying conditions.
The structure of the sentence suggests that the text after the comma explains the
meaning of the missing termtherefore, the missing term should describe something
that might appear lifeless but in fact is merely inactive. The term pliant does not
describe something that seems lifeless but actually is inactive; indeed, something that
is inactive probably cannot adapt to varying conditions.

Explanation for Incorrect Answer E :

Choice (E) is incorrect. If something is stunted, its normal growth, development, or


progress is hindered. The structure of the sentence suggests that the text after the
comma explains the meaning of the missing termtherefore, the missing term should
describe something that might appear lifeless but in fact is merely inactive. The
term stunted does not necessarily describe something that appears lifeless but
actually is inactive. A plant seed s inactivity might be completely normal and not a
result of the seed s growth or development being hindered.

4 Wanting desperately to be perceived as learned and authoritative, Johnson


affected a ------- tone when speaking in public.

(A) licentious
(B) nihilistic
(C) magisterial
(D) propitiatory
(E) rapturous

ANSWERS AND EXPLA NATIONS

Explanation for Correct Answer C :

Choice (C) is correct. Magisterial means authoritative, or having the characteristics of


a master or teacher. The sentence indicates that Johnson wanted desperately to be
perceived as learned and authoritative, so it makes sense to say that he or she
"affected," or put on, a magisterial tone when speaking.

Explanation for Incorrect Answer A :

Choice (A) is incorrect. Licentious means having no regard for rules or standards. The
sentence indicates that Johnson wanted desperately to be perceived as learned and
authoritative. Someone who is authoritative holds rules or standards in high regard, so
it is unlikely that Johnson would affect, or put on, a licentious tone if he or she wanted
to be perceived as an authoritative person.

Explanation for Incorrect Answer B :

From SAT Online Course CUUS


2007 May Sunday SAT Answers and Explanations

Choice (B) is incorrect. Someone who is nihilistic rejects or dismisses authority. The
sentence indicates that Johnson wanted desperately to be perceived as learned and
authoritative ; there is no reason to believe that someone who wants to be seen as
authoritative would affect, or put on, a nihilistic tone when speaking.

Explanation for Incorrect Answer D :

Choice (D) is incorrect. Propitiatory means intended to propitiate, or to gain or regain


someones favor or goodwill. The sentence indicates that Johnson wanted desperately
to be perceived as learned and authoritative. Someone who wants to be seen in a
positive way might affect, or put on, a tone intended to gain someones goodwill, but
there is no direct connection between using a propitiatory tone and being seen as
learned and authoritative. Further, someone who wants to be seen as authoritative
might expect others to try to gain his or her favor.

Explanation for Incorrect Answer E :

Choice (E) is incorrect. Rapture is a state or experience of being carried away with
overwhelming emotion. The sentence indicates that Johnson wanted desperately to be
perceived as learned and authoritative. Someone who wants to be seen as
authoritative would likely focus on rules, standards, and power, rather than emotion.
Therefore, it is unlikely that Johnson would affect, or put on, a rapturous tone if he or
she wanted to be perceived as an authoritative person.

5 Critics characterized the memoir as ------- because of the authors excessively


sentimental description of her childhood.

(A) maudlin
(B) candid
(C) pedantic
(D) enigmatic
(E) eclectic

ANSWERS AND EXPLA NATIONS

Explanation for Correct Answer A :

Choice (A) is correct. Maudlin means effusively sentimental, or excessively governed


by feelings and emotions. The structure of the sentence suggests that the memoir was
characterized in a certain way as a direct result of its being excessively sentimental;
it makes sense to characterize an excessively sentimental memoir as maudlin.

Explanation for Incorrect Answer B :

From SAT Online Course CUUS


2007 May Sunday SAT Answers and Explanations

Choice (B) is incorrect. In this context, candid means characterized by honest, sincere
expression. The structure of the sentence suggests that the memoir was characterized
in a certain way as a direct result of its being excessively sentimental, or excessively
governed by feelings and emotions. A memoir that is excessively sentimental might
also be honest and sincere, but the term candid does not necessarily describe
something that is sentimental.

Explanation for Incorrect Answer C :

Choice (C) is incorrect. Pedantic means unimaginative and condescending, and


typically refers to the presentation of something in a narrowly learned way. The
structure of the sentence suggests that the memoir was characterized in a certain way
as a direct result of its being excessively sentimental, or excessively governed by
feelings and emotions. Some critics might say that an excessively sentimental memoir
is also pedantic, but the term pedantic does not necessarily describe something that
is sentimental.

Explanation for Incorrect Answer D :

Choice (D) is incorrect. Enigmatic means mysterious. The structure of the sentence
suggests that the memoir was characterized in a certain way as a direct result of its
being excessively sentimental, or excessively governed by feeling and emotions. The
term enigmatic does not logically complete the sentence; nothing in the sentence
indicates that the memoir is in any way mysterious, and there is no direct connection
between being enigmatic and being excessively sentimental.

Explanation for Incorrect Answer E :

Choice (E) is incorrect. Something that is eclectic is made up of elements from a


variety of sources. The structure of the sentence suggests that the memoir was
characterized in a certain way as a direct result of its being excessively sentimental,
or excessively governed by feelings and emotions. The term eclectic does not logically
complete the sentence; nothing in the sentence indicates that the memoir is made up
of a variety of elements, and there is no direct connection between being eclectic and
being excessively sentimental.

6 Ernest Gainess A Lesson Before Dying is ------- of rural Louisiana: the writing is so
evocative that the Southern atmosphere seems almost to ------- from the books
pages.

(A) critical . . fade


(B) suggestive . . ebb
(C) reminiscent . . veer

From SAT Online Course CUUS


2007 May Sunday SAT Answers and Explanations

(D) dismissive . . seep


(E) redolent . . waft

ANSWERS AND EXPLA NATIONS

Explanation for Correct Answer E :

Choice (E) is correct. In this context, redolent means evocative, or tending to evoke a
particular time, place, or feeling. To waft is to float or drift through the air easily and
gently. The structure of the sentence suggests that the text after the colon further
explains the claim made in the first half of the sentence. The sentence indicates that
Gainess writing is evocative and has a Southern atmosphere, so it makes sense to
describe his book as redolent of Louisiana. And because the writing is so evocative, or
strongly evokes a particular time, place, or feeling, it makes sense to suggest that the
Southern atmosphere of the book actually seems to drift off of the pages and impact
the reader.

Explanation for Incorrect Answer A :

Choice (A) is incorrect. In this context, to be critical is to criticize or find fault with
something. To fade is to lose freshness, strength, or vitality. The structure of the
sentence suggests that the text after the colon further explains the claim made in the
first half of the sentence. The sentence indicates that Gainess writing is evocative;
writing that evokes a particular time, place, or feeling is not necessarily critical, and
nothing in the sentence suggests that Gaines finds fault with the Southern
atmosphere about which he writes. Further, rather than [fading] from the books
pages, the atmosphere Gaines writes about so evocatively would likely have strength
and vitality.

Explanation for Incorrect Answer B :

Choice (B) is incorrect. Suggestive means evocative or stirring mental associations.


To ebb is to fall away or back, to decline or become more distant. The structure of the
sentence suggests that the text after the colon further explains the claim made in the
first half of the sentence. The sentence indicates that Gainess writing is evocativeit
evokes a particular, time, place, or feelingso it makes sense to describe his book as
suggestive. But it does not make sense to suggest that such evocative writing would
make the atmosphere Gaines writes about seem to ebb, or fall away, from the books
pages; on the contrary, Gainess writing would likely make the atmosphere seem close
and immediate.

Explanation for Incorrect Answer C :

Choice (C) is incorrect. Reminiscent means tending to recall or suggest something in


the past. To veer is to change direction or course. The structure of the sentence
suggests that the text after the colon further explains the claim made in the first half of

From SAT Online Course CUUS


2007 May Sunday SAT Answers and Explanations

the sentence. The sentence indicates that Gainess writing is evocative, or evokes a
particular time, place, or feeling, so it might make sense to say that his book is
reminiscent of rural Louisiana; for some readers the book might evoke memories of
Louisiana. But it does not make sense to suggest that the atmosphere about which
Gaines writes would seem to veer from the books pagesit is not clear in what way
the Southern atmosphere could change direction.

Explanation for Incorrect Answer D :

Choice (D) is incorrect. To be dismissive is to show indifference or disregard. In this


context, to seep is to flow slowly or gradually. The structure of the sentence suggests
that the text after the colon further explains the claim made in the first half of the
sentence. The sentence indicates that Gainess writing is evocative, or evokes a
particular time, place, or feeling, so it might make sense to suggest that the
atmosphere about which he writes might seem to seep from the books pagesthe
atmosphere might seem to literally flow out of the pages to impact the reader. However,
there is no reason to believe that a book that so strongly evokes this Southern
atmosphere is dismissive of its setting; nothing in the sentence suggests that Gaines
shows disregard for rural Louisiana.

These passages relate to the notorious mutiny on the British naval ship Bounty in
April 1789. At the time of the mutiny, the ship had recently left Tahiti, a place
perceived by Europeans as extremely remote, after a six-month visit. Mutineers sailed
the Bounty back to Tahiti, leaving the captain, William Bligh, and some of his followers
in a small open boat with scanty supplies. Bligh successfully navigated 4,000 miles to
the nearest European settlement, and the mutiny became an international sensation.

But why did the mutiny happen? An edited version of Bligh's public answer appears in
Passage 1. A writer of British naval history provides another answer in Passage 2.

Passage 1

As soon as I had time to reflect, after being cast off from


my ship, I felt an inward satisfaction, which prevented any
depression of my spirits. With full consciousness of my
Line integrity, I found my mind wonderf ully comf orted. I began
5 to conceive hopes, notwithstanding so heavy a calamity,
that I should one day be able to account to my King and
country for the misf ortune.
A few hours earlier, my situation had been peculiarly
flattering. The voyage had been two-thirds completed, to
10 all appearance, in every promising way.
It will very naturally be asked, then, what could be

From SAT Online Course CUUS


2007 May Sunday SAT Answers and Explanations

the reason for such a revolt? I can only conjecture that


the mutineers had flattered themselves with the hopes
of a more happy lif e among the Tahitians than they
15 could possibly enjoy in England. This, in addition to
the sentimental ties many of the crew had formed with
the much admired indigenous women, most probably
occasioned the whole transaction.
The local chief s were so attached to our crew that they
20 encouraged their staying permanently and even made the
crew promises of large possessions. It is now perhaps not
so much to be wondered at (though scarcely possible to
have foreseen) that a set of sailors, most without inf luential
connections in England, should be led away.
25 The utmost, however, that any captain could have pre-
dicted is that some of the crew would have been tempted
to desert. Desertions have happened from most of the
ships that have been at Tahiti but it has always been in
the captain's power to make the chief s return the deserters.
30 The knowledge that it was unsafe to desert, perhaps, first
led my crew to consider with what ease so small a ship
as the Bounty might be surprised and taken.
The secrecy of this mutiny is beyond all conception.
To such a close-planned act of villainy, my mind being
35 entirely free from any suspicion, it is no wonder that I
fell a sacrifice. Had their mutiny been occasioned by any
grievances, either real or imaginary, I must have discovered
symptoms of their discontent, which would have put me
on my guard. The case was far otherwise. I was on most
40 friendly terms with Mr. Christian, the leader of the muti-
neers. He had previously arranged to dine with me on
the very day that the mutineers seized me from my bed
at dawn.

Passage 2

Bligh had an unpleasant character, which soon became


45 obvious to a great many people who came into contact with
him. Yes, Bligh was a courageous and eminently capable
seaman no one else could have accomplished that aston-
ishing open-boat voyage following the mutiny. And yes,
he was an efficient subordinate.
50 But he was not fit for command.
Bligh responded well to the leadership of a man such

From SAT Online Course CUUS


2007 May Sunday SAT Answers and Explanations

as Captain James Cook*; but after Cook's death while


on an expedition, Bligh fell out with most of the other
officers. In the margins of the official account of that
55 expedition, Bligh wrote the phrase "a most inf amous
lie" and other rude comments of much the same nature.
The same aggressiveness and use of gross language
came with him into the Bounty , made all the worse by
his having no superiors or even equals in rank on board.
60 His authority was final and could not be challenged. It
is said that only two weeks into a voyage expected to
last several years Bligh was scarcely on speaking terms
with his officers.
The mutiny itself is sometimes put down to the charms
65 of Tahiti, but the charms were the same for Wallis's men,
for Cook's, and for Bougainv ille's, and these earlier cap-
tains had no very grave problems. The trouble seems to
be that Bligh lacked natural authority and tried to make
up for it by railing and cursing. During that terrible open-
70 boat voyage, he quarreled steadily with the Bounty 's loyal
carpenter, and when they reached home Bligh had the
carpenter brought before a court-martial for disobedience
and disrespect.
There was another court-martial some years later
75 when Bligh, as captain of the Warrior , was accused of
tyrannous, oppressive, and unofficer-like behavior to
Lieutenant Frazier and that ship's other officers. The
evidence showed that Bligh was a foul-mouthed bully,
and the court found the charges "in part proved." Bligh
80 was reprimanded.

7 In the opening paragraph of Passage 1, the mood Bligh describes is primarily one
of

(A) intense indignation


(B) wary pragmatism
(C) cheerful forgiveness
(D) optimistic confidence
(E) patriotic pride

ANSWERS AND EXPLA NATIONS

Explanation for Correct Answer D :

Choice (D) is correct. Someone who is optimistic and confident anticipates the good
outcome of a situation and has a feeling of self-assurance. In the first paragraph of Pas

From SAT Online Course CUUS


2007 May Sunday SAT Answers and Explanations

sage 1, Bligh reflects on the mutiny that took place on the Bounty , describing his
inward satisfaction and a feeling that his mind was wonderfully comforted. He says
that, despite the disastrous event he had experienced, he is confident that he will be
able to explain and justify what has happened to his superiors and fellow citizens: I
began to conceive hopes, notwithstanding so heavy a calamity, that I should one day
be able to account to my King and country for the misfortune. Bligh clearly conveys a
mood of optimistic confidencehe is certain that the best possible outcome will occur.

Explanation for Incorrect Answer A :

Choice (A) is incorrect. Indignation is anger aroused by something unjust, unworthy, or


mean. Bligh may have felt indignation as a result of the mutineers actions, but he does
not convey any indignation in the first paragraph of Passage 1. As Bligh reflects on the
mutiny that has just befallen him, he is confident that he will be able to explain and
justify what has happened to his superiors and fellow citizens: I began to conceive
hopes, notwithstanding so heavy a calamity, that I should one day be able to account
to my King and country for the misfortune. Rather than seeming intensely indignant,
or very angry because of something unjust, Bligh seems optimistic and confidenthe is
certain that the best possible outcome will occur.

Explanation for Incorrect Answer B :

Choice (B) is incorrect. Pragmatism is a practical way of approaching or solving


problems. It could be said that in the first paragraph of Passage 1 Bligh approaches the
events of the mutiny on the Bounty pragmaticallyhe concentrates on how to deal with
the repercussions and how to ensure the best possible outcome, rather than dwelling
on what has happened or what further difficulties he may face. However, Bligh does not
seem to be wary, or cautious, in any way. On the contrary, he seems confident (he is
hopeful that he will one day be able to account to [his] King and country for the
misfortune of the mutiny) and optimistic (he has an inward satisfaction and his mind
is wonderfully comforted).

Explanation for Incorrect Answer C :

Choice (C) is incorrect. In the first paragraph of Passage 1, Bligh reflects on the mutiny
that took place on the Bounty , describing his inward satisfaction and a feeling that his
mind was wonderfully comforted. He says that, despite the disastrous event he had
experienced, he is confident that he will be able to explain and justify what has
happened to his superiors and fellow citizens: I began to conceive hopes,
notwithstanding so heavy a calamity, that I should one day be able to account to my
King and country for the misfortune. Bligh hopes for the best, but the mood he
describes is not quite cheerful, or merry and full of good spirits. Further, although Bligh
seems to hope that he will be forgiven for the Bounty s misfortune, he does not discuss
forgiving the mutineers his mood is not one of forgiveness.

From SAT Online Course CUUS


2007 May Sunday SAT Answers and Explanations

Explanation for Incorrect Answer E :

Choice (E) is incorrect. Patriotic pride can be described as a feeling of love and devotion
to ones country. Bligh does mention his King and country in the first paragraph of
Passage 1, but his concern is being able to explain and justify what has happened to his
superiors and his fellow citizens: I began to conceive hopes, notwithstanding so heavy
a calamity, that I should one day be able to account to my King and country for the
misfortune. As he reflects on the mutiny that took place on the Bounty , Blighs mood is
not one of patriotic pride; rather, Bligh is confidently optimistic, feeling comforted
because he is certain that the best possible outcome will occur.

8 The satisfaction mentioned by Bligh in line 2 of Passage 1 is based primarily on


his sense of the

(A) loyalty demonstrated by his friends


(B) feasibility of travel in the open boat
(C) correctness of his own conduct
(D) inevitability of punishment for the mutineers
(E) adventurous aspect of his situation

ANSWERS AND EXPLA NATIONS

Explanation for Correct Answer C :

Choice (C) is correct. In line 2, Bligh describes feeling an inward satisfaction despite
the mutiny that had occurred on the Bounty . This satisfaction is based primarily on
Blighs sense of the correctness of his own conduct. He believes he did nothing wrong,
nothing that would lead his crew to overthrow him as captain: With full consciousness
of my integrity, I found my mind wonderfully comforted. In other words, Bligh feels
satisfied that he adhered to, or faithfully followed, a code of moral values and was not
responsible for the actions of the mutineers.

Explanation for Incorrect Answer A :

Choice (A) is incorrect. The inward satisfaction Bligh describes in line 2 is certainly
not based on any sort of loyalty, or faithfulness and devotion, demonstrated by his
friends. The satisfaction he describes came shortly after the crew of the Bounty carried
out a mutiny after the crew revolted and overthrew Bligh as their captain. This act of
rebellion does not demonstrate loyalty in any way, and the crew members were not
Blighs friends. The satisfaction to which Bligh refers is based on his sense of the
correctness of his own behavior: With full consciousness of my integrity, I found my
mind wonderfully comforted.

Explanation for Incorrect Answer B :

From SAT Online Course CUUS


2007 May Sunday SAT Answers and Explanations

Choice (B) is incorrect. The author of Passage 2 indicates that Bligh accomplished an
astonishing open-boat voyage after the mutiny on the Bounty , but Bligh does not
mention this voyage in Passage 1 or suggest that his inward satisfaction is based on
his sense of the feasibility of travel in the open boat. Rather, Blighs satisfaction is
based on his sense of the correctness of his behavior on the Bounty : With full
consciousness of my integrity, I found my mind wonderfully comforted.

Explanation for Incorrect Answer D :

Choice (D) is incorrect. In line 2, Bligh describes feeling an inward satisfaction despite
the mutiny that had occurred on the Bounty . This satisfaction is not based on his sense
of the inevitability of punishment for the mutineers; nowhere in the passage does Bligh
discuss the likelihood of the crew members being punished for the mutiny. The
satisfaction to which Bligh refers is based on his sense of the correctness of his
behavior on the Bounty : With full consciousness of my integrity, I found my mind
wonderfully comforted.

Explanation for Incorrect Answer E :

Choice (E) is incorrect. In line 2, Bligh describes feeling an inward satisfaction despite
the mutiny that had occurred on the Bounty . This satisfaction is not based on his sense
of the adventurous aspect of his situation. Being cast off from a ship and left in a small
open boat with few supplies would likely present adventure, but Bligh does not discuss
this aspect of his situation or indicate that a sense of adventure gave him satisfaction.
Rather, the satisfaction to which Bligh refers is based on his sense of the correctness of
his behavior on the Bounty : With full consciousness of my integrity, I found my mind
wonderfully comforted.

9 The parenthetical comment in lines 22-23 of Passage 1 can best be characterized


as

(A) apologetic
(B) cynical
(C) flippant
(D) defensive
(E) insolent

ANSWERS AND EXPLA NATIONS

Explanation for Correct Answer D :

Choice (D) is correct. In the first paragraph of Passage 1 Bligh states that he believes
he did nothing to cause the mutiny on the Bounty : With full consciousness of my
integrity, I found my mind wonderfully comforted. Throughout the rest of the passage,

From SAT Online Course CUUS


2007 May Sunday SAT Answers and Explanations

Bligh attempts to explain why the mutineers chose to revolt, constantly turning any
blame away from himself. He does admit that it is now perhaps not so much to be
wondered at . . . that a set of sailors, most without influential connections in England,
should be led awaythat in hindsight, it is not surprising that a group of sailors who
were not in a good position in England would want to stay in Tahiti. But Bligh includes
the parenthetical comment that it was scarcely possible to have foreseen that the
sailors would mutiny in order to stay in Tahitithat it would have been almost
impossible for anyone to predict such an action. This comment is defensive; in an
attempt to prevent attacks on his behavior or accusations that he made a mistake,
Bligh asserts that nobody would have predicted the mutiny.

Explanation for Incorrect Answer A :

Choice (A) is incorrect. In the first paragraph of Passage 1 Bligh states that he believes
he did nothing to cause the mutiny on the Bounty : With full consciousness of my
integrity, I found my mind wonderfully comforted. Throughout the rest of the passage,
Bligh attempts to explain why the mutineers chose to revolt, constantly turning any
blame away from himself. He does admit that it is now perhaps not so much to be
wondered at . . . that a set of sailors, most without influential connections in England,
should be led away that in hindsight, it is not surprising that a group of sailors who
were not in a good position in England would want to stay in Tahiti. But Bligh includes
the parenthetical comment that it was scarcely possible to have foreseen that the
sailors would mutiny in order to stay in Tahitithat it would have been almost
impossible for anyone to predict such an action. This comment is not apologetic; Bligh
does not feel that he did anything wrong, so he is not apologizing for anything. Rather,
he is trying to prevent accusations that he made a mistake by asserting that nobody
would have predicted the mutiny.

Explanation for Incorrect Answer B :

Choice (B) is incorrect. In the first paragraph of Passage 1 Bligh states that he believes
he did nothing to cause the mutiny on the Bounty : With full consciousness of my
integrity, I found my mind wonderfully comforted. Throughout the rest of the passage,
Bligh attempts to explain why the mutineers chose to revolt, constantly turning any
blame away from himself. He does admit that it is now perhaps not so much to be
wondered at . . . that a set of sailors, most without influential connections in England,
should be led awaythat in hindsight, it is not surprising that a group of sailors who
were not in a good position in England would want to stay in Tahiti. But Bligh includes
the parenthetical comment that it was scarcely possible to have foreseen that the
sailors would mutiny in order to stay in Tahitithat it would have been almost
impossible for anyone to predict such an action. Bligh may have been cynical, or
distrustful of human nature and motives, after his crew overthrew him, but the
parenthetical comment is not a cynical statement. Rather, the statement is defensive;
in an attempt to prevent accusations that he made a mistake, Bligh asserts that nobody
would have predicted the mutiny.

From SAT Online Course CUUS


2007 May Sunday SAT Answers and Explanations

Explanation for Incorrect Answer C :

Choice (C) is incorrect. In the first paragraph of Passage 1 Bligh states that he believes
he did nothing to cause the mutiny on the Bounty : With full consciousness of my
integrity, I found my mind wonderfully comforted. Throughout the rest of the passage,
Bligh attempts to explain why the mutineers chose to revolt, constantly turning any
blame away from himself. He does admit that it is now perhaps not so much to be
wondered at . . . that a set of sailors, most without influential connections in England,
should be led awaythat in hindsight, it is not surprising that a group of sailors who
were not in a good position in England would want to stay in Tahiti. But Bligh includes
the parenthetical comment that it was scarcely possible to have foreseen that the
sailors would mutiny in order to stay in Tahitithat it would have been almost
impossible for anyone to predict such an action. This comment is not flippant, or glib
and not serious; Bligh seems to be serious when, in an attempt to prevent accusations
that he made a mistake, he asserts that nobody would have predicted the mutiny.

Explanation for Incorrect Answer E :

Choice (E) is incorrect. In the first paragraph of Passage 1 Bligh states that he believes
he did nothing to cause the mutiny on the Bounty : With full consciousness of my
integrity, I found my mind wonderfully comforted. Throughout the rest of the passage,
Bligh attempts to explain why the mutineers chose to revolt, constantly turning any
blame away from himself. He does admit that it is now perhaps not so much to be
wondered at . . . that a set of sailors, most without influential connections in England,
should be led awaythat in hindsight, it is not surprising that a group of sailors who
were not in a good position in England would want to stay in Tahiti. But Bligh includes
the parenthetical comment that it was scarcely possible to have foreseen that the
sailors would mutiny in order to stay in Tahitithat it would have been almost
impossible for anyone to predict such an action. This comment is not insolent, or
insultingly contemptuous; Bligh is not saying that he despises anything or trying to
insult anyone. Rather, in an attempt to prevent accusations that he made a mistake,
Bligh simply asserts that nobody would have predicted the mutiny.

10 In line 36, Blighs choice of the word sacrifice most directly reveals his

(A) anxiety about punishment for having lost a naval ship


(B) willingness to place great importance on his duties
(C) desire to be perceived as an innocent victim
(D) eagerness to portray his crew as reckless and impulsive
(E) fear of future repercussions from relatives of the mutineers

ANSWERS AND EXPLA NATIONS

Explanation for Correct Answer C :

From SAT Online Course CUUS


2007 May Sunday SAT Answers and Explanations

Choice (C) is correct. In this context, a "sacrifice" is someone or something that is


harmed for the sake of something else. As Bligh attempts to answer the question,
what could be the reason for such a revolt? he claims that the mutineers had hopes
of a more happy life among the Tahitians and that local Tahitian chiefs had encouraged
the mutineers to stay there permanently, making promises of large possessions. Bligh
uses the word sacrifice to suggest that the mutineers overthrew him for the sake of
living a better life than they were living in England. Bligh is clearly portraying himself as
an innocent victimas someone who did nothing wrong but was harmed nonetheless.

Explanation for Incorrect Answer A :

Choice (A) is incorrect. There is no indication that Bligh feels any anxiety, or fearful
concern, about being punished for having lost a naval ship, or that his reference to
himself as a "sacrifice"someone or something that is harmed for the sake of
something elsereveals any such anxiety. In fact, Bligh describes feeling an inward
satisfaction because he is hopeful that he will one day be able to account to [his] King
and country for the misfortune. Bligh uses the word sacrifice to suggest that the
mutineers overthrew him for the sake of living a better life than they were living in
England. Bligh is portraying himself as an innocent victimas someone who did nothing
wrong but was harmed nonetheless.

Explanation for Incorrect Answer B :

Choice (B) is incorrect. Although Bligh does seem to place great importance on his
duties as a captain, his use of the word sacrifice does not reveal his willingness to
place importance on his duties; Bligh is not discussing the importance of his duties.
Rather, Bligh uses the word sacrifice to portray himself as an innocent victimas
someone who did nothing wrong but was harmed nonetheless. Bligh claims that the
mutineers had hopes of a more happy life among the Tahitians and that they
overthrew him for the sake of living a better life than they were living in England.

Explanation for Incorrect Answer D :

Choice (D) is incorrect. Rather than suggesting that the crew was reckless and
impulsive, Bligh describes the mutiny as a close-planned act of villainyan act that
was carefully planned. His use of the word sacrifice does not reveal his eagerness to
portray the crew as reckless, but rather his desire to be seen as an innocent victimas
someone who did nothing wrong but was harmed nonetheless. Bligh claims that the
mutineers had hopes of a more happy life among the Tahitians and that they
overthrew him for the sake of living a better life than they were living in England.

Explanation for Incorrect Answer E :

Choice (E) is incorrect. There is no indication that Bligh fears future repercussions from
relatives of the mutineers, or that his reference to himself as a sacrificesomeone or s

From SAT Online Course CUUS


2007 May Sunday SAT Answers and Explanations

omething that is harmed for the sake of something elsereveals any such fear. In fact,
the mutineers relatives are not mentioned at all. Bligh uses the word sacrifice to
portray himself as an innocent victimas someone who did nothing wrong but was
harmed nonetheless. Bligh claims that the mutineers had hopes of a more happy life
among the Tahitians and that they overthrew him for the sake of living a better life
than they were living in England.

11 In Passage 1, Bligh presents himself as trying to

(A) defend a controversial decision


(B) revisit a seemingly settled dispute
(C) admit to a catastrophic mistake
(D) condemn a ruthless crime
(E) make sense of a puzzling event

ANSWERS AND EXPLA NATIONS

Explanation for Correct Answer E :

Choice (E) is correct. A puzzling event is one that is difficult to understand. For Bligh,
the mutiny is a puzzling eventhe has trouble understanding why it happened,
because everything seemed to be progressing smoothly during their voyage: A few
hours earlier, my situation had been peculiarly flattering. The voyage had been two-
thirds completed, to all appearance, in every promising way. Throughout Passage 1,
Bligh tries to answer the question what could be the reason for such a revolt? Bligh
states that he can only conjecturecan only guessthat the mutineers had hopes of
a more happy life among the Tahitians. Underlying this self-presentation, of course, is
Bligh's need to defend himself from charges that the mutiny was his fault or that he
should have seen it coming. To Bligh, the mutiny was "scarcely possible to have
foreseen."

Explanation for Incorrect Answer A :

Choice (A) is incorrect. Throughout Passage 1, Bligh tries to answer the question what
could be the reason for such a revolt?what led to the mutiny on the Bounty ? Trying
to make sense of the puzzling event, Bligh can only conjecturecan only guessthat
the mutineers had hopes of a more happy life among the Tahitians. The mutineers
decision to overthrow their captain may have caused controversy, but Bligh is certainly
not trying to defend, or justify, their decision. And Bligh is not trying to defend any
decision he made, controversial or otherwise; he does not mention any decisions he
made.

Explanation for Incorrect Answer B :

From SAT Online Course CUUS


2007 May Sunday SAT Answers and Explanations

Choice (B) is incorrect. Throughout Passage 1, Bligh tries to answer the question what
could be the reason for such a revolt?what led to the mutiny on the Bounty ? Bligh
can only conjecturecan only guessthat the mutineers had hopes of a more happy
life among the Tahitians. He is not trying to revisit a seemingly settled dispute, or
disagreement; indeed, there is no indication that any dispute related to the mutiny has
been settled. Rather, Bligh is trying to make sense of an event that even he finds
puzzling. Bligh believed there was no conflict between him and his crew, and he
suggests that it was scarcely possible to have foreseen that the mutiny would occur.

Explanation for Incorrect Answer C :

Choice (C) is incorrect. Throughout Passage 1, Bligh tries to answer the question what
could be the reason for such a revolt?what led to the mutiny on the Bounty ? Bligh
can only conjecturecan only guessthat the mutineers had hopes of a more happy
life among the Tahitians. Bligh is not trying to admit to a catastrophic, or tragic,
mistake. On the contrary, Bligh seems certain that he behaved with integrity on the B
ounty and that he was not responsible for the mutiny; he is trying to make sense of the
mutiny, an event that he claims to find puzzling.

Explanation for Incorrect Answer D :

Choice (D) is incorrect. Throughout Passage 1, Bligh tries to answer the question what
could be the reason for such a revolt?what led to the mutiny on the Bounty ? Trying
to make sense of the puzzling event, Bligh can only conjecturecan only guessthat
the mutineers had hopes of a more happy life among the Tahitians. Bligh might
consider the mutineers actions to be a ruthless, or cruel, crime; however, Blighs
intention seems to be to try to explain what caused the mutinyan event that he
claims to find puzzling and not to condemn, or criticize, the mutineers actions.

12 In line 64, charms most nearly means

(A) ornaments
(B) attractions
(C) incantations
(D) deceptions
(E) novelties

ANSWERS AND EXPLA NATIONS

Explanation for Correct Answer B :

Choice (B) is correct. In lines 64-65 the author of Passage 2 states that the mutiny on
the Bounty is sometimes put down to the charms of Tahiti. In this context, the term
charms most nearly means attractions, or things that attract people by appealing to

From SAT Online Course CUUS


2007 May Sunday SAT Answers and Explanations

their desires or tastes. The author suggests that some people think the mutiny
occurred because Tahiti, and the thought of living there, was so appealing to the Bounty
crew that the sailors did not want to leave.

Explanation for Incorrect Answer A :

Choice (A) is correct. In lines 64-65 the author of Passage 2 states that the mutiny on
the Bounty is sometimes put down to the charms of Tahiti. In some contexts the term
charm can refer to an ornament, or an accessory that lends grace or beauty to
something. But the author of Passage 2 does not use the term charms to suggest that
Tahiti had physical accessories ornamenting it. Rather, the author uses the term
charms to refer to the attractions of Tahitithe aspects of Tahiti that appealed to the
sailors desires or tastes. The author suggests that some people think the mutiny
occurred because Tahiti was so appealing that the Bounty crew did not want to leave.

Explanation for Incorrect Answer C :

Choice (C) is incorrect. In lines 64-65 the author of Passage 2 states that the mutiny on
the Bounty is sometimes put down to the charms of Tahiti. In some contexts, the term
charm can refer to an incantation, or a written or recited formula of words designed
to produce a particular effect. But the author of Passage 2 does not use the term
charms to suggest that the Bounty crew encountered any incantations in Tahiti. Rather,
the author uses the term charms to refer to the attractions of Tahitithe aspects of
Tahiti that appealed to the sailors desires or tastes. The author suggests that some
people think the mutiny occurred because Tahiti was so appealing that the Bounty crew
did not want to leave.

Explanation for Incorrect Answer D :

Choice (D) is incorrect. In lines 64-65 the author of Passage 2 states that the mutiny
on the Bounty is sometimes put down to the charms of Tahiti. In some contexts, to
charm someone can mean to deceive or trick that person. But the author of Passage 2
does not use the term charms to suggest that the Bounty crew members were in any
way deceived in Tahiti. Rather, the author uses the term charms to refer to the
attractions of Tahiti the aspects of Tahiti that appealed to the sailors desires or tastes.
The author suggests that some people think the mutiny occurred because Tahiti was so
appealing that the Bounty crew did not want to leave.

Explanation for Incorrect Answer E :

Choice (E) is incorrect. In lines 64-65 the author of Passage 2 states that the mutiny on
the Bounty is sometimes put down to the charms of Tahiti. It could have been the new
or unusual aspects of Tahitian life (or "novelties") that the sailors found charming, but,
as Bligh suggests, it is more likely that they were charmed by "the much admired
indigenous women" and the "promises of large possessions." The author uses the term

From SAT Online Course CUUS


2007 May Sunday SAT Answers and Explanations

charms to refer to the attractions of Tahitithe aspects of Tahiti that appealed to the
sailors desires or tastes. The author suggests that some people think the mutiny
occurred because Tahiti was so appealing that the Bounty crew did not want to leave.

13 In Passage 2, the story about the carpenter (lines 69-73) primarily serves to

(A) illustrate one of Blighs personality traits


(B) suggest an alternative view of Blighs skills as a seaman
(C) introduce a revealing episode in maritime history
(D) provide an example of Blighs respect for military justice
(E) explain an apparent contradiction in Blighs nature

ANSWERS AND EXPLA NATIONS

Explanation for Correct Answer A :

Choice (A) is correct. In lines 68-69, the author of Passage 2 claims that Bligh lacked
natural authority and tried to make up for it by railing and cursingthat is, by scolding
in harsh or abusive language. The author then illustrates this personality trait by
relating the story about the carpenter, stating that Bligh quarreled steadily with the Bo
unty s loyal carpenter . . . and had the carpenter brought before a court-martial for
disobedience and disrespect.

Explanation for Incorrect Answer B :

Choice (B) is incorrect. The author of Passage 2 never questions Blighs skills as a
seaman; in fact, he or she states that Bligh was a courageous and eminently capable
seaman. Rather than serving to suggest an alternative view of Blighs skills as a
seaman, the story about the carpenter illustrates Blighs tendency to make up for his
lack of natural authority by railing and cursingthat is, by scolding in harsh or
abusive language. Bligh quarreled steadily with the Bounty s loyal carpenter . . . and
had the carpenter brought before a court-martial for disobedience and disrespect.

Explanation for Incorrect Answer C :

Choice (C) is incorrect. The author of Passage 2 does not indicate that any event in
particular is a revealing episode in maritime history, and he or she does not relate the
story about the carpenter to introduce any such episode. Rather, the story about the
carpenter illustrates Blighs tendency to make up for his lack of natural authority by
railing and cursingthat is, by scolding in harsh or abusive language. Bligh
quarreled steadily with the Bounty s loyal carpenter . . . and had the carpenter brought
before a court-martial for disobedience and disrespect.

Explanation for Incorrect Answer D :

From SAT Online Course CUUS


2007 May Sunday SAT Answers and Explanations

Choice (D) is incorrect. The author of Passage 2 never indicates that Bligh had any
particular respect for military justice, although he once brought someone before a
court-martial. Rather than serving to provide an example of Blighs respect for military
justice, the story about the carpenter illustrates Blighs tendency to make up for his
lack of natural authority by railing and cursingthat is, by scolding in harsh or
abusive language. Bligh quarreled steadily with the Bounty s loyal carpenter . . . and
had the carpenter brought before a court-martial for disobedience and disrespect.

Explanation for Incorrect Answer E :

Choice (E) is incorrect. The author of Passage 2 never suggests that there was any
contradiction in Blighs nature. He or she portrays Bligh as a man who was an efficient
subordinate but was not fit for command, and as a man who consistently behaved in
a harsh manner when he lacked natural authority. The story about the carpenter
illustrates Blighs tendency to make up for his lack of natural authority by railing and
cursingthat is, by scolding in harsh or abusive language; Bligh quarreled steadily
with the Bounty s loyal carpenter . . . and had the carpenter brought before a court-
martial for disobedience and disrespect.

14 Passage 2 implies that which change to the Bounty expedition might have
prevented the mutiny?

(A) A more equitable use of punishment by Bligh


(B) The complete isolation of British sailors from Tahitian leaders
(C) The presence of other high-ranking officers on board
(D) Some regularly scheduled intervals of shore leave during the voyage
(E) The inclusion of some of Cooks former staff among the crew

ANSWERS AND EXPLA NATIONS

Explanation for Correct Answer C :

Choice (C) is correct. The author of Passage 2 states that Bligh treated the crew of the
Bounty with aggressiveness and use of gross language, and that his behavior was
made all the worse by his having no superiors or even equals in rank on boardthe
author suggests that Blighs treatment of the crew likely led to the mutiny. However,
the author also notes that Bligh was an efficient subordinate and had at one time
responded well to the leadership of a captain; these comments suggest that Blighs
behavior might have been kept in check, and the mutiny might have been prevented, if
other high-ranking officers had been on board the Bounty .

Explanation for Incorrect Answer A :

Choice (A) is incorrect. The author of Passage 2 does not discuss Blighs use of punishm

From SAT Online Course CUUS


2007 May Sunday SAT Answers and Explanations

ent during the Bounty expedition; he or she mentions one incident that could be seen
as punishment, but it is an event that took place after the expedition was overBligh
brought the Bounty s carpenter before a court-martial when they reached home.
Although the author notes that Blighs authority was final and could not be
challenged, he or she does not imply that Blighs use of punishment during the
expedition was not equitable, or fair and equal, or that the mutiny might have been
prevented if Blighs punishments had been more equitable.

Explanation for Incorrect Answer B :

Choice (B) is incorrect. The author of Passage 2 notes that the mutiny itself is
sometimes put down to the charms of Tahitithat is, some say the mutiny happened
because the crew wanted to stay in Tahiti. However, the author suggests that the
charms of Tahiti were not responsible for the mutiny; other crews (Walliss . . . Cooks
. . . Bougainvilles) had been to Tahiti and had not risen up against their captains.
Further, unlike the author Passage 1, who points out that the local chiefs . . .
encouraged [the crews] staying permanently and even made . . . promises of large
possessions, the author of Passage 2 does not mention interactions between British
sailors and Tahitian leaders. Therefore, there is no indication that the complete isolation
of the sailors from Tahitian leaders might have prevented the mutiny.

Explanation for Incorrect Answer D :

Choice (D) is incorrect. The author of Passage 2 does not discuss the importance of
shore leave during a voyage or suggest that the mutiny might have been prevented if
there had been regularly scheduled intervals of shore leave during the Bounty expeditio
n. The author suggests that some people would argue that shore leave actually causedt
he mutinythat the charms of Tahiti were to blamebut he or she disagrees, noting
that other crews (Walliss . . . Cooks . . . Bougainvilles) had been to Tahiti and not
risen up against their captains.

Explanation for Incorrect Answer E :

Choice (E) is incorrect. The author of Passage 2 states that Bligh was an efficient
subordinate and had at one time responded well to the leadership
of . . . Captain James Cook. However, he or she also states that after Cooks
death . . . Bligh fell out with most of the other officers. The author does not suggest
that the inclusion of some of Cooks former staffpeople with whom Bligh did not get
alongamong the crew of the Bounty might have prevented the mutiny.

15 In contrast to the author of Passage 2, Bligh suggests in Passage 1 that the


mutineers acted primarily out of

(A) desire for power

From SAT Online Course CUUS


2007 May Sunday SAT Answers and Explanations

(B) contempt for authority


(C) despair
(D) loneliness
(E) self-interest

ANSWERS AND EXPLA NATIONS

Explanation for Correct Answer E :

Choice (E) is correct. The author of Passage 2 suggests that Blighs behavior toward the
crew caused the mutiny: Bligh was not fit for command and treated the crew with
aggressiveness and use of gross language. But Bligh suggests in Passage 1 that the
mutineers acted primarily out of self-interestthat the mutineers were concerned with
their own wants and needs. Bligh speculates that the mutineers hoped for a more
happy life among the Tahitians than they could possibly enjoy in England and had
developed sentimental ties with Tahitian women. He also suggests that Tahitian chiefs
may have encouraged [the crews] staying permanently by making promises of large
possessions.

Explanation for Incorrect Answer A :

Choice (A) is incorrect. The author of Passage 2 suggests that Blighs behavior toward
the crew caused the mutiny: Bligh was not fit for command and treated the crew with
aggressiveness and use of gross language. But Bligh suggests in Passage 1 that the
reason for such a revolt was the crews self-interestthe mutineers concern with their
own wants and needs. Bligh speculates that the crew rebelled because they hoped for
a more happy life among the Tahitians than they could possibly enjoy in England,
because they had developed sentimental ties with Tahitian women, and because
Tahitian chiefs may have encouraged their staying permanently by making promises
of large possessions ; he does not indicate that the crew wanted power.

Explanation for Incorrect Answer B :

Choice (B) is incorrect. The author of Passage 2 suggests that the mutineers acted out
of contempt for authority contempt for their captain, William Bligh. Bligh, however,
suggests in Passage 1 that the reason for such a revolt was the crews self-
interestthe mutineers concern with their own wants and needs. Bligh speculates that
the crew rebelled because they hoped for a more happy life among the Tahitians than
they could possibly enjoy in England, because they had developed sentimental ties
with Tahitian women, and because Tahitian chiefs may have encouraged their staying
permanently by making promises of large possessions. Bligh certainly does not
consider the possibility that the crew had any contempt for his authority; at the time,
he even believed he was on most friendly terms with . . . the leader of the mutineers.

Explanation for Incorrect Answer C :

From SAT Online Course CUUS


2007 May Sunday SAT Answers and Explanations

Choice (C) is incorrect. The author of Passage 2 suggests that Blighs behavior toward
the crew caused the mutiny: Bligh was not fit for command and treated the crew with
aggressiveness and use of gross language. But Bligh suggests in Passage 1 that the
reason for such a revolt was the crews self-interestthe mutineers concern with their
own wants and needs (a more happy life among the Tahitians, large possessions).
Bligh does not indicate that the crew acted out of despair. On the contrary, he writes
that if the mutiny had been the result of grievances, he would have discovered
symptoms of [the mutineers ] discontentand that the case was far otherwise.

Explanation for Incorrect Answer D :

Choice (D) is incorrect. The author of Passage 2 suggests that Blighs behavior toward
the crew caused the mutiny: Bligh was not fit for command and treated the crew with
aggressiveness and use of gross language. But Bligh suggests in Passage 1 that the
reason for such a revolt was the crews self-interestthe mutineers concern with their
own wants and needs (a more happy life among the Tahitians, large possessions). It
is possible that the crew members formed sentimental ties with Tahitian women and
hoped to live among the Tahitians because they were lonely, but Bligh does not address
this possibility or suggest that the mutineers acted out of loneliness.

16 In contrast to Passage 1, Passage 2 suggests that the primary motivation of the


mutineers was

(A) exasperation with Bligh


(B) nostalgia for Tahiti
(C) frustration with their long journey
(D) fear of British authorities
(E) contempt for Blighs use of favoritism

ANSWERS AND EXPLA NATIONS

Explanation for Correct Answer A :

Choice (A) is correct. The author of Passage 2 suggests that the mutineers motivation
was exasperation with their captain, William Bligh. He or she states that Bligh was not
fit for command, describing him as having an unpleasant character, being aggressive,
and using gross language; the author suggests that Blighs behavior was the
trouble behind the mutiny. In contrast, in Passage 1, Bligh does not consider
exasperation as a cause of the mutiny; he suggests that the mutineers acted out of
hope for a more happy life among the Tahitians than they could possibly enjoy in
England.

Explanation for Incorrect Answer B :

From SAT Online Course CUUS


2007 May Sunday SAT Answers and Explanations

Choice (B) is incorrect. It is the author of Passage 1, not the author of Passage 2, who
suggests that the mutineers primary motivation was a desire to stay in Tahiti. The
author of Passage 2 mentions that the mutiny itself is sometimes put down to the
charms of Tahitithat is, that some say Tahitis charms are to blamebut suggests
that the crews affection for Tahiti was not the cause of the mutiny. The author explains
that the crews of other ships experienced the same charms and did not mutiny,
suggesting that some other factor likely caused the crew to rebel. The author indicates
that exasperation with Bligh, a captain who was not fit for command, was the
mutineers primary motivation.

Explanation for Incorrect Answer C :

Choice (C) is incorrect. The author of Passage 2 notes that the Bounty expedition was
expected to last several years, but he or she does not suggest that the mutineers
were frustrated with their long journey. Rather, the author suggests that frustration and
exasperation with Bligh, a captain who was not fit for command, was the mutineers
primary motivation; Bligh had an unpleasant character, was aggressive, and used
gross language.

Explanation for Incorrect Answer D :

Choice (D) is incorrect. It is the author of Passage 1, not the author of Passage 2, who
might be said to hint at fear of authorities in Passage 1, Bligh notes that the crew
knew desertion was unsafe because it was in the captains power to get the
deserters back. However, there is no suggestion in either Passage 1 or Passage 2 that
fear of British authorities was the mutineers primary motivation. Bligh, the author of
Passage 1, suggests that the crew acted out of hope for a more happy life among the
Tahitians than they could possibly enjoy in England, while the author of Passage 2
suggests that the crew acted out of exasperation with Bligh, a captain who was not fit
for command.

Explanation for Incorrect Answer E :

Choice (E) is incorrect. There is no indication in either Passage 1 or Passage 2 that


contempt for Blighs use of favoritism was the mutineers primary motivation; in fact,
neither Bligh nor the author of Passage 2 suggests that Bligh used favoritism. Instead,
Bligh, the author of Passage 1, suggests that the crew acted out of hope for a more
happy life among the Tahitians than they could possibly enjoy in England, while the
author of Passage 2 suggests that the crew acted out of exasperation with Bligh, a
captain who was not fit for command.

17 Which additional information provided by Passage 2 is most useful in evaluating


the persuasiveness of the argument made in Passage 1?

From SAT Online Course CUUS


2007 May Sunday SAT Answers and Explanations

(A) Blighs bravery and skill were beyond dispute .


(B) Bligh had no superiors or equals in rank on board the Bounty .
(C) An otherwise loyal sailor from the Bounty behaved disrespectfully .
(D) Bligh was officially punished for his behavior on a subsequent voyage.
(E) Bligh had gotten access to a logbook on another expedition.

ANSWERS AND EXPLA NATIONS

Explanation for Correct Answer D :

Choice (D) is correct. In Passage 1, Bligh indicates that he was conscious of his
integrity his firm adherence to a code of valuesand, as a result, felt an inward
satisfaction after the mutiny on the Bounty . Blighs statements suggest that he
believed it was not his behavior and actions that were to blame for the mutiny. Rather,
Bligh claims that the crews desire to stay among the Tahitians led to the mutiny. The
author of Passage 2, however, suggests that Blighs behavior did lead to the mutiny and
that Bligh was not fit for command. He or she supports these claims by explaining
that Bligh was officially punished for his behavior as captain on a subsequent voyage:
Bligh was accused of tyrannous, oppressive, and unofficer-like behavior on the Warrio
r, and was reprimanded. This information is helpful in evaluating the persuasiveness
of the argument made in Passage 1because Bligh was punished for his behavior on
another voyage, we cannot accept at face value Blighs claim in Passage 1 that he acted
with complete integrity on the Bounty .

Explanation for Incorrect Answer A :

Choice (A) is incorrect. The author of Passage 2 does acknowledge that Bligh was a
courageous and eminently capable seamanthat he had bravery and skill. But this
information is not very useful in evaluating the persuasiveness of the argument made
in Passage 1. In Passage 1, Bligh argues that he acted with integrity that he firmly
adhered to a code of valueson the Bounty , and that the mutineers acted out of self-
interest. Bligh suggests that his behavior and actions were not to blame for the mutiny;
his argument has nothing to do with his skill or bravery.

Explanation for Incorrect Answer B :

Choice (B) is incorrect. The author of Passage 2 does indicate that Bligh had no
superiors or equals in rank on board the Bounty he or she states that Blighs behavior
was made all the worse by his having no superiors or even equals in rank on board.
But this information is not very useful in evaluating the persuasiveness of the argument
made in Passage 1. In Passage 1, Bligh argues that he acted with integrity that he
firmly adhered to a code of valueson the Bounty , and that the mutineers acted out of
self-interest. Bligh suggests that his behavior and actions were not to blame for the
mutiny; his argument has nothing to do with having no superiors or officers equal in
rank on board the Bounty .

From SAT Online Course CUUS


2007 May Sunday SAT Answers and Explanations

Explanation for Incorrect Answer C :

Choice (C) is incorrect. The author of Passage 2 does refer to the Bounty s loyal
carpenter, who Bligh brought before a court-martial for disobedience and disrespect.
However, the author does not indicate that the carpenter did, in fact, behave
disrespectfully; he or she only indicates that Bligh and the carpenter quarreled
steadily and that Bligh accused the carpenter of disrespectful behavior. Even if the
author of Passage 2 asserted that the loyal carpenter was disrespectful, this
information would not be very useful in evaluating the persuasiveness of the argument
made in Passage 1. Although Bligh would likely say that all of the mutineers were
disrespectful, his argument rests on his belief that his own behavior and actions were
not to blame for the mutiny and that the mutineers acted out of self-interest; Blighs
argument has nothing to do with the disrespectful behavior of one sailor.

Explanation for Incorrect Answer E :

Choice (E) is incorrect. Neither Passage 1 nor Passage 2 contains any information about
Bligh having gotten access to a logbook on another expedition. Even if Passage 2
mentioned that Bligh had gotten access to a logbook, this information would not be
very useful in evaluating the persuasiveness of the argument in Passage 1. In Passage
1, Bligh argues that he acted with integrity that he firmly adhered to a code of
valueson the Bounty , and that the mutineers acted out of self-interest. Bligh suggests
that his behavior and actions were not to blame for the mutiny; his argument has
nothing to do with having access to a logbook at any time.

18 Lines 57-63 (Passage 2) suggest that Bligh failed to address his crews
discontent (line 38, Passage 1) for which reason?

(A) He believed that all sailors have grievances, so there was no particular cause for
worry.
(B) He did not suspect that the sailors were disgruntled as they took pains to hide
their feelings.
(C) He knew the sailors were dissatisfied but hoped to win them over in the future.
(D) He sensed problems among the crew yet hesitated to trust his impressions.
(E) He had closed off communication with officers and so did not know the extent of
the crews displeasure.

ANSWERS AND EXPLA NATIONS

Explanation for Correct Answer E :

Choice (E) is correct. In the last paragraph of Passage 1, Bligh indicates that if the crew
members had grievances, he would have known and would have been on his guard.
But Bligh asserts that the case was far otherwisethat the crew showed no signs of b

From SAT Online Course CUUS


2007 May Sunday SAT Answers and Explanations

eing discontent. The author of Passage 2, however, notes in lines 61-63 that only two
weeks into a voyage expected to last several years Bligh was scarcely on speaking
terms with his officers. This suggests that Bligh had closed off communication with his
officers and, as a result, did not know the extent of the crews displeasure.

Explanation for Incorrect Answer A :

Choice (A) is incorrect. In lines 57-63, the author of Passage 2 indicates that Bligh had
closed off communication with his officershe was aggressive and used gross
language, and after only two weeks on the Bounty , was scarcely on speaking terms
with his officers. The author does not indicate in these lines that Bligh believed all
sailors have grievances and that there was no cause for worry on the Bounty .

Explanation for Incorrect Answer B :

Choice (B) is incorrect. In lines 57-63, the author of Passage 2 indicates that Bligh had
closed off communication with his officershe was aggressive and used gross
language, and after only two weeks on the Bounty , was scarcely on speaking terms
with his officers. Because he had closed off communication with his officers, Bligh
might have been unaware that the sailors were disgruntled but the author does not
suggest in lines 57-63 that the sailors took pains to hide their feelings.

Explanation for Incorrect Answer C :

Choice (C) is incorrect. In lines 57-63, the author of Passage 2 indicates that Bligh had
closed off communication with his officershe was aggressive and used gross
language, and after only two weeks on the Bounty , was scarcely on speaking terms
with his officers. It is possible that even without communicating with his officers, Bligh
was aware that some of the sailors were dissatisfied, but this is not indicated in lines
57-63. The author also does not indicate in these lines that Bligh hoped to win the
sailors over in the future.

Explanation for Incorrect Answer D :

Choice (D) is incorrect. In lines 57-63, the author of Passage 2 indicates that Bligh had
closed off communication with his officershe was aggressive and used gross
language, and after only two weeks on the Bounty , was scarcely on speaking terms
with his officers. It is possible that even without communicating with his officers, Bligh
sensed that there were problems among the crew, but this is not indicated in lines 57-
63. The author also does not indicate in these lines that Bligh was hesitant to trust his
impressions.

From SAT Online Course CUUS


2007 May Sunday SAT Answers and Explanations

Section10: Writing

View Answers and Explanations

Online - Practice Test #2

1 Although my brother and I took our dogs to the same training school, my dog won
more awards at obedience trials than my brother.

(A) than my brother


(B) than my brothers dog wins
(C) than my brothers dog
(D) but my brothers dog won fewer awards
(E) but my brother did not

ANSWERS AND EXPLA NATIONS

Explanation for Correct Answer C :

Choice (C) is correct. It avoids the illogical comparison of the original by appropriately
comparing my dog to my brothers dog.

Explanation for Incorrect Answer A :

Choice (A) involves an illogical comparison. It does not make sense to compare my
dog to my brother.

Explanation for Incorrect Answer B :

Choice (B) results in a tense shift. The present-tense verb wins is inconsistent with
the preceding past-tense verb won.

Explanation for Incorrect Answer D :

Choice (D) results in redundancy and unidiomatic phrasing. The two clauses (my dog
won more awards at obedience trials and my brothers dog won fewer awards) give
exactly the same information. In addition, it is not idiomatic to join two similar
statements with the contrastive conjunction but.

Explanation for Incorrect Answer E :

Choice (E) results in vague phrasing and an illogical comparison. The meaning of did
in my brother did not is unclear. In addition, it does not make sense to compare my
dog with my brother.

From SAT Online Course CUUS


2007 May Sunday SAT Answers and Explanations

2 Nancy is a better skier than the rest of us because she has been doing it since she
arrived in Colorado at age five.

(A) she has been doing it


(B) she has been skiing
(C) of learning how
(D) of learning to ski
(E) she has been learning to do it

ANSWERS AND EXPLA NATIONS

Explanation for Correct Answer B :

Choice (B) is correct. It avoids the vague pronoun of the original by providing the verb
skiing.

Explanation for Incorrect Answer A :

Choice (A) involves a vague pronoun. There is nothing in the sentence to which the
singular pronoun it can logically refer.

Explanation for Incorrect Answer C :

Choice (C) results in unidiomatic, vague phrasing. The phrase because of should be
followed by a noun, not the participle learning. Also, there is no main verb to indicate
what was learned. In addition, the verb to learn usually does not imply any kind of
duration; therefore, it is unidiomatic to say that Nancy has been learning how since . . .
age five.

Explanation for Incorrect Answer D :

Choice (D) results in awkward, unidiomatic phrasing. The phrase because of should
be followed by a noun, not the participle learning. In addition, the verb to learn
usually does not imply any kind of duration; therefore, it is unidiomatic to say that
Nancy has been learning to ski since . . . age five.

Explanation for Incorrect Answer E :

Choice (E) results in a vague pronoun. There is nothing in the sentence to which the
singular pronoun it can logically refer.

3 Under the new regulations governing tobacco advertising, the well-known ad that
shows people riding horses smoking cigarettes will no longer be allowed.

From SAT Online Course CUUS


2007 May Sunday SAT Answers and Explanations

(A) that shows people riding horses smoking cigarettes


(B) that shows people smoking cigarettes while riding horses
(C) in which people are shown on horses smoking cigarettes while riding
(D) showing people riding and smoking cigarettes while on horses
(E) of people shown while riding horses and smoking cigarettes

ANSWERS AND EXPLA NATIONS

Explanation for Correct Answer B :

Choice (B) is correct. It avoids the modification error of the original by placing the
modifying phrase smoking cigarettes immediately after the subject (people) it
modifies.

Explanation for Incorrect Answer A :

Choice (A) involves a modification error. The sentence illogically implies that the horses
are smoking cigarettes.

Explanation for Incorrect Answer C :

Choice (C) results in a modification error. The sentence illogically implies that the
horses are smoking cigarettes.

Explanation for Incorrect Answer D :

Choice (D) results in awkward, redundant phrasing. The phrase people riding . . . while
on horses is awkward and redundant.

Explanation for Incorrect Answer E :

Choice (E) results in unidiomatic phrasing. It is not idiomatic to say that the well-
known ad of people shown.

4 Jared behaved like a person permanently on stage, he was always speaking in a


dramatic voice and looking around to see who was listening.

(A) he was always speaking


(B) always he would speak
(C) and he always spoke
(D) always having spoken
(E) always speaking

ANSWERS AND EXPLA NATIONS

From SAT Online Course CUUS


2007 May Sunday SAT Answers and Explanations

Explanation for Correct Answer E :

Choice (E) is correct. It avoids the comma splice of the original by recasting the second
independent clause as the modifying phrase always speaking . . .

Explanation for Incorrect Answer A :

Choice (A) involves a comma splice. Two independent clauses (Jared behaved like a
person permanently on stage and he was always speaking in a dramatic voice . . .
was listening ) are joined by only a comma.

Explanation for Incorrect Answer B :

Choice (B) results in a comma splice and a lack of parallelism. Two independent clauses
(Jared behaved like a person permanently on stage and always he would speak in a
dramatic voice . . . was listening ) are joined by only a comma. In addition, the verb
phrase would speak is not parallel with the following verb phrase looking around.

Explanation for Incorrect Answer C :

Choice (C) results in a subordination error and a lack of parallelism. The second clause
(and he always spoke in a dramatic voice . . .) is not grammatically equal to the first
clause (Jared behaved like a person permanently on stage), and so the two clauses
should not be joined with a comma and the conjunction and. In addition, the verb
phrase always spoke is not parallel with the following verb phrase looking around.

Explanation for Incorrect Answer D :

Choice (D) results in an improper verb form. The present perfect progressive verb
phrase having spoken is not parallel with the following present progressive verb
looking.

5 The Wooden Canoe Heritage Association is devoted to study, building, restoring


and use of the traditional North American canoe.

(A) is devoted to study, building, restoring, and use of


(B) is devoting itself to the study of, building and restoring, and using
(C) is devoted to studying, building, restoring, and using
(D) has been devoting itself to the study, building, restoration, and using
of
(E) has been devoted to studying, building, restoration, and how we use

ANSWERS AND EXPLA NATIONS

Explanation for Correct Answer C :

From SAT Online Course CUUS


2007 May Sunday SAT Answers and Explanations

Choice (C) is correct. It avoids the lack of parallelism of the original by providing the
gerunds studying, building, restoring, and using.

Explanation for Incorrect Answer A :

Choice (A) involves a lack of parallelism. The infinitive verb study and the noun use
are not parallel with the gerunds building and restoring.

Explanation for Incorrect Answer B :

Choice (B) results in a lack of parallelism and wordiness. The phrase the study of is
not parallel with the gerunds building, restoring, and using. In addition, the
conjunction and between building and restoring is not necessary and should be
replaced with a comma.

Explanation for Incorrect Answer D :

Choice (D) results in unidiomatic phrasing. It is not idiomatic to say that the association
has been devoting itself to the . . . using of. The appropriate idiomatic phrase is the
use of.

Explanation for Incorrect Answer E :

Choice (E) results in a lack of parallelism. The noun restoration and the phrase how
we use are not parallel with the gerunds studying and building.

6 On many college campuses, the study of film has become as common as that of
the novel.

(A) as common as that of the novel


(B) like the novel, as common
(C) common, as is that of the novel
(D) as common as the novels study
(E) just as the novels study is common

ANSWERS AND EXPLA NATIONS

Explanation for Correct Answer A :

Choice (A) is correct. It avoids the errors of the other options by creating a logical
comparison between the study of film and that of the novel.

Explanation for Incorrect Answer B :

Choice (B) results in illogical and unidiomatic phrasing. It does not make sense to say t

From SAT Online Course CUUS


2007 May Sunday SAT Answers and Explanations

hat the study of film has become like the novel. In addition, it is unidiomatic to end
the sentence with the phrase as common.

Explanation for Incorrect Answer C :

Choice (C) results in a lack of parallelism. The present-tense verb is that follows the
conjunction as is not parallel with the verb phrase has become, which precedes the
conjunction as.

Explanation for Incorrect Answer D :

Choice (D) results in awkward phrasing. The phrase the novels study is awkward and
should be replaced by the study of the novel.

Explanation for Incorrect Answer E :

Choice (E) results in awkward, unidiomatic phrasing. It is awkward and unidiomatic to


say the study of film has become just as the novels study is common.

7 Milkweed has many varieties, and one of these provide the monarch butterfly with
a place to lay its eggs.

(A) Milkweed has many varieties, and one of these provide


(B) The many varieties of milkweed, one of which provides
(C) One of the many varieties of milkweed provides
(D) One of many varieties of milkweed provide
(E) There are the many varieties of milkweed, and one of them provide

ANSWERS AND EXPLA NATIONS

Explanation for Correct Answer C :

Choice (C) is correct. It avoids the subject-verb disagreement of the original by


providing the singular verb provides for the singular subject One.

Explanation for Incorrect Answer A :

Choice (A) involves subject-verb disagreement. The singular subject one does not
agree with the plural verb provide.

Explanation for Incorrect Answer B :

Choice (B) results in a sentence fragment. There is no main verb to carry out the action
of the sentence, only the modifying phrase beginning with one of which provides.

From SAT Online Course CUUS


2007 May Sunday SAT Answers and Explanations

Explanation for Incorrect Answer D :

Choice (D) results in subject-verb disagreement. The singular subject One does not
agree with the plural verb provide.

Explanation for Incorrect Answer E :

Choice (E) results in subject-verb disagreement. The singular subject one does not
agree with the plural verb provide.

8 In his novels, Thackeray displayed his consciousness of the world about him, as
does Dickens, but he sees the world from an entirely different vantage point.

(A) as does Dickens, but he sees


(B) as with Dickens, but he saw
(C) as Dickens, but Thackeray sees
(D) as in those of Dickens, but he has seen
(E) as did Dickens, but Thackeray saw

ANSWERS AND EXPLA NATIONS

Explanation for Correct Answer E :

Choice (E) is correct. It avoids the tense shift and vague pronoun of the original by
providing consistent past-tense verbs and by replacing the vague pronoun he with the
proper noun Thackeray.

Explanation for Incorrect Answer A :

Choice (A) results in a tense shift and a vague pronoun. The sentence inappropriately
shifts from past tense (displayed) to present tense (does and sees ). In addition, it
is unclear to whom the pronoun he is referring.

Explanation for Incorrect Answer B :

Choice (B) results in a lack of parallelism and a vague pronoun. The conjunction as
inappropriately links the subject-and-verb phrase Thackeray displayed with the
prepositional phrase with Dickens. In addition, it is unclear to whom the pronoun he
is referring.

Explanation for Incorrect Answer C :

Choice (C) results in a lack of parallelism and a tense shift. The conjunction as
inappropriately links the subject-and-verb phrase Thackeray displayed with the noun
Dickens. In addition, the present-tense verb sees is inconsistent with the preced ing

From SAT Online Course CUUS


2007 May Sunday SAT Answers and Explanations

past-tense verb displayed.

Explanation for Incorrect Answer D :

Choice (D) results in a lack of parallelism and a vague pronoun. The conjunction as
inappropriately links the subject-and-verb phrase Thackeray displayed with the
prepositional phrase in those of Dickens. In addition, it is unclear to whom the
pronoun he is referring.

9 Though three-fourths of Kenya is a barren plain, the highland area, where most
people live, is farmed extensively.

(A) the highland area, where most people live, is farmed extensively
(B) the highland area is where most people lived and it is extensively farmed
(C) where most people live is in the highland area, it is extensively farmed
(D) where most people live is in the highland area, extensively farmed
(E) most people live and extensively farm the highland area

ANSWERS AND EXPLA NATIONS

Explanation for Correct Answer A :

Choice (A) is correct. It avoids the errors of the other options by providing an
appropriate verb tense and modifying phrase.

Explanation for Incorrect Answer B :

Choice (B) results in inconsistent verb tenses. The past-tense verb lived is
inconsistent with the present-tense verb phrase is extensively farmed.

Explanation for Incorrect Answer C :

Choice (C) results in awkward phrasing and a coordination error. The phrase where
most people live is in the highland area is awkward and unidiomatic. In addition, the
clauses Though three-fourths of Kenya is a barren plain, where most people live is in
the highland area and it is extensively farmed are improperly joined by only a
comma.

Explanation for Incorrect Answer D :

Choice (D) results in awkward, unidiomatic phrasing. The phrase where most people
live is in the highland area is awkward and unidiomatic. In addition, the relationship
between the highland area and the modifying phrase extensively farmed should be
shown as a nonrestrictive clause (the highland area, which is extensively farmed).

From SAT Online Course CUUS


2007 May Sunday SAT Answers and Explanations

Explanation for Incorrect Answer E :

Choice (E) results in awkward phrasing. The phrase most people live . . . the highland
area requires that the preposition in precede the highland area.

10 Since they are advised to write what they know about, first novels tend to be
heavily autobiographical.

(A) Since they are advised to write what they know about
(B) Being advised to write about what they know
(C) Based on advice to beginning writers to write about what they know
(D) Advising beginning writers to write about what they know
(E) Because beginning writers are advised to write about what they know

ANSWERS AND EXPLA NATIONS

Explanation for Correct Answer E :

Choice (E) is correct. It avoids the vague pronoun of the original by providing the noun
phrase beginning writers as the referent for the pronoun they.

Explanation for Incorrect Answer A :

Choice (A) involves a vague pronoun. There is nothing in the sentence to which the
plural pronoun they can logically refer.

Explanation for Incorrect Answer B :

Choice (B) results in a vague pronoun. There is nothing in the sentence to which the
plural pronoun they can logically refer.

Explanation for Incorrect Answer C :

Choice (C) results in a modification error. It does not make sense to say that first
novels are Based on advice to beginning writers.

Explanation for Incorrect Answer D :

Choice (D) results in a modification error. It does not make sense to say that first
novels are Advising beginning writers to write about what they know.

11 The proposed methods for controlling pollution were presented more as temporary
remedies than as permanent solutions to the problem.

From SAT Online Course CUUS


2007 May Sunday SAT Answers and Explanations

(A) more as temporary remedies than as


(B) as temporary remedies more than
(C) more for temporary remedies than
(D) for temporary remedies more than
as
(E) as more temporary remedies than

ANSWERS AND EXPLA NATIONS

Explanation for Correct Answer A :

Choice (A) is correct. It avoids the errors of the other options by providing the parallel
comparison phrase more as temporary remedies than as permanent solutions.

Explanation for Incorrect Answer B :

Choice (B) results in awkward phrasing. The comparison phrase as temporary


remedies more than permanent solutions is incomplete and awkward without as
preceding permanent solutions.

Explanation for Incorrect Answer C :

Choice (C) results in an unidiomatic preposition. It is not idiomatic to say that the
methods were presented for temporary remedies. The idiomatic preposition to follow
presented in this context is as.

Explanation for Incorrect Answer D :

Choice (D) results in an unidiomatic preposition. It is not idiomatic to say that the
methods were presented for temporary remedies. The idiomatic preposition to follow
presented in this context is as.

Explanation for Incorrect Answer E :

Choice (E) results in awkward phrasing. It does not make sense to describe the
proposed methods for controlling pollution as more temporary. In addition, this
phrasing causes permanent solutions to the problem to change illogically from a
hypothetic al standard to concrete solutions that already exist.

12 Caricaturists evoke humor by blending the realistic with the comedy in portraits.

(A) by blending the realistic with the comedy


(B) by blending the realistic and the comic
(C) in a blending of realism with the comic

From SAT Online Course CUUS


2007 May Sunday SAT Answers and Explanations

(D) when their blending reality with the comic


(E) through blends of reality and comic

ANSWERS AND EXPLA NATIONS

Explanation for Correct Answer B :

Choice (B) is correct. It avoids the lack of parallelism of the original by providing two
adjectives functioning as nouns (the realistic and the comic) as the compound
object of blending.

Explanation for Incorrect Answer A :

Choice (A) involves a lack of parallelism. The noun phrase the comedy is not parallel
with the preceding phrase the realistic, which is an adjective functioning as a noun.

Explanation for Incorrect Answer C :

Choice (C) results in a lack of parallelism. The phrase the comic, which is an adjective
functioning as a noun, is not parallel with the preceding noun realism.

Explanation for Incorrect Answer D :

Choice (D) results in faulty sentence structure. There is no main verb to carry out the
action of the clause their blending reality with the comic in portraits.

Explanation for Incorrect Answer E :

Choice (E) results in a lack of parallelism. In the phrase reality and comic, which is
the compound object of the preposition of, the adjective comic is not parallel with
the noun reality.

13 We now realize that the Earth has limited resources which, if they are wasted, you
put everyone in danger.

(A) which, if they are wasted, you put everyone in danger


(B) and if you waste them it puts everyone in danger
(C) and wasting it will be perilous
(D) and it puts everyone in danger to waste them
(E) that we waste at our own peril

ANSWERS AND EXPLA NATIONS

Explanation for Correct Answer E :

From SAT Online Course CUUS


2007 May Sunday SAT Answers and Explanations

Choice (E) is correct. It avoids the point-of-view error of the original by maintaining a
first-person plural (we) point of view.

Explanation for Incorrect Answer A :

Choice (A) involves a point-of-view error. The sentence inappropriately shifts from a
first-person plural point of view (we) to a second-person point of view (you).

Explanation for Incorrect Answer B :

Choice (B) results in a point-of-view error. The sentence inappropriately shifts from a
first-person plural point of view (we) to a second-person point of view (you).

Explanation for Incorrect Answer C :

Choice (C) results in a vague pronoun. There is nothing in the sentence to which the
singular pronoun it can logically refer.

Explanation for Incorrect Answer D :

Choice (D) results in unidiomatic phrasing. The pronoun it can only logically refer to
the following phrase to waste them, but this type of structure needs a linking verb (it
is dangerous to waste them ), not the verb puts.

14 Digging at the site, all power was cut off when the backhoe hit an electric cable;
fortunately, no one was hurt.

(A) Digging at the site, all power was cut off when the backhoe hit an electric cable
(B) Digging at the site, an electric cable was hit by a backhoe, cutting off all power
(C) Cutting off all power, the backhoe hit an electric cable digging at the site
(D) All power was cut off when the backhoe digging at the site hit an electric cable
(E) When digging at the site, an electric cable hit by the backhoe cut off all power

ANSWERS AND EXPLA NATIONS

Explanation for Correct Answer D :

Choice (D) is correct. It avoids the modification error of the original by placing the
modifying phrase (digging at the site) immediately after the subject it modifies
(backhoe).

Explanation for Incorrect Answer A :

Choice (A) involves a modification error. It does not make sense to say that all power
was Digging at the site.

From SAT Online Course CUUS


2007 May Sunday SAT Answers and Explanations

Explanation for Incorrect Answer B :

Choice (B) results in a modification error. It does not make sense to say that an
electric cable was Digging at the site.

Explanation for Incorrect Answer C :

Choice (C) results in a modification error. It does not make sense to say that an
electric cable was digging at the site.

Explanation for Incorrect Answer E :

Choice (E) results in a modification error. It does not make sense to say an electric
cable cut off power When digging at the site.

From SAT Online Course CUUS

You might also like